/
ESAP-ITE 2017 Slide Deck ESAP-ITE 2017 Slide Deck

ESAP-ITE 2017 Slide Deck - PowerPoint Presentation

tatyana-admore
tatyana-admore . @tatyana-admore
Follow
487 views
Uploaded On 2019-12-25

ESAP-ITE 2017 Slide Deck - PPT Presentation

ESAPITE 2017 Slide Deck ADRENAL ITE 2017 Question 1 A 48yearold woman has sustained 2 left metatarsal fractures in the past 18 months and is found to have low bone density mean total hip T score of 24 She has a history of poorly controlled hypertension and her cardiologist performs a che ID: 771464

mmol answer patients pmid answer mmol pmid patients diabetes patient therapy thyroid risk rationale insulin disease glucose weight treatment

Share:

Link:

Embed:

Download Presentation from below link

Download Presentation The PPT/PDF document "ESAP-ITE 2017 Slide Deck" is the property of its rightful owner. Permission is granted to download and print the materials on this web site for personal, non-commercial use only, and to display it on your personal computer provided you do not modify the materials and that you retain all copyright notices contained in the materials. By downloading content from our website, you accept the terms of this agreement.


Presentation Transcript

ESAP-ITE 2017 Slide Deck

ADRENAL

ITE 2017 Question 1 A 48-year-old woman has sustained 2 left metatarsal fractures in the past 18 months and is found to have low bone density (mean total hip T score of –2.4). She has a history of poorly controlled hypertension, and her cardiologist performs a chest CT to exclude coarctation of the aorta (negative), but a 1.4-cm left adrenal nodule (2 Hounsfield units) is discovered. Her medications are lisinopril , amlodipine, and metoprolol. On physical examination, her blood pressure is 152/96 mm Hg and pulse rate is 64 beats/min. Her height is 64 in (162.6 cm), and weight is 136 lb (61.8 kg) (BMI = 23.3 kg/m2). Examination findings are normal. Laboratory test results: Electrolytes, normal Serum aldosterone = 9 ng/ dL (1-21 ng/ dL ) (SI: 249.7 pmol /L [27.7-285.5 pmol /L]) Plasma renin activity = 7.3 ng/mL per h (0.6-4.3 ng/mL per h)

Which of the following would be the most sensitive test for the diagnosis of autonomous cortisol secretion from this nodule? Early-morning serum cortisol measurement 24-hour urinary free cortisol measurement Late-night salivary cortisol measurement MRI of adrenal glands Overnight 1-mg dexamethasone suppression test Correct Answer: E Learning objective : Recommend the low-dose dexamethasone suppression test as the most sensitive test for establishing the diagnosis of adrenal-dependent hypercortisolism .

Rationale: Adrenal nodules are seen in approximately 3% to 4% of adults who have CT of the chest or abdomen. Most of the nodules (particularly those smaller than 4 cm) are benign; however, 3% to 5% may represent unrecognized pheochromocytoma (attenuation value >15 Hounsfield units). Approximately 10% to 20% of patients with adrenal nodules, particularly those with low attenuation values, have evidence of mild ACTH-independent hypercortisolism . This condition is often characterized as subclinical Cushing syndrome. Nonetheless, many of these patients have clinical or metabolic derangements related to longstanding, unrecognized mild hypercortisolism. These patients often have low bone density, with fractures, hypertension, and other features of the metabolic syndrome. All patients with adrenal nodules larger than 1 cm should certainly undergo diagnostic evaluation to exclude a functioning lesion. The most reliable and sensitive way to exclude adrenal-dependent hypercortisolism is with the overnight 1-mg dexamethasone suppression test (Answer C). There is no consensus on what constitutes a normal postdexamethasone cortisol value. Many guidelines have recommended that cortisol should suppress to less than 1.8 µg/ dL (<49.7 nmol /L) after this test. However, some believe that a higher cortisol cutoff should be used in patients with incidentally discovered adrenal nodules. Adrenal-dependent hypercortisolism is characterized by low basal ACTH and an attenuated ACTH response to corticotropin -releasing hormone. The suppressed ACTH lowers adrenal androgen production and most patients have a low DHEA-S level.

Rationale Continued: Laboratory test results of the described patient: Serum cortisol after 1-mg dexamethasone suppression test = 8.1 µg/ dL (SI: 223.5 nmol/L)Basal ACTH = 2 pg/mL (10-60 pg /mL) (SI: 0.44 pmol /L [2.2-13.2 pmol /L]) DHEA-S = 16 µg/ dL (18-244 µg/ dL ) (SI: 0.43 µ mol /L [0.49-6.61 µ mol /L]) Late-night salivary cortisol = 0.09 µg/ dL (<0.13 µg/ dL ) (SI: 2.5 nmol /L [<3.6 nmol /L]) Urinary free cortisol = 11 µg/24 h (4-50 µg/24 h) (SI: 30.5 nmol /d [11-138 nmol /d]) She developed secondary adrenal insufficiency after laparoscopic left adrenalectomy. Her hypertension resolved. Although a 24-hour urinary free cortisol (Answer B) and late-night salivary cortisol (Answer E) may be helpful in this patient’s evaluation, these studies provide inadequate sensitivity to detect mild adrenal-dependent hypercortisolism . Early-morning measurement of cortisol (Answer A) does not discriminate between patients with or without hypercortisolism regardless of the cause. MRI of the adrenal glands (Answer D) does not establish the functional significance of an adrenal nodule.

Reference(s): Reincke M. Subclinical Cushing’s syndrome. Endocrinol Metab Clin North Am. 2000;29(1):43-56. PMID: 10732263Tsagarakis S, Vassiliadi D, Thalassinos N. Endogenous subclinical hypercortisolism: diagnostic uncertainties and clinical implications. J Endocrinol Invest. 2006;29(5):471-482. PMID: 16794373Deutschbein T, Unger N, Hinrichs J, Walz MK, Mann K, Petersenn S. Late-night and low-dose dexamethasone-suppressed cortisol in saliva and serum for the diagnosis of cortisol-secreting adrenal adenomas. Eur J Endocrinol . 2009;161(5):747-753. PMID: 19726566 Masserini B, Morelli V, Bergamaschi S, et al. The limited role of midnight salivary cortisol levels in the diagnosis of subclinical hypercortisolism in patients with adrenal incidentaloma. Eur J Endocrinol . 2009;160(1):87-92. PMID: 18835977

ITE 2017 Question 20 An 18-year-old man is referred to you because bilateral adrenal masses were identified on abdominal CT performed to evaluate intermittent abdominal pain (see image). Axial view of abdominal CT. Bilateral adrenal masses (arrows) are documented with density values of 28 to 42 Hounsfield units.

He has no notable medical history and is unaware of any relevant family history. Physical examination findings and general appearance are normal. His resting pulse rate is 88 beats/min, and blood pressure is 142/95 mm Hg (supine) and 112/78 mm Hg (standing). His height is 72 in (182.9 cm), and weight is 185 lb (84.1 kg) (BMI = 25.1 kg/m2). Laboratory test results: Serum aldosterone = 14 ng/ dL (1-21 ng/dL) (SI: 388.4 pmol /L [27.7-582.5 pmol /L]) Plasma renin activity = 2.4 ng/mL per h (0.6-4.3 ng/mL per h) Serum cortisol after overnight 1-mg dexamethasone suppression test = 0.9 μ g/ dL (SI: 24.8 nmol /L) Serum calcium = 8.9 mg/ dL (8.2-10.2 mg/ dL ) (SI: 2.2 mmol /L [2.1-2.6 mmol /L]) Urinary catecholamines : Epinephrine = 21 µg/24 h (<35 µg/24 h) (SI: 114.5 nmol /d [<191 nmol /d]) Norepinephrine = 1425 µg/24 h (<170 µg/24 h) (SI: 8427.5 nmol /d [<1005 nmol /d]) Urinary metanephrines : Metanephrine = 232 µg/24 h (<400 µg/24 h) (SI: 1176.2 nmol /d [<2028 nmol /d]) Normetanephrine = 3567 µg/24 h (<900 µg/24 h) (SI: 19,475.8 nmol /d [<4914 nmol /d])

Given his age, he is offered genetic screening. Which of the following genes is most likely to harbor a mutation? Correct Answer: E Learning objective : Determine the most likely underlying genetic syndrome associated with pheochromocytoma on the basis of clinical, imaging, and biochemical findings. RET proto-oncogene NF1 MEN1 SDHD VHL

Rationale: Pheochromocytoma is rare and occurs in 0.2% to 0.6% of patients with hypertension in the outpatient setting. Although most cases of pheochromocytoma are sporadic, more than 10% of tumors (probably closer to 25%) have an underlying genetic basis. This is more likely in younger patients, and genetic screening should be routinely offered to patients with pheochromocytoma who are younger than 50 years even in the absence of other affected family members. The genetic syndromes associated with pheochromocytoma are summarized (see table). Pheochromocytoma occurs in up to 50% of persons with multiple endocrine neoplasia type 2 (MEN 2; due mutations in the RET proto-oncogene), in 10% to 20% of persons with von Hippel–Lindau syndrome (VHL; due to mutations in the VHL gene), and in 0.1% to 5% of persons with neurofibromatosis type 1 (NF 1; due to mutations in the NF1 gene). The clinical phenotype of pheochromocytoma associated with VHL and MEN 2 differs, largely because of differences in biochemical phenotype. Persons with MEN 2 tend to be more symptomatic with a higher incidence of hypertension because the tumors in such patients express phenylethanolamine N-methyltransferase (the enzyme that converts norepinephrine to epinephrine), as well as tyrosine hydroxylase (rate-limiting enzyme of catecholamine biosynthesis). As a result, pheochromocytomas in patients with MEN 2 often secrete epinephrine, while those in patients with VHL produce higher levels of norepinephrine .

Rationale Continued: The patient in this clinical vignette, therefore, is more likely to have a mutation in the VHL gene (Answer E) than in the RET proto-oncogene associated with MEN 2 (Answer A). SDHD mutations (Answer D) are usually associated with paraganglioma; pheochromocytoma has only rarely been reported. NF 1 would be suspected only if the clinical phenotype were suggestive. In the vignette, the patient was not noted to have neurofibromas, axillary freckling, or café-au- lait spots; thus, an NF1 mutation (Answer B) is unlikely. MEN 1 (due to mutations in the MEN1 gene [Answer C]) is not routinely associated with pheochromocytoma and the patient’s normal serum calcium concentration would also make this an unlikely diagnosis.

    Syndrome Gene and Mode of Inheritance     Clinical Features   Risk of Pheochromocytoma   Risk of malignancy Multiple endocrine neoplasia type 2A and 2B RET proto-oncogene   Autosomal dominant Medullary thyroid cancer, primary hyperparathyroidism, pheochromocytoma (MEN 2A)   Medullary thyroid cancer, pheochromocytoma, marfanoid habitus, mucosal neuromas, ganglioneuromatosis of the gastrointestinal tract (MEN 2B) 50%   Usually benign von Hippel–Lindau syndrome VHL Autosomal dominantRetinal angioma, cerebellar and spinal hemoangioblastoma, clear cell renal cell carcinoma, renal and pancreatic cysts, pheochromocytoma10%-20% 5%Neurofibromatosis type 1NF1  Autosomal dominantCafé-au-lait macules, axillary freckling, cutaneous neurofibromas, Lisch nodules, optic glioma, pheochromocytoma<5%UnknownFamilial paraganglioma SDHB  Autosomal dominantExtra-adrenal catecholamine-secreting paraganglioma, pheochromocytoma, gastrointestinal stromal tumor, head and neck paraganglioma69% have disease by age 60 years80% lifetime risk 30%-70%Familial paragangliomaSDHC  Autosomal dominantHead and neck paraganglioma, gastrointestinal stromal tumorRareUnknownFamilial paragangliomaSDHD  Autosomal dominant with maternal imprintingHead and neck paraganglioma, benign extra-adrenal catecholamine-secreting paraganglioma, pheochromocytoma,gastrointestinal stromal tumorRareUnknownFamilial paragangliomaSDHAF2 Autosomal dominantHead and neck paragangliomaUnknownUnknown Table. Genetic Syndromes Associated with Pheochromocytoma

Reference(s): Waguespack SG, Rich T, Grubbs E, et al. A current review of the etiology, diagnosis, and treatment of pediatric pheochromocytoma and paraganglioma. J Clin Endocrinol Metab. 2010;95(5):2023-2037. PMID: 20215394Jimenez C, Cote G, Arnold A, Gagel RF. Review: should patients with apparently sporadic pheochromocytomas or paragangliomas be screened for hereditary syndromes? J Clin Endocrinol Metab. 2006;91(8):2851-2858. PMID: 16735498Lenders JW, Duh QY, Eisenhofer G, et al; Endocrine Society. Pheochromocytoma and paraganglioma: an Endocrine Society clinical practice guideline. J Clin Endocrinol Metab . 2014;99(6):1915-1942. PMID: 24893135 Neumann HP, Bausch B, McWhinney SR, et al; Freiburg-Warsaw-Columbus Pheochromocytoma Study Group. Germ-line mutations in nonsyndromic pheochromocytoma. N Engl J Med. 2002;346(19):1459-1466. PMID: 12000816Eisenhofer G, Walther MM, Huynh TT, et al. Pheochromocytomas in von Hippel-Lindau syndrome and multiple endocrine neoplasia type 2 display distinct biochemical and clinical phenotypes. J Clin Endocrinol Metab. 2001;86(5):1999-2008. PMID: 11344198Dluhy RG. Pheochromocytoma--death of an axiom. N Engl J Med. 2002;346(19):1486-1488. PMID: 12000821

ITE 2017 Question 26 A 33-year-old man has been referred to you for evaluation of Cushing syndrome. He has an 11-year history of HIV infection. He is taking antiretroviral therapy, including didanosine , tenofovir , and ritonavir. His viral load is zero and his CD4 cell count is normal. He has developed asthma in the past 6 months and has been taking fluticasone/salmeterol twice daily and an albuterol metered-dose inhaler as needed. He has gained 30 lb (13.6 kg) in the past 3 months. Hypertension and fasting hyperglycemia have developed. On physical examination, he has a very cushingoid appearance, with wide, violaceous abdominal striae and proximal muscle weakness.

Which of the following laboratory profiles is most likely to be found in this man? Correct Answer: D Learning objective : Diagnose exogenous Cushing syndrome in patients receiving a combination of inhaled steroids and drugs that inhibit CYP3A4, particularly antiretroviral therapy. Answer Plasma ACTH DHEA-S Cortisol A. Normal Normal Normal B. ⇑ ⇑ ⇑ C. ⇓ ⇓ ⇑ D. ⇓ ⇓ ⇓ E.⇑⇓⇓

Rationale: When considering the clinical diagnosis of Cushing syndrome, taking a thorough history regarding possible exogenous glucocorticoid exposure is essential. Inhaled, oral, ingested, and topical steroids should all be considered as potential causes of Cushing syndrome. In addition, potent progestational agents, such as medroxyprogesterone and megestrol acetate, should be included because these steroids may also bind to the glucocorticoid receptor. Most glucocorticoids are substrates for liver metabolism with CYP3A4. Inhibitors of this enzyme can decrease the clearance of steroids and amplify their clinical impact. There is a very long list of drugs that inhibit CYP3A4, including protease inhibitors, macrolides, diltiazem, verapamil, and itraconazole. The patient described here was receiving inhaled corticosteroids (fluticasone) and an antiretroviral medication (ritonavir) that is a potent inhibitor of CYP3A4. The combination of inhaled steroids and protease inhibitors causing iatrogenic Cushing syndrome is well recognized. The inhaled steroids are very potent, with a high affinity for the glucocorticoid receptor. Accordingly, patients may have suppression of the hypothalamic-pituitary-adrenal axis, with subnormal levels of ACTH, cortisol, and adrenal androgen production (Answer D). A change in therapy to an antiretroviral agent that does not inhibit CYP3A4 will resolve the exogenous Cushing syndrome and help facilitate recovery of the hypothalamic-pituitary-adrenal axis.

Rationale Continued: Antiretroviral therapy in patients with HIV infection has also been associated with a lipodystrophy syndrome, and some of its signs, such as fat redistribution and central adiposity, overlap with characteristics of Cushing syndrome. Patients with the lipodystrophy syndrome do not have disturbance of pituitary-adrenal function (Answer A). The lipodystrophy syndrome does not cause rapid weight gain, violaceous striae , proximal myopathy, or edema. Fat atrophy is a hallmark clinical finding. Although it was initially thought that chronic HIV infection might itself cause deterioration in adrenocortical function, longitudinal studies (before effective antiretroviral therapies) provided no convincing evidence of any progressive cortisol deficiency (Answer E). Although there is nothing about HIV infection per se that would necessarily preclude the development of endogenous Cushing syndrome from an adrenal neoplasm or an ACTH-secreting neoplasm (Answers B and C), the rapid onset of signs and symptoms of Cushing syndrome after the initiation of inhaled corticosteroids supports the diagnosis of iatrogenic Cushing syndrome.

Reference(s): Foisy MM, Yakiwchuk EM, Chiu I, Singh AE. Adrenal suppression and Cushing’s syndrome secondary to an interaction between ritonavir and fluticasone: a review of the literature. HIV Med . 2008;9(6):389-396. PMID: 18459946Samaras K, Pett S, Gowers A, McMurchie M, Cooper DA. Iatrogenic Cushing’s syndrome with osteoporosis and secondary adrenal failure in human immunodeficiency virus-infected patients receiving inhaled corticosteroids and ritonavir-boosted protease inhibitors: six cases. J Clin Endocrinol Metab. 2005;90(7):4394-4398. PMID: 15755851Grinspoon S, Carr A. Cardiovascular risk and body-fat abnormalities in HIV-infected adults. N Engl J Med . 2005;352(1);48-62. PMID: 15635112 Findling JW, Buggy BP, Gilson IH, Brummitt CF, Bernstein BM, Raff H. Longitudinal evaluation of adrenocortical function in patients infected with the human immunodeficiency virus. J Clin Endocrinol Metab . 1994;79(4):1091-1096. PMID: 7962279

ITE 2017 Question 36 A 55-year-old woman is receiving adjuvant mitotane therapy after left adrenalectomy for a 6-cm adrenocortical cancer. She takes 1 g 4 times daily, and her last serum mitotane value was therapeutic at 15 mg/L. She also takes hydrocortisone, 60 mg on upon waking in the morning and 30 mg in the midafternoon. A serum cortisol level 4 hours after the morning dose is 7 μg / dL (193.1 nmol/L).

Which of the following mechanisms explains the need for higher hydrocortisone dosages in this patient? Decreases corticosteroid-binding globulin Induces hepatic CYP3A4 activity Impairs cortisol absorption Increases urinary cortisol excretion Increases 5α- reductase activity Correct Answer: B Learning objective : Adjust the dose of hydrocortisone during mitotane therapy based on changes in metabolism.

Rationale: Mitotane therapy for adrenal cancer lowers cortisol production by several mechanisms besides direct adrenal toxicity. Mitotane is one of the most potent inducers of CYP3A4 and drug metabolism (thus, Answer B is correct). CYP3A4 also metabolizes hydrocortisone, primarily by 6β-hydroxylation, and 6β- hydroxycortisol production increases 10-fold with mitotane therapy. While mitotane actually increases (not decreases) corticosteroid-binding globulin (thus, Answer A is incorrect), this effect would, if anything, slow cortisol metabolism. Mitotane has no significant effect on cortisol (hydrocortisone) absorption or urinary excretion (thus, Answers C and D are incorrect). Mitotane inhibits rather than induces 5α-reductase activity (thus, Answer E is incorrect), and because 5α-reduction is one means of cortisol catabolism, this action would also slow cortisol elimination.

Reference(s): Chortis V, Taylor AE, Schneider P, et al. Mitotane therapy in adrenocortical cancer induces CYP3A4 and inhibits 5  -reductase, explaining the need for personalized glucocorticoid and androgen replacement. J Clin Endocrinol Metab. 2013;98(1):161-171. PMID: 23162091Kroiss M, Quinkler M, Lutz WK, Allolio B, Fassnacht M. Drug interactions with mitotane by induction of CYP3A4 metabolism in the clinical management of adrenocortical carcinoma. Clin Endocrinol (Oxf). 2011;75(5):585-591. PMID: 21883349 Terzolo M, Zaggia B, Allasino , B, De Francia S. Practical treatment using mitotane for adrenocortical carcinoma. Curr Opin Endocrinol Diabetes Obes. 2014;21(3):159-165. PMID: 24732405

ITE 2017 Question 40 A 27-year-old pregnant woman seeks your advice at 6 weeks’ gestation. This is her first pregnancy. She has salt-wasting congenital adrenal hyperplasia due to 21-hydroxylase deficiency, for which she takes hydrocortisone and fludrocortisone. She is concerned that her baby may have the same condition.

Which of the following is the best next step in this patient’s management? Measure serum 17-hydroxyprogesterone Add dexamethasone, 0.5 mg at bedtime Obtain fetal DNA via chorionic villus sampling for CYP21A2 mutation analysis Perform amniocentesis to determine fetal sex Perform molecular genetic testing for CYP21A2 mutations in the father Correct Answer: E Learning objective : Counsel a pregnant woman with classic congenital adrenal hyperplasia regarding the risk to her fetus, bearing in mind that it is an autosomal recessive disorder.

Rationale: Although the prenatal treatment of classic congenital adrenal hyperplasia (CAH) is controversial, the patient’s concern is appropriate, and it is important to determine whether the fetus is at risk for CAH. Because CAH is an autosomal recessive disorder, the fetus is at risk of being affected only if the father has CAH or is a carrier of a CYP21A2 mutation. Thus, the most straightforward way to address this scenario is to perform molecular genetic testing for CYP21A2 mutations in the father (Answer E). If the father is a carrier of a CYP21A2 mutation, then the fetus has a 50% chance of inheriting CAH. Only female fetuses would benefit from in utero exposure to dexamethasone (to prevent virilization ). In theory, only half of affected fetuses are female, so treatment with dexamethasone would potentially benefit only 1 in 4 fetuses in the clinical setting discussed in this vignette.

Rationale Continued: Obtaining fetal DNA via chorionic villus for CYP21A2 mutation analysis (Answer C) should be considered if the baby’s father is determined to be a carrier of a CYP21A2 mutation. However, chorionic villus sampling cannot be done until 10 to 12 weeks’ gestation, and virilization of the female fetus begins 6 weeks after conception. Another approach (if the baby’s father has a documented CYP21A2 mutation) would be to determine fetal sex with amniocentesis (Answer D), chorionic villus sampling, or analysis for fetal Y-chromosomal DNA in maternal blood because only female fetuses would potentially require treatment. Dexamethasone (Answer B) crosses the placenta and can suppress the fetal hypothalamic-pituitary-adrenal axis. Dexamethasone should not be used to treat pregnant women with CAH unless it is documented that the fetus is at risk, and, even then, it has been recommended that it be used through protocols approved by institutional review boards at centers capable of collecting outcomes data on a sufficiently large number of patients, so that the risks and benefits of this treatment can be defined. Measuring the patient’s blood concentration of 17-hydroxyprogesterone (Answer A) would not affect clinical decision-making in this case. Finally, it should be noted that although a negative CYP21A2 panel in the father would be reassuring and would markedly reduce their chance of having an affected child, CYP21A2-targeted mutation analysis does not have a 100% detection rate; thus, unless full gene sequencing is performed, the father would still have a residual carrier risk.

Reference(s): Finkielstain GP, Chen W, Mehta SP, et al. Comprehensive genetic analysis of 182 unrelated families with congenital adrenal hyperplasia due to 21-hydroxylase deficiency. J Clin Endocrinol Metab. 2011;96(1):E161-E172. PMID: 20926536Hirvikoski T, Nordenstrom A, Lindholm T, et al. Cognitive functions in children at risk for congenital adrenal hyperplasia treated prenatally with dexamethasone. J Clin Endocrinol Metab. 2007;92(2):542-548. PMID: 17148562Speiser PW, Azziz R, Baskin LS, et al; Endocrine Society. Congenital adrenal hyperplasia due to steroid 21-hydroxylase deficiency: an Endocrine Society clinical practice guideline [published correction appears in J Clin Endocrinol Metab . 2010;95(11):5137]. J Clin Endocrinol Metab . 2010;95(9):4133-4160. PMID: 20823466

ITE 2017 Question 48 A 28-year-old woman in her 20th week of pregnancy is referred to you because of new-onset hypertension and hypokalemia. Before pregnancy she had mild hypertension, which was well controlled with a single agent. However, since she has entered her second trimester, the hypertension has become more difficult to control and her serum potassium levels have been low. Her medications include methyldopa, 250 mg twice daily, and amlodipine, 10 mg daily. On physical examination, she is a healthy appearing woman in her second trimester of pregnancy. Her blood pressure is 144/98 mm Hg, and pulse rate is 72 beats/min. Laboratory test results: Sodium = 140 mEq /L (136-142 mEq /L) (SI: 140 mmol /L [136-142 mmol /L]) Potassium = 3.2 mEq /L (3.5-5.0 mEq /L) (SI: 3.2 mmol /L [3.5-5.0 mmol/L]) Creatinine = 0.7 mg/ dL (0.6-1.1 mg/ dL ) (SI: 61.9 µ mol/L [53.0-97.2 µmol/L])Plasma aldosterone = 35 ng/dL (1-21 ng/dL) (SI: 970.9 pmol/L [27.7-582.5 pmol/L])Plasma renin activity = <0.6 ng/mL per h (0.6-4.3 ng/mL per h)Measurement of urinary protein is normal. Abdominal MRI without gadolinium administration shows a 1.4-cm left adrenal mass and a normal-appearing right adrenal gland.

In addition to adjusting her medications, which of the following is the best next step in this patient’s evaluation and management? Saline infusion test Adrenal venous sampling Iodocholesterol scintigraphy No additional testing needed; start spironolactone No additional testing needed; perform laparoscopic left adrenalectomy Correct Answer: E Learning objective : Recommend management with unilateral laparoscopic adrenalectomy during the second trimester in pregnant women with confirmed primary aldosteronism and a clear-cut unilateral adrenal macroadenoma (>10 mm).

Rationale: Primary aldosteronism is uncommon in pregnancy, with fewer than 40 patients reported in the medical literature. Most of these cases are due to aldosterone-producing adenomas. While pregnancy is associated with an increase in aldosterone, this is due to increased renin activity. Primary aldosteronism can lead to intrauterine growth retardation, preterm delivery, intrauterine fetal demise, and placental abruption. Case-detection testing for primary aldosteronism in pregnant women is the same as for nonpregnant patients: morning blood sample for the measurement of aldosterone and plasma renin activity or renin mass measurement. Suppressed renin and an aldosterone concentration greater than 15 ng/ dL (>416.1 pmol /L) is a positive case-detection test for primary aldosteronism . If spontaneous hypokalemia is present with a markedly increased plasma aldosterone concentration (>20 ng/ dL [>550 pmol /L]) and suppressed renin, confirmatory testing is not needed—this key point is highlighted in the May 2016 Endocrine Society clinical practice guidelines. However, in normokalemic patients with a positive case-detection test, confirmatory testing should be pursued. In the setting of pregnancy, confirmatory testing with a saline infusion test (Answer A) or measurement of sodium and aldosterone in a 24-hour urine collection while the patient is on a generous sodium diet could be considered. However, as mentioned, confirmatory testing is not needed in this patient with spontaneous hypokalemia and marked primary aldosteronism (aldosterone >20 ng/ dL [>550 pmol /L]).

Rationale Continued: In pregnant women with primary aldosteronism , subtype classification with abdominal MRI without gadolinium is the test of choice. Adrenal imaging with CT, iodocholesterol scintigraphy (Answer C), and adrenal venous sampling (Answer B) should be avoided in pregnancy. As highlighted in the revised Endocrine Society clinical practice guidelines on primary aldosteronism, adrenal venous sampling may not be needed in patients with vigorous primary aldosteronism who are younger than 35 years and have a clear-cut unilateral adrenal adenoma on cross-sectional imaging. Primary aldosteronism in pregnancy is fascinating in that the degree of disease may be improved or exacerbated by pregnancy. In some women with primary aldosteronism , the high blood levels of pregnancy-related progesterone are antagonistic at the mineralocorticoid receptor and partially block the action of aldosterone—these patients have an improvement in the manifestations of primary aldosteronism during pregnancy. In other pregnant women, increased expression of the LH choriogonadotropin receptor and the gonadotropin-releasing hormone receptor has been documented in aldosterone-producing adenomas, and the degree of hyperaldosteronism is exacerbated by the increased pregnancy-related blood levels of hCG .

Rationale Continued: The therapeutic strategy for primary aldosteronism in pregnancy depends on how difficult it is to manage the hypertension and hypokalemia. If the patient has a remission in the degree of primary aldosteronism , then surgery or treatment with a mineralocorticoid antagonist can be avoided until after delivery. However, if hypertension and hypokalemia are marked, then surgical and/or medical intervention is indicated. Unilateral laparoscopic adrenalectomy (Answer E) during the second trimester can be considered in those women with confirmed primary aldosteronism and a clear-cut unilateral adrenal macroadenoma (>10 mm). This is the best course of action for this patient. Spironolactone (Answer D) should be avoided in pregnancy because it crosses the placenta. It is a US FDA pregnancy category C drug, as feminization of newborn male rats has been documented. However, there is only 1 human case in the medical literature in which treatment with spironolactone in pregnancy led to ambiguous genitalia in a male infant—this occurred in a woman treated with spironolactone for polycystic ovary syndrome before pregnancy and through the fifth week of gestation. For those pregnant women who will be managed medically, hypertension should be treated with standard antihypertensive drugs approved for use during pregnancy. Hypokalemia, if present, should be treated with oral potassium supplements. For those patients with refractory hypertension and/or hypokalemia, the addition of eplerenone , a US FDA pregnancy category B drug, may be cautiously considered

Reference(s): Campino C, Trejo P, Carvajal CA, et al. Pregnancy normalized familial hyperaldosteronism type I: a novel role for progesterone? J Hum Hypertens. 2015;29(2):138-139. PMID: 24943290Eguchi K, Hoshide S, Nagashima S, Maekawa T, Sasano H, Kario K. An adverse pregnancy-associated outcome due to overlooked primary aldosteronism. Intern Med. 2014;53(21):2499-2504. PMID: 25366010Riester A, Reincke M. Progress in primary aldosteronism: mineralocorticoid receptor antagonists and management of primary aldosteronism in pregnancy. Eur J Endocrinol . 2015;172(1):R23-R30. PMID: 25163723 Ronconi V, Turchi F, Zennaro MC, Boscaro M, Giachetti G. Progesterone increase counteracts aldosterone action in a pregnant woman with primary aldosteronism. Clin Endocrinol (Oxf). 2011;74(2):278-279. PMID: 21054479Teo AE, Garg S, Shaikh LH, et al. Pregnancy, primary aldosteronism, and adrenal CTNNB1 mutations. N Engl J Med. 2015;373(15):1429-1436. PMID: 26397949Funder JW, Carey RM, Mantero F, et al. The management of primary aldosteronism: case detection, diagnosis, and treatment: an Endocrine Society clinical practice guideline. J Clin Endocrinol Metab . 2016; 101(5):1889-1916. PMID: 26934393

ITE 2017 Question 50 A 27-year-old man has been referred to you for a second opinion regarding endogenous Cushing syndrome. He has a 1-year history of hypertension and has gained 43 lb (19.5 kg) in the past 6 months. He also has facial fullness; muscle weakness; insomnia; and wide, violaceous striae . He is very cushingoid and has a blood pressure of 156/94 mm Hg despite treatment with lisinopril , hydrochlorothiazide, and amlodipine. Laboratory test results (ordered by another endocrinologist): Sodium = 138 mEq /L (136-142 mEq /L) (SI: 138 mmol /L [136-142 mmol /L]) Potassium = 2.9 mEq /L (3.5-5.0 mEq /L) (SI: 2.9 mmol /L [3.5-5.0 mmol /L]) Chloride = 98 mEq/L (96-106 mEq/L) (SI: 98 mmol/L [96-106 mmol/L])Serum urea nitrogen = 24 mg/dL (8-23 mg/dL) (SI: 8.6 mmol/L [2.9-8.2 mmol/L])Creatinine = 1.1 mg/dL (0.7-1.3 mg/dL) (SI: 97.2 µmol/L [61.9-114.9 µmol/L])Bicarbonate = 30 mEq/L (21-28 mEq/L) (SI: 30 mmol/L [21-28 mmol/L])Plasma ACTH = 102 pg/mL (10-60 pg/mL) (SI: 22.4 pmol/L [2.2-13.2 pmol/L])Urinary free cortisol = 1944 µg/24 h (4-50 µg/24 h) (SI: 5365 nmol/d [11-138 nmol/d])Late-night salivary cortisol = 2.4 µg/dL (<0.13 µg/dL) (SI: 66 nmol/L [<3.6 nmol/L])Serum cortisol after overnight 1-mg dexamethasone suppression test = 21.4 µg/dL (SI: 590 nmol/L)

MRI of the pituitary is normal. CT of the chest and abdomen shows a 1.5-cm calcified lesion in the right upper lung lobe and bilateral adrenal enlargement without nodules. A right upper lobectomy has been recommended.

Which of the following should you recommend? Bilateral inferior petrosal sinus sampling for ACTH High-dose dexamethasone suppression test (2-mg dexamethasone every 6 hours for 8 doses, with measurement of urine and plasma steroids) Octreotide acetate scintigraphy DDAVP stimulation test Proceeding with the recommended thoracic surgery Correct Answer: A Learning objective : Recommend inferior petrosal sinus sampling in the evaluation of patients with ACTH-dependent Cushing syndrome and a normal or equivocal pituitary imaging study.

Rationale: The differential diagnosis of ACTH-dependent Cushing syndrome is often challenging. Most affected patients harbor ACTH-secreting pituitary tumors (Cushing disease); therefore, pituitary MRI should always be the initial diagnostic study. However, 30% of 40% of patients with Cushing disease have either normal or equivocal pituitary imaging results. In addition, approximately 10% of healthy individuals have small pituitary imaging abnormalities. The only way to reliably distinguish a pituitary ACTH-secreting neoplasm from a nonpituitary ACTH-secreting neoplasm is with bilateral inferior petrosal sinus sampling for ACTH (Answer A). In the absence of an obvious nonpituitary ACTH-secreting neoplasm, patients should be promptly referred to a specialized center that can perform this procedure safely. High-dose dexamethasone suppression testing (Answer B) is not reliable in the differential diagnosis of ACTH-dependent Cushing syndrome and has been abandoned. The rapid onset of obvious endogenous Cushing syndrome and the prodigious hypercortisolism accompanied by hypokalemic metabolic alkalosis suggest the possibility of a nonpituitary neoplasm in this patient. The abnormality detected on chest imaging is an upper pulmonary lobe calcified lesion. Although a bronchial carcinoid may be calcified on rare occasions, a secure diagnosis of ectopic ACTH must be established before considering any thoracic surgery (Answer E). Octreotide acetate scintigraphy (Answer C) should be performed only if the diagnosis of ectopic ACTH has been secured by inferior petrosal sinus sampling. An exhaustive imaging search for a nonexistent ectopic ACTH-secreting neoplasm can only lead to surgical misadventures.

Rationale Continued: This patient had inferior petrosal sinus ACTH sampling with corticotropin -releasing hormone stimulation. He had a markedly elevated central-to-peripheral pituitary ACTH gradient that lateralized to the left petrosal sinus. Pituitary microsurgery was performed to remove a 3-mm left-sided corticotroph microadenoma; surgery was followed by secondary adrenal insufficiency and resolution of Cushing syndrome. Because some ectopic ACTH-secreting tumors (especially bronchial carcinoids) harbor vasopressin and corticotropin -releasing hormone receptors, peripheral DDAVP (Answer D) and corticotropin -releasing hormone stimulation tests cannot reliably distinguish pituitary from ectopic ACTH-dependent Cushing syndrome.

Reference(s): Javorsky BR, Carroll TB, Findling JW. Differential diagnosis of Cushing syndrome. In: Swearingen B, Biller BMK, eds. Cushing’s Disease . New York, NY: Springer; 2011: 85-106. Javorsky BR, Findling JW. Inferior petrosal sampling for the differential diagnosis of ACTH-dependent Cushing’s syndrome. In: Bronstein MD, ed. Cushing’s Syndrome: Pathophysiology, Diagnosis and Treatment. New York, NY: Humana Press, Inc; 2010: 105-120.

ITE 2017 Question 56 A 23-year-old man whom you have been treating for classic congenital adrenal hyperplasia returns to clinic after a prolonged absence. Shortly after birth, newborn screening led to the diagnosis of salt-losing congenital adrenal hyperplasia due to 21-hydroxylase deficiency. He was appropriately treated through infancy and childhood with hydrocortisone 3 times daily and fludrocortisone twice daily. Pubertal development was normal, and he has remained well. His current therapy consists of dexamethasone, 0.25 mg daily, and fludrocortisone, 0.1 mg daily. He now presents to reestablish care. He describes an enlarged and painful testis. On physical examination, his height is 69 in (175.3 cm), and weight is 205 lb (93.2 kg) (BMI = 30.3 kg/m2). His blood pressure is 127/64 mm Hg, and pulse rate is 78 beats/min. His skin is darker than you would expect for the season, and he has brown palmar creases. His right testis is normal on examination, but the left testis is enlarged and tender.

Laboratory test results: 17-Hydroxyprogesterone = 2600 ng/ dL (<220 ng/ dL ) (SI: 78.8 nmol/L [<6.67 nmol/L]) DHEA-S = 500 μg / dL (89-457 μg / dL ) (SI: 13.6 μmol /L [2.41-12.38 μmol /L]) Testosterone = 806 ng/dL (300-900 ng/dL ) (SI: 27.9 nmol /L [10.4-31.2 nmol /L]) ACTH = 745 pg/mL (10-60 pg/mL) (SI: 163.9 pmol/L [2.2-13.2 pmol/L])Basic metabolic panel, normal

In addition to ordering ultrasonography and counseling him on the importance of medication adherence, which of the following is the best next step? Refer for testicular resection Refer for testicular radiation Increase dexamethasone to 0.75 mg daily at bedtime Change dexamethasone to hydrocortisone, 20 mg daily in divided doses Add mitotane Correct Answer: C Learning objective : Manage testicular adrenal rest tumors in the setting of congenital adrenal hyperplasia.

Rationale: Testicular adrenal rest tumors are found in 80% to 90% of adolescent and adult males with classic congenital adrenal hyperplasia (CAH). These tumors generally present in the teenage years but they can be found at any age. Adrenal rest tumors are more common in salt-losing forms of CAH, especially in patients with poor adherence to medication regimens, which leads to poor control and higher ACTH levels. In general, testicular adrenal rest tumors are bilateral, and size ranges from 2 to 40 mm in diameter. These tumors may cause gonadal dysfunction, infertility, and/or pain. Patients can also have intra-abdominal adrenal tumors, but female patients rarely have ovarian adrenal rest tumors. The patient in this vignette most likely has had poor adherence to his medication regimen or is not on enough glucocorticoids, as confirmed by his elevated laboratory values. The ACTH elevation most likely contributed to the growth of the testicular rest tumor. The initial management is to restart glucocorticoid therapy or to increase his dosage (Answer C) given his elevated ACTH level. Nighttime administration may result in better ACTH suppression. Infertility may not resolve with steroids or with surgery. Therefore, because adrenal rest tumors are so common, general recommendations include periodic testicular ultrasonography starting in adolescence and semen analysis if infertility is present.

Rationale Continued: Changing from dexamethasone to hydrocortisone (Answer D) is not likely to have any additional benefit. In individuals such as this patient, reinitiating or increasing glucocorticoid therapy can lower ACTH and prevent further growth of the adrenal rest tumors. However, if patients do not respond to glucocorticoids or if pain is severe, surgery may ultimately be necessary. However, surgery (Answer A) is not the first step. Testicular radiation (Answer B) is not indicated. Use of mitotane (Answer E), which has antisecretory and antitumoral activities, has been described in a single case report of testicular adrenal rest tumors. While mitotane could lead to improvements, including shrinkage of tumors and normalization of laboratory values (described in the cited case report), adverse effects include hyponatremia and weight loss, and it is not considered standard of care.

Reference(s): Yu MK, Jung MK, Kim KE, et al. Clinical manifestations of testicular adrenal rest tumor in males with congenital adrenal hyperplasia. Ann Pediatr Endocrinol Metab. 2015;20(3):155-161. PMID: 26512352Bry-Gauillard H, Cartes A, Young J. Mitotane for 21-hydroxylase deficiency in an infertile man. N Engl J Med. 2014;371(21):2042-2044. PMID: 25409392Aycan Z, Bas VN, Cetinkaya S, Yilmaz Agladioglu S, Tiryaki T. Prevalence and long-term follow-up outcomes of testicular adrenal rest tumors in children and adolescent males with congenital adrenal hyperplasia. Clin Endocrinol ( Oxf ). 2013;78(5):667-672. PMID: 23057653

ITE 2017 Question 68 A 33-year-old man has been referred to you for evaluation of Cushing syndrome. He has had a 20-lb (9.1-kg) weight gain in the past 9 months accompanied by the onset of hypertension and edema. He has noted marked weakness, especially when climbing stairs. His family history is negative for any endocrine disorders. He takes no medications. On physical examination, he has facial rounding and supraclavicular and dorsocervical fat accumulation. His blood pressure is 168/110 mm Hg, and pulse rate is 94 beats/min. His height is 75 in (190.5 cm), and weight is 220 lb (100 kg) (BMI = 27.5 kg/m2). He has wide, violaceous striae on his abdomen; proximal muscle weakness; and 2+ pretibial edema.

Laboratory test results: Sodium = 142 mEq /L (136-142 mEq /L) (SI: 142 mmol/L [136-142 mmol/L]) Potassium = 3.0 mEq /L (3.5-5.0 mEq /L) (SI: 3.0 mmol /L [3.5-5.0 mmol /L]) Creatinine = 1.1 mg/ dL (0.7-1.3 mg/ dL ) (SI: 97.2 μmol/L [61.9-114.9 μ mol /L]) Urinary free cortisol = 632 μ g/24 h (4-50 μg/24 h) (SI: 1744 nmol/d [11-138 nmol/d])Late-night salivary cortisol = 2.1 μg/dL (<0.13 μg/dL) (SI: 58 nmol/L [<3.6 nmol/L])DHEA-S = 678 μg/dL (65-334 μg/dL) (SI: 18.4 μmol/L [1.76-9.05 μmol/L])Basal plasma ACTH = <5 pg/mL (10-60 pg/mL) (SI: <1.1 pmol/L [2.2-13.2 pmol/L])

Which of the following imaging studies and accompanying descriptions are most likely in this man? Hounsfield units <10 in each nodule

Increased T2-weighted signal

Unenhanced Hounsfield units = 32 with relative enhancement washout of 58%

Enhanced Hounsfield units = 78 with relative enhancement washout of 12%

Unenhanced Hounsfield units = 52 with absolute enhancement washout of 74%

Correct Answer: D Learning objective : Identify the biochemical and imaging characteristics suggestive of adrenocortical carcinoma.

Rationale: The clinical and biochemical findings in this patient are consistent with the rapid onset of severe endogenous hypercortisolism . The presence of hypokalemic metabolic alkalosis and a relatively sudden onset of clinical Cushing syndrome should always raise the possibility of the ectopic ACTH syndrome, especially in a male patient. However, the plasma ACTH level is undetectable, which confirms the presence of an ACTH-independent process. Adrenal-dependent (ACTH-independent) hypercortisolism may be due to either a benign or malignant adrenocortical neoplasm. In addition, some patients have autonomous cortisol hypersecretion associated with bilateral nodular adrenal hyperplasia. Some of the molecular mechanisms for this group of unique disorders have been identified: mutations in the gene encoding protein kinase A (PRKAR1A; Carney complex) or activating mutations in the gene encoding the receptor GS α subunit (GNAS; McCune-Albright syndrome) are well-recognized causes of bilateral nodular hyperplasia and hypercortisolism . Nodular adrenal-dependent hypercortisolism can also be due to the expression of aberrant hormone receptor signaling with a remarkable group of diverse receptors ( eg , GIP, β-adrenergic, vasopressin, angiotensin II, and gonadotropins).

Rationale Continued: The key diagnostic study in this patient is the marked elevation of adrenal androgen secretion (DHEA-S). With the exception of 1 isolated case report of a patient with bilateral nodular adrenal hyperplasia and adrenal hyperandrogenism , the significant elevation of DHEA-S in a patient with ACTH-independent Cushing syndrome strongly suggests the presence of an adrenocortical carcinoma. Adrenal androgen secretion is ACTH-dependent and patients with either Cushing disease or the ectopic ACTH syndrome usually have normal and sometimes elevated DHEA-S levels. Patients with ACTH-independent hypercortisolism related to a benign adrenocortical neoplasm almost always have low or subnormal DHEA-S. However, patients with adrenocortical carcinoma usually have a normal or elevated DHEA-S level. Consequently, the ominous biochemical findings in this patient suggest adrenocortical carcinoma, which is depicted in Answer D.

Rationale Continued: At the time of discovery, most primary adrenocortical carcinomas are larger than adenomas (usually >4 cm). The margins may be irregular and the contents inhomogeneous with areas of necrosis, hemorrhage, or calcification. Adrenocortical carcinoma has less fat content and will, therefore, have a high attenuation value on unenhanced studies (usually >20 Hounsfield units). Adrenocortical carcinomas usually enhance significantly with contrast because of their vascularity. The relative percentage washout of carcinoma is usually less than 40% in contrast to a much greater washout in benign adenomas. On MRI, adrenal carcinomas are often noted for their heterogeneity and may also show high signal intensity on T2-weighted images with intermediate to high signal intensity on T1-weighted images. Chemical shift signal loss on out-of-phase sequences is usually less than 30% with adrenocortical carcinoma and usually greater than 30% with a benign adrenal adenoma. On PET/CT, adrenocortical carcinomas show a standardized uptake value greater than that of the liver, while the uptake is typically less than that of the liver in benign adenomas. The large left adrenal mass and the imaging findings shown in Answer D are most consistent with adrenocortical carcinoma. Moreover, the small right adrenal gland in this image suggests ACTH suppression and atrophy of the contralateral gland. The differences in imaging characteristics of adrenocortical carcinomas, benign adrenal adenomas, and pheochromocytomas are illustrated (see table).

Table. Imaging Characteristics of Benign Adrenal Adenomas, Pheochromocytomas , and Adrenocortical Carcinoma or Adrenal Metastases a Absolute enhancement washout = 100 X enhanced Hounsfield units – delayed Hounsfield units / enhanced Hounsfield units –unenhanced Hounsfield units. b Relative enhancement washout = 100 X enhanced Hounsfield units – delayed Hounsfield units / enhanced Hounsfield units. Characteristics Benign Adenoma Pheochromocytoma Malignant (Adrenocortical Carcinoma or Metastases) Size <4 cm Any size >4 cm (adrenal cortical carcinoma); variable for metastases Homogeneity Homogenous Variable Heterogeneous Growth Slow Variable Rapid Hounsfield units on CT        Unenhanced<10 HU>18 HU>18 HU EnhancedVariable>30 HU>30 HU % Absolute enhancement washouta>60%<60%<60% % Relative enhancement washoutb>40%<40%<40%MRI Signal    T2-weightedLowHighHigh % Chemical shift loss (out-of- phase sequence)>30%<10%<30%PET/CT Fluorodeoxyglucose uptake<liver>liver>liver

Rationale Continued: The image in Answer A depicts a patient with adrenal-dependent subclinical hypercortisolism due to bilateral nodular adrenal hyperplasia with small, low-attenuation nodules. The imaging characteristics in these patients usually show low signal intensity on T2-weighted images and greater than 30% chemical shift single loss on out-of-phase sequences. Although most benign adrenocortical adenomatous nodules do not show uptake on PET, as many as 5% to 15% of patients with benign adrenal adenomas may have positive images. Of course, benign adrenal adenomas usually have a relatively low attenuation value (Hounsfield units <10 in unenhanced images), as well as a relative washout greater than 40%. The large pituitary neoplasm shown in Answer B is not consistent with ACTH-independent hypercortisolism due to the fact that a pituitary ACTH-secreting neoplasm would be associated with a normal or elevated plasma ACTH.

Rationale Continued: The image in Answer C shows bilateral adrenal hyperplasia, which is due to a small cell carcinoma of the lung. The normal adrenal glands are approximately the same width as the diaphragm—these adrenal glands are symmetrically enlarged. The liver has multiple lesions due to metastatic disease. The washout is greater than 40%, also making it an unlikely source of adrenal carcinoma. The image in Answer E demonstrates a small (1.4-cm) adrenal nodule with an indeterminate attenuation value, as well as a high washout value.

Reference(s): Zeiger MA, Siegelman SS, Hamrahian AH. Medical and surgical evaluation and treatment of adrenal incidentalomas . J Clin Endocrinol Metab. 2011;96(7):2004-2015. PMID: 21632813Flecchia D, Mazza E, Carlini M, et al. Reduced serum levels of dehydroepiandrosterone sulphate in adrenal incidentalomas: a marker of adrenocortical tumour. Clin Endocrinol (Oxf). 1995;42(2):129-134. PMID: 7704956Javorsky BR, Carroll TB, Findling JW. Differential diagnosis of Cushing’s syndrome. In: Swearingen B, Biller BMK, eds. Cushing’s Disease . Springer, New York: 2011: 85-106. Ilias I, Sahdev A, Reznek RH, Grossman AB, Pacak K. The optimal imaging of adrenal tumors: a comparison of different methods. Endocr Relat Cancer . 2007;14(3):587-599. PMID: 17914090

BONE / CALCIUM

ITE 2017 Question 3 A 27-year-old woman with a 3-year history of nephrotic syndrome is found to have the following laboratory values: Serum calcium = 7.8 mg/ dL (8.2-10.2 mg/ dL) (SI: 2.0 mmol/L [2.1-2.6 mmol /L]) Serum creatinine = 1.0 mg/ dL (0.6-1.1 mg/ dL ) (SI: 88.4 µ mol /L [53.0-97.2 µ mol /L]) Phosphorus = 2.6 mg/ dL (2.3-4.7 mg/dL) (SI: 0.8 mmol /L [0.7-1.5 mmol /L]) She reports no paresthesias or muscle cramps.On physical examination, she has 2+ pitting pretibial edema but the Trousseau and Chvostek signs are absent.

Which of the following would be most useful to measure next to determine the etiology of her low serum calcium level? Serum PTH Serum 25-hydroxyvitamin D Serum 1,25-dihydroxyvitamin D Serum magnesium Serum albumin Correct Answer: E Learning objective : Guide the evaluation of hypocalcemia, bearing in mind that calcium correction is required in the setting of low albumin levels.

Rationale: Fifty percent of circulating calcium is bound to serum proteins, primarily to albumin. In this patient with nephrotic syndrome, the serum albumin (Answer E) is almost certainly low. The correction factor is to adjust the serum calcium up or down by 0.8 mg/ dL (0.2 mmol /L) for each 1-mg/dL deviation of serum albumin. If her albumin concentration were 2.6 mg/dL (26 g/L) (lower limit of normal 3.5 mg/ dL [35 g/L]), then her adjusted serum calcium would be at least 8.8 mg/ dL (2.2 mmol /L). Ionized calcium measurement might also be considered, but it is not widely available and it is rarely measured correctly. The patient could also have low magnesium (Answer D), albumin, and vitamin D (Answer B), but the first step is to determine whether her serum unbound calcium level is truly low. 1,25-Dihydroxyvitamin D measurement (Answer C) has limited utility. Its measurement is helpful in renal disease, before treatment with teriparatide , and in investigating non–PTH-mediated hypercalcemia. Measuring PTH (Answer A) is key in determining the cause of hypocalcemia or hypercalcemia, but again it must be confirmed that true hypocalcemia is present before searching for a cause.

Reference(s): Ariyan CE, Sosa JE. Assessment and management of patients with abnormal calcium. Crit Care Med . 2004;32( Suppl 4):S146-S154. PMID: 15064673Schafer AL and Shoback D. Hypocalcemia: definition, etiology, pathogenesis, diagnosis, and management. In: Rosen CJ, Bouillon R, Compston JE, Rosen V, eds. Primer on the Metabolic Bone Diseases and Disorders of Mineral Metabolism. 8th ed. Washington, DC: American Society for Bone and Mineral Research; 2013:572-578.

ITE 2017 Question 11 A 35-year-old woman is referred to you for evaluation of hyperphosphatemia. As you elicit a medical history, the patient reports that she began to develop calcium deposits in different body sites at age 5 years. Initially, the affected areas were her right elbow and left calf, and the deposits were surgically removed. Since then, she has had nearly a dozen of these nodules on her hands, feet, chest wall, and breasts. They developed less frequently after puberty. When she was a child, blood work revealed an elevated phosphorous level. She was treated with dietary modification and, for a few years, with aluminum-containing antacids. For the past 6 years, she has followed a diet with limited phosphorous intake. She has 1 child, age 10 years, who has normal phosphorous levels and no calcium deposits. Because the placenta was highly calcified at birth, the patient elected to have no other children. She uses oral contraceptives for birth control. On physical examination, her blood pressure is 102/62 mm Hg and pulse rate is 62 beats/min. Her height is 63 in (160 cm), and weight is 110 lb (50 kg) (BMI = 19.5 kg/m2). Physical examination findings are normal except for small soft-tissue calcifications in her right hand (see image), right arm near the elbow, left calf, and both breasts.

Results from a chemistry panel: Creatinine = 1.0 mg/ dL (0.6-1.1 mg/ dL ) (SI: 88.4 µmol/L [53.0-97.2 µmol/L])Calcium = 9.5 mg/ dL (8.2-10.2 mg/ dL ) (SI: 2.4 mmol /L [2.1-2.6 mmol /L]) Phosphorus = 6.2 mg/ dL (2.3-4.7 mg/ dL ) (SI: 2.0 mmol/L [0.7-1.5 mmol/L]) Albumin = 4.4 g/ dL (3.5-5.0 g/ dL ) (SI: 44 g/L [35-50 g/L]) 25-Hydroxyvitamin D = 44 ng/mL (25-80 ng/mL [optimal]) (SI: 109.8 nmol/L [62.4-199.7 nmol/L])1,25-Dihydroxyvitamin D = 38 pg/mL (16-65 pg/mL) (SI: 98.8 pmol/L [41.6-169.0 pmol/L])PTH = 17.3 pg/mL (10-65 pg/mL) (SI: 17.3 ng/L [10-65 ng/L])Urinary calcium = 70 mg/24 h (0.8 g creatinine in sample) (100-300 mg/24 h) (SI: 1.8 mmol/d [2.5-7.5 mmol/d])

An inactivating mutation in the gene encoding which of the following proteins is the most likely cause of hyperphosphatemia in this patient? Calcium-sensing receptor or cyclin D1 Low-density lipoprotein receptor–related protein 5 or sclerostin PTH receptor or PTHrP receptor Vitamin D receptor or 1 α- hydroxylase UDP- GalNAc transferase 3 or fibroblast growth factor 23 Correct Answer: E Learning objective : Identify the genetic causes of hyperphosphatemic tumoral calcinosis.

Rationale: Uncovering the etiology of isolated hyperphosphatemia is the central focus for this patient. Hyperphosphatemia due to renal insufficiency is the most common diagnosis overall, but that would not be the underlying cause in this woman because her creatinine concentration is within the reference range and her PTH concentration is not elevated (secondary hyperparathyroidism). On the basis of her symptoms and the age of onset (childhood), this appears to be an inherited syndrome. Specifically, her clinical picture fits with the diagnosis of hyperphosphatemic familial tumoral calcinosis (HFTC). This is a rare syndrome characterized by ectopic subcutaneous and vascular calcified masses in the presence of hyperphosphatemia. HFTC exemplifies disturbances in the physiologic factors regulating phosphate balance.

Rationale Continued: The phosphaturic hormone fibroblast growth factor 23 (FGF23) appears to have a central role in HFTC. Over-expression of FGF23 due to gain-of-function mutations results in autosomal dominant hypophosphatemic rickets. Loss of function of the pathways responsible for FGF23 degradation, such as that caused by mutations in PHEX, results in X-linked hypophosphatemic rickets. Similarly, tumor-induced hypophosphatemic osteomalacia represents humoral factors that inhibit the PHEX protein and increase phosphaturic compounds such as FGF23. Mutations resulting in a loss of FGF23 function cause HFTC. To date, at least 7 separate mutations have been described. Additionally, inactivating mutations in the GALNT3 gene, which encodes UDP- GalNAc transferase 3, reduce biologically active FGF23 by decreasing O-glycosylation. Finally, mutations in an FGF23 coreceptor , the KL gene (klotho), prevent FGF23 signaling. Disease-causing mutations in any of these 3 genes lead to increased serum phosphorus that complexes with calcium to cause ectopic deposits and the clinical syndrome. Thus, inactivating mutations in the genes encoding UDP- GalNAc transferase 3 or fibroblast growth factor 23 (Answer E) are correct.

Rationale Continued: The calcium-sensing receptor, encoded by the CASR gene, is important for parathyroid cell recognition of calcium concentration and subsequent PTH secretion. Inactivating CASR mutations result in hypercalcemia (familial hypocalciuric hypercalcemia) with phosphate levels in the normal or low-normal range. Activating mutations cause hypoparathyroidism and hypocalcemia with hyperphosphatemia, which is not the pattern seen in this patient. The gene encoding cyclin D1 (CCND1, the parathyroid adenomatosis 1 gene, previously PRAD1) is involved in the development of parathyroid adenomas via a centromere inversion (thus, Answer A is incorrect).The Wnt /β-catenin signaling system is a ubiquitous regulatory cascade that is important in many physiologic functions, including osteoblast activity. Wnt binding to the low-density lipoprotein receptor–related protein 5 (encoded by the LRP5 gene) inhibits intracellular GSK3 activity, resulting in an increase in β-catenin that stimulates osteoblast differentiation and bone formation via actions on nuclear transcription factors. Inhibitors of the Wnt pathway, such as sclerostin , the product of the SOST gene, prevent intracellular accumulation of β-catenin and reduce osteoblast activity. Indeed, sclerostin has been the focus of pharmacologic investigation using neutralizing antibodies as a means to increase bone density. While their actions on bone metabolism are important, changes in Wnt / sclerostin do not alter phosphate levels and would not cause the biochemical picture in this patient (thus, Answer B is incorrect).

Rationale Continued: PTH and PTHrP share a common receptor. Changes in receptor sensitivity, via inactivating mutations would have profound effects on both calcium and phosphate levels. Inactivating mutations cause pseudohypoparathyroidism with hypocalcemia and hyperphosphatemia. In this patient, there is no evidence of an altered calcium level (thus, Answer C is incorrect).The actions of vitamin D on calcium homeostasis are complex and multifaceted. Loss of function of the vitamin D receptor, encoded by the VDR gene, or loss of function of the 1α-hydroxylase enzyme, encoded by the CYP27B1 gene, which is needed to activate 25-hydroxyvitamin D, results in reduced intestinal absorption of both calcium and phosphorus and thus in hyperparathyroidism. Further decreases in the phosphate concentration occur while trying to restore normal calcium levels (thus, Answer D is incorrect).

Reference(s): Farrow EG, Imel EA, White KE. Miscellaneous non-inflammatory musculoskeletal conditions. Hyperphosphatemic familial tumoral calcinosis (FGF23, GALNT3 and Klotho). Best Pract Res Clin Rheumatol. 2011;25(5):735-747. PMID: 22142751Ichikawa S, Baujat G, Seyahi A, et al. Clinical variability of familial tumoral calcinosis caused by novel GALNT3 mutations. Am J Med Genet A. 2010;152A(4):896-903. PMID: 20358599Chefetz I, Sprecher E. Familial tumoral calcinosis and the role of O-glycosylation in the maintenance of phosphate homeostasis. Biochim Biophys Acta . 2009;1792(9):847-852. PMID: 19013236 Sprecher E. Familial tumoral calcinosis: from characterization of a rare phenotype to the pathogenesis of ectopic calcification. J Invest Dermatol. 2010;130(3):652-660. PMID: 19865099

ITE 2017 Question 21 You are evaluating a 74-year-old woman in whom osteoporosis was diagnosed at age 67 years. She has since been treated with an oral bisphosphonate. She underwent menopause at age 53 years, and she has not taken estrogen. Results from a laboratory evaluation (including a complete blood cell count, chemistry panel, PTH level, 25-hydroxyvitamin D level, and 24-hour urinary calcium excretion) were normal 2 years ago and again last month. Ten years ago, breast cancer was diagnosed, and she was treated with surgery, chemotherapy, and radiation. Over the past 2 years, her bone mineral density has decreased. She has lost 2 in (5.1 cm) in height. Vertebral fracture assessment shows 2 thoracic compression fractures of uncertain age. The patient asks about treatment with teriparatide .

Which of the following is a contraindication to the use of teriparatide in this patient? Her age History of breast cancer Previous bisphosphonate therapyHistory of radiation treatment Concurrent treatment with an aromatase inhibitor Correct Answer: D Learning objective : Recognize radiation as a contraindication to the use of teriparatide .

Rationale: Teriparatide increases the risk of osteosarcoma in rats. Although there is no evidence of an increased risk in humans, use of teriparatide in patients whose risk of osteosarcoma is higher than baseline is not recommended. This includes children and adolescents with unfused epiphyses, patients with Paget disease, and those with a history of skeletal irradiation (thus, Answer D is correct and Answer E is incorrect). Age (Answer A) is not a contraindication to the use of teriparatide . Pharmacovigilance surveys have not confirmed any cases of osteosarcoma in users of teriparatide. Teriparatide should not be used by any patient with a malignant bone tumor, skeletal metastasis, or high risk for skeletal metastasis. However, breast cancer (Answer B) and other malignancies, in the absence of high risk of skeletal metastases, are not a contraindication. Teriparatide probably should not be used in patients with elevated PTH levels, although the evidence for this is not conclusive. Previous bisphosphonate therapy (Answer C) may blunt the anabolic effect of teriparatide , but it is not a contraindication.

Reference(s): Silverman SL, Nasser K. Teriparitide update. Rheum Dis Clin North Am. 2011;37(3):471-477. PMID: 22023903Cosman F. Parathyroid hormone treatment for osteoporosis. Curr Opin Endocrinol Diabetes Obes. 2008;15(6):495-501. PMID: 18971677Andrews EB, Gilsenan AW, Midkiff K, et al. The US postmarketing surveillance study of adult osteosarcoma and teriparatide: study design and findings from the first 7 years. J Bone Miner Res . 2012;27(12):2429-2437. PMID: 22991313

ITE 2017 Question 23 A 65-year-old man is referred to you for evaluation of possible Paget disease. He was found to have an elevated alkaline phosphatase level on recent laboratory studies done before cataract surgery. He had bariatric surgery 15 years ago and takes cholecalciferol, 2000 IU daily. He has no chronic medical problems and has not been to a physician in the past 5 years. Generally, he feels well. Laboratory test results: Alkaline phosphatase = 220 U/L (50-120 U/L) (SI: 3.7 µ kat /L [0.8-2.0 µkat /L]) Serum calcium = 8.6 mg/ dL (8.2-10.2 mg/ dL ) (SI: 2.2 mmol /L [2.1-2.6 mmol /L]) Serum creatinine = 1.3 mg/ dL (0.7-1.3 mg/ dL) (SI: 114.9 µmol/L [61.9-114.9 µ mol /L]) Gamma- glutamyltranspeptidase , normal

Which of the following should be the next step in this patient’s care? Whole-body bone scan Skeletal survey Intravenous zoledronic acid infusion Serum C-telopeptide measurement 25-Hydroxyvitamin D and PTH measurement Correct Answer: E Learning objective : Rule out vitamin D deficiency and secondary hyperparathyroidism before evaluating for Paget disease.

Rationale: Although this patient may have Paget disease, other causes of elevated alkaline phosphatase must be considered before proceeding to bone scan, including vitamin D deficiency and/or secondary hyperparathyroidism, particularly given his age and chronic kidney disease. This patient was indeed found to have vitamin D deficiency with secondary hyperparathyroidism. Thus, measurement of 25-hydroxyvitamin D and PTH (Answer C) is the correct next step. Although he has been taking 2000 IU of vitamin D daily, this is not enough to maintain a normal 25-hydroxyitamin D level in some patients after bariatric surgery. In this situation, a whole-body bone scan (Answer A) may be spuriously abnormal, showing multiple areas of uptake due to increased bone turnover. Serum C- telopeptide (Answer D) may be elevated, but its measurement will not help to determine the cause of his elevated alkaline phosphatase. A skeletal survey (Answer B) would be helpful if multiple myeloma were the suspected diagnosis, which is not the case here. While intravenous zoledronic acid is very effective treatment for Paget disease, it should not be given until the diagnosis is confirmed. Also, if this were to be given in the setting of severe vitamin D deficiency, it could cause severe hypocalcemia.

Reference(s): Karefylakis C, Näslund I, Edholm D, Sundbom M, Karlsson FA, Rask E. Vitamin D status 10 years after primary gastric bypass: gravely high prevalence of hypovitaminosis D and raised PTH levels. Obes Surg. 2014;24(3):343-348. PMID: 24163201

ITE 2017 Question 30 A 55-year-old white woman underwent menopause 3 years ago and she has no menopausal symptoms. Her BMI is 20 kg/m2. She is generally healthy, but she fractured 2 fingers after a minor fall 2 years ago. She does not smoke cigarettes, she drinks 1 glass of wine nightly, and she takes no medications. Her diet provides 1200 mg of calcium daily and 1000 IU of vitamin D daily. Her mother had a hip fracture. The patient’s DXA documents the following: Site BMD, g/cm 2 T Score Total hip 0.698 –1.5 Femoral neck 0.658 –1.7 L1-L4 0.983 –0.3 Vertebral fracture assessment does not identify any compression fractures. Her FRAX 10-year risk is 12% for any major osteoporotic fracture and 0.7% for hip fracture.

Which of the following is the correct management plan? Start a bisphosphonate and perform DXA in 1 year Continue dietary calcium and vitamin D and perform DXA in 2 years Start raloxifene and perform DXA in 1 yearStart estrogen replacement therapy Start teriparatide and perform DXA in 1 year Correct Answer: B Learning objective : Use the FRAX tool to guide treatment of patients with low bone mass.

Rationale: According to the National Osteoporosis Foundation, osteoporosis can be diagnosed by a T score in the hip or spine of less than –2.5 or by a fracture of the hip or spine. Fractures of fingers do not meet the diagnostic criteria for osteoporosis. Therefore, this patient has low bone mass (osteopenia), and the best tool to determine whether she should be treated pharmacologically is FRAX. Current National Osteoporosis Foundation guidelines recommend that patients who have a 10-year fracture risk of 20% or higher for any major fracture or 3% or higher for hip fracture should be treated pharmacologically. This patient’s fracture risk is well below the treatment threshold, and she should therefore continue with her dietary calcium and vitamin D for now and undergo DXA again in 2 years (Answer B). Treating her with a bisphosphonate (Answer A), teriparatide (Answer E), or the selective estrogen receptor modulator raloxifene (Answer C) is incorrect because these therapies are indicated for those at increased risk of fracture. Estrogen replacement therapy (Answer D) is no longer approved by the US FDA for treating osteoporosis (only for prevention), so even if she required treatment now, this would be an incorrect choice.

Reference(s): Camacho PM, Petak SM, Binkley N, et al. American Association of Clinical Endocrinologists and American College of Endocrinology clinical practice guidelines for the diagnosis and treatment of postmenopausal osteoporosis - 2016. Endocr Pract. 2016;22(Suppl 4):1-42. PMID: 27662240National Osteoporosis Foundation. Clinician’s Guide to Prevention and Treatment of Osteoporosis. Washington, DC: National Osteoporosis Foundation; 2014.

ITE 2017 Question 35 A 38-year-old man presents with asymptomatic hypercalcemia. He has normal findings on physical examination and the following laboratory workup: Total calcium = 11.5 mg/ dL (8.2-10.2 mg/ dL) (SI: 2.9 mmol/L [2.1-2.6 mmol /L]) PTH = 50 pg /mL (10-65 pg /mL) (SI: 50 ng/L [10-65 ng/L]) Creatinine = 1.1 mg/ dL (0.7-1.3 mg/ dL ) (SI: 97.2 µ mol /L [61.9-114.9 µmol/L]) 25-Hydroxyvitamin D = 22 ng/mL (25-80 ng/mL [optimal]) (SI: 54.9 nmol /L [62.4-199.7 nmol /L]) Magnesium = 3.0 mg/dL (1.5-2.3 mg/dL) (SI: 1.2 mmol/L [0.6-0.9 mmol/L])Urinary calcium excretion is 36 mg/24 h with a calcium-to-creatinine clearance ratio of 0.01. Neck ultrasonography does not identify an enlarged parathyroid gland. There are no previous calcium values for comparison. The patient is adopted and has no knowledge of his family’s medical history.He undergoes parathyroidectomy, and 2 enlarged glands are resected. However, intraoperative PTH levels remain elevated. Pathologic examination reveals hyperplasia in both glands. Postoperatively, his calcium concentration is 11.6 mg/dL (2.9 mmol/L). On repeated testing 8 months postoperatively, he continues to have hypercalcemia and a PTH concentration in the upper-normal range.

Which of the following is the best next step in this patient’s care? Start cinacalcet , 30 mg daily Perform a second surgery with biopsy and resection of both remaining parathyroid glands Perform genetic testingPerform a 4D CT of the neck Perform neck ultrasonography again Correct Answer: C Learning objective : Suspect familial hypocalciuric hypercalcemia in patients with an indeterminate calcium-to-creatinine clearance ratio.

Rationale: PTH-dependent hypercalcemia was correctly diagnosed in this patient. The consideration of familial hypocalciuric hyperparathyroidism (FHH) was also appropriately considered. Most patients with this pattern of laboratory results have primary hyperparathyroidism; other diagnoses can present with similar biochemical findings. In addition, this patient had no previously documented calcium values for comparison, so it was not possible to distinguish an acquired problem from a congenital one. His evaluation also appropriately included measurement of 24-hour urinary calcium excretion, and the result was suggestive of FHH given that the urinary calcium excretion was low and the fractional excretion of calcium (calcium-to-creatinine clearance ratio) was indeterminate at 0.01. Calculating the calcium-to-creatinine clearance ratio is helpful to distinguish FHH from primary hyperparathyroidism, as 80% of patients with FHH have a ratio less than 0.01 and most patients with primary hyperparathyroidism have a ratio greater than 0.02. The calcium-to-creatinine clearance ratio is calculated with the following equation: Ca/Cr clearance ratio = [24-hour urine Ca x serum Cr] ÷ [serum Ca x 24-hour urine Cr]

Rationale Continued: FHH is an autosomal dominant disorder with high penetrance and is caused by inactivating mutations in the calcium-sensing receptor gene (CASR). More than 200 CASR mutations have been described. In this disorder, the parathyroid glands have reduced sensitivity to calcium and higher serum calcium levels are needed to reduce parathyroid release. The calcium-PTH curve is shifted to the right, resetting the serum calcium to a higher level. In the kidney, this produces an increase in tubular calcium and magnesium reabsorption. Typical biochemical abnormalities include hypercalcemia, hypocalciuria , normal or slightly elevated PTH, and elevated magnesium. There are 3 known subtypes of FHH. The most common subtype, which accounts for more than two-thirds of cases, is caused by inactivating CASR mutations. Persons who are heterozygous for an inactivating CASR mutation typically have mild hypercalcemia due to only partial loss of calcium-sensing receptor function. Homozygous mutations cause neonatal severe hyperparathyroidism and represent the most severe expression of FHH, as neonates present with life-threatening hypercalcemia. FHH type 2 is rarer and is associated with mutations in the GNA11 gene. FHH type 3 has been recently described in more than 50 patients and is caused by heterozygous missense germline mutations in the AP2S1 gene. The AP2S1 gene encodes a protein complex that participates in recycling of cell-surface proteins.

Rationale Continued: Genetic screening for FHH may be indicated in patients such as the one in this vignette who have an indeterminate calcium-to-creatinine ratio (genetic screening is rarely needed if the ratio is <0.01 or >0.02). If a patient with suspected FHH tests negative for CASR mutations (as indeed the patient in this vignette did), then consideration of additional genetic screening for AP2S1 mutations may avoid unnecessary surgery. If genetic testing is not available or the cost is prohibitive, then a more thorough review of family history is indicated to screen for FHH (this was not possible in this adopted patient). Although further imaging of the neck with ultrasonography (Answer E) or 4D CT (Answer D) would be indicated if the patient had persistent primary hyperparathyroidism despite attempted surgical resection, visualization of the parathyroid glands is not indicated in the setting of FHH. Indeed, resection of all 4 glands would result in severe hypocalcemia and thereby cause harm. Most patients with FHH are asymptomatic. They do not develop nephrolithiasis due to the low urinary calcium excretion, and they do not experience complications such as bone loss, hypertension, or peptic ulcer disease. Neither a second surgery (Answer B) nor the addition of a calcimimetic (Answer A) is indicated.

ITE 2017 Question 38 A 30-year-old woman presents for evaluation of hypercalcemia after workup for a 6-month history of worsening fatigue and anorexia documented a serum calcium concentration of 12.9 mg/ dL (3.2 mmol /L). During her summer vacation at the beach, she became so dehydrated that she required intravenous hydration at a local emergency department. She reports poor sleep because she gets up multiple times to urinate. Menses have become irregular and she has not had a period for 2 months. She notes constipation, dry skin, hair loss, and a 10-lb (4.5-kg) weight loss. She has no night sweats. She has had no fractures. She has no history of kidney stones, but she recalls an episode of hematuria. She has no family history of hypercalcemia or other endocrine disorders. Other than a daily multivitamin, she takes no medication. On physical examination, she appears fatigued. While sitting, her pulse rate is 80 beats/min and blood pressure is 105/70 mm Hg. While standing, her pulse rate is 100 beats/min and blood pressure is 90/65 mm Hg. Her temperature is 97.7ºF (36.5ºC). Her height is 64 in (162.6 cm), and weight is 137 lb (62.3 kg) (BMI = 23.5 kg/m2). Mucous membranes are dry, periorbital edema is present, and visual fields are intact. Findings on thyroid examination are normal. She has expressible galactorrhea. She has no organomegaly . You observe no rashes or hyperpigmentation, but her skin is dry and there is some tenting of the skin. Muscle bulk, tone, and strength are normal, but her deep tendon reflexes are delayed.

Laboratory test results: Sodium = 129 mEq /L (136-142 mEq /L) (SI: 129 mmol/L [136-142 mmol/L])Potassium = 4.0 mEq /L (3.5-5.0 mEq /L) (SI: 4.0 mmol /L [3.5-5.0 mmol /L]) Creatinine = 1.3 mg/ dL (0.6-1.1 mg/ dL ) (SI: 114.9 µ mol/L [53.0-97.2 µmol/L]) Serum urea nitrogen = 20 mg/ dL (8-23 mg/ dL ) (SI: 7.1 mmol/L [2.9-8.2 mmol/L])Hematocrit = 36% (41%-50%) (SI: 0.36 [0.41-0.51])Calcium = 12.9 mg/dL (8.2-10.2 mg/dL) (SI: 3.2 mmol/L [2.1-2.6 mmol/L])Phosphorus = 4.9 mg/dL (2.3-4.7 mg/dL) (SI: 1.6 mmol/L [0.7-1.5 mmol/L])Intact PTH = 5 pg/mL (10-65 pg/mL) (SI: 5 ng/L [10-65 ng/L])Prolactin = 42 ng/mL (4-30 ng/mL) (SI: 1.8 nmol/L [0.17-1.30 nmol/L])TSH = 0.5 mIU/L (0.5-5.0 mIU/L)Free T4 = 0.5 ng/dL (0.8-1.8 ng/dL) (SI: 6.4 pmol/L [10.30-23.17 pmol/L])25-Hydroxyvitamin D = 25 ng/mL (25-80 ng/mL [optimal]) (SI: 62.4 nmol/L [62.4-199.7 nmol/L])1,25-Hydroxyvitamin D = 85 pg/mL (16-65 pg/mL) (SI: 221 pmol/L [41.6-169.0 pmol/L])Alkaline phosphatase = 98 U/L (50-120 U/L) (SI: 1.6 µkat/L [0.84-2.00 µkat/L])FSH = 0.1 mIU/mL (4.0-36.0 mIU/mL) (SI: 0.1 IU/L [4.0-36.0 IU/L])

Which of the following medications should be prescribed initially? Zoledronic acid Cinacalcet Calcitonin Prednisone Levothyroxine Correct Answer: D Learning objective : Diagnose sarcoidosis after performing the differential diagnosis of non–PTH-mediated hypercalcemia and review other endocrine complications of sarcoidosis.

Rationale: Both granulomatous and lymphoproliferative disorders are associated with increased production of 1,25-dihydroxyvitamin D because of increased ectopic 1α-hydroxylase activity. Although the renal formation of 1,25-dihydroxyvitamin D is tightly regulated, the extrarenal formation is not regulated by normal feedback mechanisms, allowing inappropriate elevation of 1,25-dihydroxyvitamin D, which in turn leads to hypercalcemia. Granulomatous disorders, such as sarcoidosis, have been associated with hypercalcemia due to elevated 1,25-dihydroxyvitamin D levels. Patients with sarcoidosis or other granulomatous disorders that drive unregulated 1,25-dihydroxyvitamin D production often have worsening hypercalcemia with sun exposure and are prone to dehydration. This patient, however, has more manifestations of sarcoidosis than just hypercalcemia. Sarcoid can infiltrate a number of organs including the pituitary and the hypothalamus. This patient has disturbed menses, an elevated prolactin concentration, signs of adrenal insufficiency, and evidence of central hypothyroidism. In addition, she has symptomatic hypercalcemia (symptoms include fatigue, anorexia, and increased urinary calcium excretion with nephrolithiasis) and nocturia due to nephrogenic diabetes insipidus. The best initial treatment for sarcoidosis is administration of glucocorticoids, which decrease 1α-hydroxylation and conversion to 1,25-dihydroxyvitamin D by macrophages. In this case, prednisone (Answer D) would also treat adrenal insufficiency. Although the patient has central hypothyroidism and needs treatment, levothyroxine (Answer E) should be delayed until steroids have been administered to avoid adrenal crisis.

Rationale Continued: This patient has non–PTH-mediated hypercalcemia, as the PTH level is appropriately suppressed. For this reason, use of a calcimimetic such as cinacalcet (Answer B) to further suppress PTH is not appropriate.Calcitonin (Answer C) lowers serum calcium by reducing bone resorption and increasing renal calcium excretion. Tachyphylaxis occurs after 48 hours of treatment, most likely due to receptor down-regulation. Therefore, use of calcitonin should be limited to symptomatic patients when combined with other therapies. In addition, calcitonin does not reduce the conversion of 25-hydroxyvitamin D to 1,25-dihydroxyvitamin D and would not address the underlying pathology. Zoledronic acid (Answer A) effectively lowers serum calcium when excessive bone resorption is the cause of hypercalcemia such as in cancer. However, it would be only modestly effective in this case because it would not reduce the conversion of 25-hydroxyvitamin D to 1,25-dihydroxyvitamin D.

Reference(s): Sandler LM, Winearls CG, Fraher LJ, Clemens TL, Smith R, O’Riordan JL. Studies of the hypercalcaemia of sarcoidosis: effect of steroids and exogenous vitamin D3 on the circulating concentrations of 1,25- dihydroxyvitamin D3. Q J Med. 1984;53(210):165-180. PMID: 6087399Insogna KL, Dreyer BE, Mitnick M, Ellison AF, Broadus AE. Enhanced production rate of 1,25- dihydroxyvitamin D in sarcoidosis. J Clin Endocrinol Metab. 1988;66(1):72-75. PMID: 3335611

Reference(s): Nesbit MA, Hanna FM, Howles SA, et al. Mutations in AP2S1 cause familial hypocalciuric hypercalcemia type 3. Nat Genet. 2013;45(1):93-97. PMID: 23222959Hendy GN, D’Souza-Li L, Yang B, Canaff L, Cole DE. Mutations of the calcium-sensing receptor (CASR) in familial hypocalciuric hypercalcemia, neonatal severe hyperparathyroidism, and autosomal dominant hypocalcemia. Hum Mutat. 2000;16(4):281-296. PMID: 11013439Brown EM. The calcium-sensing receptor: physiology, pathophysiology and CaR-based therapeutics. Subcell Biochem. 2007;45:139-167. PMID: 18193637

ITE 2017 Question 41 A 56-year-old woman presents with increased pain in her right hip, right groin, and left tibia, which has been limiting her mobility. Paget disease of bone was diagnosed at age 34 years when an elevated alkaline phosphatase level was documented on routine blood testing performed because of a family history of Paget disease in her mother and multiple maternal relatives. Since diagnosis, she has received a 7-year course of intermittent treatment with bisphosphonate therapy. She is an 80 pack-year active cigarette smoker with chronic obstructive lung disease, and she also has a history of severe gastroesophageal reflux disease. She has had no fractures, loss of height, diminished hearing, or neuropathy. On physical examination, she appears older than her stated age and has a hoarse voice. She has some discoloration of her teeth with no gingivitis or exposed bone. There is no frontal bossing. She has asymmetry of her left tibia compared with the right tibia with some irregular protrusion and increased warmth overlying the skin of the left tibia. The right pelvic brim has no palpable abnormalities.

Laboratory test results: Alkaline phosphatase = 200 U/L (50-120 U/L) (SI: 3.3 µ kat /L [0.84-2.00 µ kat /L])Bone-specific alkaline phosphatase = 114 µg/L (≤22 µg/L)Calcium = 9.4 mg/dL (8.2-10.2 mg/dL ) (SI: 2.4 mmol /L [2.1-2.6 mmol /L]) Phosphorus = 3.3 mg/ dL (2.3-4.7 mg/ dL ) (SI: 1.1 mmol /L [0.7-1.5 mmol /L])Albumin = 4.3 g/dL (3.5-5.0 g/ dL ) (SI: 43 g/L [35-50 g/L]) Creatinine = 0.8 mg/ dL (0.6-1.1 mg/ dL) (SI: 70.7 µmol/L [53.0-97.2 µmol/L])25-Hydroxyvitamin D = 18 ng/mL (25-80 ng/mL [optimal]) (SI: 44.9 nmol/L [62.4-199.7 nmol/L])Imaging results are shown (see images).

Bone Scan

Plain x-rays of the left tibia (left panel, lateral view; right panel, anteroposterior view).

Plain x-ray of the pelvis (anteroposterior view).

Which of the following is the most appropriate next step in this patient’s management? Administer denosumab , 60 mg subcutaneously (1 dose) Administer zoledronic acid, 5 mg intravenously (1 dose)Prescribe ergocalciferol , 50,000 IU orally weekly for 6 to 8 weeks Prescribe risedronate , 30 mg daily for 2 months Nothing now; she has been exposed to long-term bisphosphonate therapy and is at risk for osteonecrosis of the jaw Correct Answer: C Learning objective : Diagnose familial Paget disease of bone and recommend appropriate management strategies.

Rationale: Paget disease of bone is characterized by focal areas of accelerated bone remodeling leading to overgrowth of bone in one or more areas of the skeleton. Pathogenically, it is a disorder of the osteoclasts. There are increased osteoclast cell numbers, with abnormal size and appearance, and stimulated osteoblastic activity leads to abnormal and disorganized bone formation. Affected patients typically present with focal areas of pain, osteoarthritis, deformity, fracture, neurologic disease from nerve impingement, or less commonly high-output heart failure in the setting of extensive polyostotic disease. There is an increased risk of osteosarcoma and it is more likely to be seen in those with extensive and/or longstanding disease. Paget disease has a predilection for certain regions. In all patients with Paget disease, the following regions commonly have involvement: pelvis (70%), femur (55%), lumbar spine (53%), skull (42%), and long bones of the lower extremities (especially the tibia, 32%). Notably, the sites involved rarely change over time in a patient.

Rationale Continued: Her family history is notable for other relatives with Paget disease. Linkage studies and genome-wide association studies have identified several genes and loci that predispose to Paget disease. The involved genes are important for osteoclast differentiation and function. The most common susceptibility gene is SQSTM1 (which encodes sequestosome-1), and it accounts for approximately 40% of patients with familial Paget disease. Familial Paget disease is inherited in an autosomal dominant pattern with variable penetrance. Environmental factors ( eg , arsenic, lead, viruses) may also have a role in Paget disease. Management of Paget disease of bone is directed towards the symptoms ( ie, pain control with analgesics) and decreasing bone turnover with either intravenous or oral bisphosphonates. In this patient who has a low vitamin D level (18 ng/mL [44.9 nmol /L]), vitamin D replacement should be implemented first before any other therapy to prevent bisphosphonate-related hypocalcemia (thus, Answer C is correct and Answers B and D are incorrect). Additionally, she should be counseled on maintaining good oral hygiene, the importance of tobacco cessation, the rare risk of medication-related osteonecrosis of the jaw, and possible referral to a dental specialist before re-exposure to a bisphosphonate. Denosumab (Answer A) is currently not approved by the US FDA for the treatment of Paget disease, but it may be considered at the dosage of 120 mg monthly if there is a giant-cell tumor related to Paget disease.

Rationale Continued: Doing nothing (Answer E) would be inappropriate, as the patient needs vitamin D replacement followed by treatment with a bisphosphonate because she is symptomatic (after appropriate counseling is provided regarding the risk of medication-related osteonecrosis of the jaw). As she has had numerous exposures to oral bisphosphonate and because she has severe GERD, the best next treatment would be intravenous bisphosphonate such as zoledronic acid given its greater potency.

Reference(s): Reid IR, Sharma S, Kalluru R, Eagleton C. Treatment of Paget's disease of bone with denosumab : case report and literature review. Calcif Tissue Int. 2016;99(3):322-325. PMID: 27193832Singer FR. Paget’s disease of bone-genetic and environmental factors. Nat Rev Endocrinol. 2015;11(11):662- 671. PMID: 26284446Singer FR, Bone HG 3rd, Hosking DJ, et al; Endocrine Society. Paget’s disease of bone: an endocrine society clinical practice guideline. J Clin Endocrinol Metab. 2014;99(12):4408-4422. PMID: 25406796Tan A, Ralston SH. Paget’s disease of bone. QJM . 2014;107(11):865-869. PMID: 24757095

ITE 2017 Question 53 A 21-year-old man with osteogenesis imperfecta is transferring his care from pediatric endocrinology to your adult endocrinology practice.. He is wheelchair bound. Osteogenesis imperfecta was diagnosed at birth when a femur fracture was noted shortly after delivery. He has had multiple fractures of all extremities including bilateral femur fractures and bilateral tibial fractures. He recently developed pseudoarthrosis of his bilateral humeri. He has been treated with weekly alendronate. Because of worsening dysphagia and abdominal pain, which limits his adherence to this medication, he would like his regimen to be transitioned to an intravenous bisphosphonate. There is no family history of osteogenesis imperfecta or other metabolic bone disorders. On physical examination, he is markedly short, has severe scoliosis, and has shortened and bowed deformities of the upper and lower extremities. Sclerae are slightly tinted blue. His blood pressure is 128/76 mm Hg, and pulse rate is 94 beats/min. His height is 39 in (99.1 cm), and weight is 50 lb (22.7 kg) (BMI = 23.1 kg/m2).

In addition to musculoskeletal complications, this patient is at highest risk for which of the following? Aortic root dilation Premature atherosclerotic cardiovascular disease Bone cancers Retinal detachment Premature cataracts Correct Answer: A Learning objective : Explain the clinical presentation and manifestations of osteogenesis imperfecta.

Rationale: Osteogenesis imperfecta (OI) is an inherited connective tissue disorder of type 1 collagen. The estimated incidence is 1 in 20,000. It is associated with mutations in the COL1A1 and COL1A2 genes. Most forms of OI are inherited in an autosomal dominant manner, although autosomal recessive forms have been described. Type 1 collagen is a constituent of bone, ligaments, skin, and sclerae . OI is classified into multiple subtypes on the basis of clinical characteristics. The severity of the clinical presentation depends not only on the gene mutation, but also on other underlying defects in collagen. The phenotypic presentation is variable within the same family, and the clinical picture can range from mild, premature osteoporosis to multiple fractures precipitated by little or no trauma. Cardiovascular abnormalities are relatively common in patients with OI because of the underlying defect in connective tissue. Multiple small studies have documented the prevalence of valvular heart diseases and aortic root dilation to be higher in children with osteogenesis imperfecta than in control patients. Aortic and mitral valvular insufficiency occurs, as does aortic root dilation (thus, Answer A is correct). However, premature atherosclerotic disease (Answer B) has not been reported in OI.

Rationale Continued: Of all the forms of OI, type 1 has the least severe presentation with minimal deformity and variable fracture rate. Patients with type 1 OI may escape diagnosis until a pattern of premature osteoporosis and hearing loss presents in adulthood. The patient in this vignette, however, has type 3 OI as evidenced by his more severe presentation and bone fragility leading to fracture and deformity. Persons with type 3 OI have marked short stature. Discoloration of the sclerae (blue-gray color) is caused by defective type 1 collagen leading to transparency of the underlying choroidal veins. Blue sclerae at birth may lessen over time. There have been rare reports of cataracts associated with the perinatal lethal form of OI, but no reports of premature cataracts (Answer E). There are very rare reports of blindness due to corneal opacity, hyperplasia of the vitreous, and secondary glaucoma. Retinal detachment (Answer D) is very rare, and the proposed mechanism is decreased scleral rigidity, which leads to increased tractional forces on the peripheral retina.

Rationale Continued: Other complications include basilar skull invagination, which can occur due to pressure on the spinal column at the skull base. This can lead to headaches, muscle weakness, and paresthesias . Persons with OI can develop both a conductive and a sensorineural hearing loss due to development of ossicular dislocation, stapes fixation, or fractures of the ossicles. Fifty percent of persons with OI develop some degree of hearing loss. Restrictive lung disease, rather than obstructive lung disease, can develop secondary to kyphosis, which results in decreased chest volume. Other contributing factors include rib fractures, muscle weakness, chronic bronchitis, and asthma. Some patients develop related sleep apnea. Nephrocalcinosis is more common in persons with OI than in the general population, and up to 20% of affected individuals develop kidney stones. Bone cancer (Answer C) such as sarcoma is not associated with OI. Recommended screening for adult patients with OI includes a hearing test, assessment of bone mineral density, and spirometry every 2 years; electrocardiogram and echocardiogram every 2 years (in patients with type 3 deforming OI or other moderate to severe types); neurologic examination and cranial assessment as indicated by symptoms or behavioral changes; and skeletal radiographs at the time of diagnosis and then every 1 to 2 years.

Reference(s): Bishop N. Characterising and treating osteogenesis imperfecta. Early Hum Dev . 2010;86(11):743-746. PMID: 20846798van Dijk FS, Cobben JM, Kariminejad A, et al. Osteogenesis imperfecta: a review with clinical examples. Mol Syndromol. 2011;2(1):1-20. PMID: 22570641

ITE 2017 Question 64 A 32-year-old man presents with hip pain and the radiographic findings shown (see image).

Laboratory test results: Serum calcium = 8.2 mg/ dL (8.2-10.2 mg/ dL ) (SI: 2.1 mmol/L [2.1-2.6 mmol/L])Phosphorus = 2.2 mg/ dL (2.3-4.7 mg/ dL ) (SI: 0.7 mmol /L [0.7-1.5 mmol /L]) Creatinine = 0.9 mg/ dL (0.7-1.3 mg/ dL ) (SI: 79.6 µ mol/L [61.9-114.9 µmol/L]) Serum alkaline phosphatase = 346 U/L (50-120 U/L) (SI: 5.78 µ kat /L [0.84-2.00 µ kat /L])

Measurement of which of the following is most likely to provide this patient’s diagnosis? Fibroblast growth factor 23 1,25-Dihydroxyvitamin D 25-Hydroxyvitamin D PTH C-telopeptide Correct Answer: C Learning objective : Identify clinical and radiographic findings in osteomalacia (severe vitamin D deficiency).

Rationale: The radiograph shows a Looser zone, characteristic of osteomalacia . Mechanical stress of blood vessels overlying the uncalcified cortical bone affected by osteomalacia is thought to cause “pseudo fractures” that appear as transverse zones of rarefaction, sometimes as wide as 1 cm, often multiple, and generally symmetric. Typical locations are the ischium, ilium, pubis, femur, tibia, radius, fibula, lower ribs, and scapula. This patient has a longstanding history of celiac disease and nonadherence to dietary recommendations. His serum 25-hydroxyvitamin D level (Answer C) was undetectable (<7 ng/mL [<17.5 nmol /L]). High-dosage vitamin D3, 100,000 IU daily, did not correct the vitamin D deficiency, but ultraviolet light (tanning salon) was successful. Chemical clues to osteomalacia include hypocalcemia, hypophosphatemia, and elevated alkaline phosphatase (thus, Answers A, B, D, and E are incorrect).

Reference(s): Reginato AJ, Falasca GF, Pappu R, McKnight B, Agha A. Musculoskeletal manifestations of osteomalacia: report of 26 cases and literature review. Semin Arthritis Rheum. 1999;28(5):287-304. PMID: 10342386Lips P, van Schoor NM. The effect of vitamin D on bone and osteoporosis. Best Pract Res Clin Endocrinol Metab. 2011;25(4):585-591. PMID: 21872800Thacher TD, Clarke BL. Vitamin D insufficiency. Mayo Clin Proc . 2011;86(1):50-60. PMID: 21193656 Bhan A, Rao AD, Rao DS. Osteomalacia as a result of vitamin D deficiency. Endocrinol Metab Clin North Am . 2010;39(2):321-331. PMID: 20511054

ITE 2017 Question 67 A 27-year-old man is referred to you for evaluation of hypocalcemia. He has a long history of bone pain and has only recently sought medical attention because his symptoms have worsened. Recent laboratory testing revealed low calcium and elevated PTH values. He has no fracture history and has never had a kidney stone. He is a high school graduate and reports his academic achievement was average. Additional medical problems include hypothyroidism and hypogonadism (also recently diagnosed); he takes appropriate replacement therapy and is adherent to this regimen. His local physician told him that his condition may run in families. The patient and his wife have a 7-year-old son, so the patient has questions regarding inheritance. The patient is adopted and his family history is unavailable. On physical examination, he is a short, obese man with a round face and shortened fourth and fifth metacarpals bilaterally. His blood pressure is 139/86 mm Hg, and pulse rate is 77 beats/min. His height is 60 in (152.4 cm), and weight is 216 lb (98.2 kg) BMI = 42.2 kg/m2).

Laboratory test results: Total calcium = 7.7 mg/ dL (8.2-10.2 mg/ dL ) (SI: 1.9 mmol/L [2.1-2.6 mmol/L])25-Hydroxyvitamin D = 20 ng/mL (25-80 ng/mL [optimal] (SI: 49.9 nmol /L [62.4-199.7 nmol /L]) 1,25-Dihydroxyvitamin D = 45 pg /mL (16-65 pg /mL) (SI: 117 pmol /L [41.6-169.0 pmol /L]) Phosphorus = 6.5 mg/dL (2.3-4.7 mg/dL ) (SI: 2.1 mmol /L [0.7-1.5 mmol /L]) PTH = 475 pg/mL (10-65 pg/mL) (SI: 475 ng/L [10-65 ng/L])Creatinine = 2.3 mg/dL (0.7-1.3 mg/dL) (SI: 203.3 µmol/L [61.9-114.9 µmol/L])Albumin = 4.2 g/dL (3.5-5.0 g/dL) (SI: 42 g/L [35-50 g/L])TSH = 1.8 mIU/L (0.5-5.0 mIU/L)Free testosterone = 23 ng/dL (9.0-30 ng/dL) (SI: 0.80 nmol/L [0.31-1.04 nmol/L])

How should you counsel him regarding the likely inheritance of this disorder? He most likely inherited a gene mutation from his father; his son has a 50% chance of inheriting the mutation and being affected He most likely inherited a gene mutation from his mother; his son has a 50% chance of inheriting the mutation but likely will not have endocrine abnormalities He most likely inherited 2 gene mutations (1 from his mother and 1 from his father); his son is at risk only if the patient’s wife is a carrier The disorder results from a postzygotic gene mutation; his son has the same risk as the general population This disorder is idiopathic and has no clear genetic basis; his son has the same risk as the general population

Correct Answer: B Learning objective : Counsel patients about the inheritance and imprinting of pseudohypoparathyroidism .

Rationale: Pseudohypoparathyroidism represents a constellation of disorders characterized by end-organ resistance to PTH, primarily in the bone and kidney (see table). Type 1a pseudohypoparathyroidism is caused by a loss-of-function GNAS mutation that is manifested when the mutation is maternally inherited, and it is associated with Albright hereditary osteodystrophy. Disease-causing mutations result in an inability to activate adenylyl cyclase upon binding of PTH to the receptor and hence failure of signal transduction that reduces end-organ responsiveness to PTH. Phenotypically, this disorder is characterized by a round face, short stature, shortened fourth and fifth metacarpals, obesity, and developmental delay. High phosphorus with secondary hyperparathyroidism can lead to bone disease. Resistance to TSH, LH, FSH, and GnRH may also occur. Biochemical findings are notable for low serum calcium and elevated phosphorus and PTH. The epigenetic imprinting associated with the GNAS gene leads to tissue-specific expression of the allele depending on whether the allele was inherited from the mother or father. Imprinting occurs through inactivation of genes in sperm or ova during gametogenesis. Maternal transmission of the mutation is required for manifestation of type 1a pseudohypoparathyroidism , as described in this patient. Renal expression of GNAS is determined by the maternal allele, thus explaining the unresponsiveness of renal tubule PTH binding.

Rationale Continued: The maternal allele is also responsible for GNAS expression in the thyroid, gonads, and pituitary, which also explains this patient’s hypothyroidism and hypogonadism. Paternal transmission presents with the Albright hereditary osteodystrophy phenotype, but without the biochemical abnormalities, and this is known as pseudopseudohypoparathyroidism . Thus, the patient’s son has a 50% chance of inheriting the GNAS mutation; however, because of imprinting, he likely will not have endocrine abnormalities (thus, Answer B is correct and Answers A, C, D, and E are incorrect).

Rationale Continued: Characteristics Type 1a Type 1b Type 1c Type II Pseudohypoparathyroidism Albright hereditary osteodystrophy phenotype   +   –   +   –   + PTH ⇑ ⇑ ⇑ ⇑ Normal Calcium level ⇓⇓⇓⇓NormalResponse to PTH infusionDecreased urinary cAMP and phosphorusDecreased urinary cAMP and phosphorusDecreased urinary cAMP and phosphorusNormal urinary cAMP and decreased phosphorusNormal urinary cAMP and phosphorusGNAS inactivating gene mutationMaternalImprinting defectFew case reportsNonePaternalHormone resistancePTH, TSH, LH, FSH, GHRHPTH, TSHPTH, TSH, LH, FSHPTHNone

Reference(s): Brandi ML. Genetics of hypoparathyroidism and pseudohypoparathyroidism . J Endocrinol Invest. 2011;34(7 Suppl):27-34. PMID: 21985977Bastepe M. The GNAS locus and pseudohypoparathyroidism. Adv Exp Med Biol. 2008;626:27-40. PMID: 18372789

ITE 2017 Question 74 A 67-year-old man with end-stage kidney disease due to hypertension has been receiving hemodialysis for 8 years. He comes to you because of multiple vertebral fractures, a pelvic fracture, and a femoral neck T score of −3.9. Current medications include calcitriol, 0.5 mcg twice daily, and cinacalcet , 90 mg twice daily. Laboratory test results: Serum calcium = 7.8 mg/dL (8.2-10.2 mg/ dL ) (SI: 2.0 mmol /L [2.1-2.6 mmol /L]) Albumin = 4.0 g/ dL (3.5-5.0 g/ dL ) (SI: 40 g/L [35-50 g/L]) Phosphorus = 5.2 mg/ dL (2.3-4.7 mg/dL) (SI: 1.7 mmol /L [0.7-1.5 mmol /L]) 25-Hydroxyvitamin D = 24 ng/mL (25-80 ng/mL [optimal]) (SI: 59.9 nmol/L [62.4-199.7 nmol/L])PTH = 78 pg/mL (10-65 pg/mL) (SI: 78 ng/L [10-65 ng/L])Total alkaline phosphatase = 125 U/L (50-120 U/L) (SI: 2.09 µkat/L [0.84-2.00 µkat/L])An iliac crest biopsy is done after double-tetracycline labeling and results will be available in 4 weeks.

While awaiting bone biopsy results, which of the following changes in management should be made immediately? Increase the calcitriol dosage Decrease or stop cinacalcet Add alendronate Add denosumab Add zoledronic acid Correct Answer: B Learning objective : Diagnose and manage adynamic bone disease in a patient undergoing dialysis.

Rationale: This patient’s PTH level is lower than the goal in patients undergoing dialysis and may indicate underlying adynamic bone disease. Because of the low PTH and hypocalcemia, the cinacalcet dosage should be reduced or stopped (Answer B). Adynamic bone disease, a type of renal osteodystrophy (now called chronic kidney disease mineral bone disorder [CKD-MBD]), is present in at least one-third of patients receiving dialysis. Adynamic bone disease is characterized by markedly low bone turnover, no accumulation of osteoid on bone histomorphometry, and a high fracture risk. Serum PTH levels in adynamic bone disease are relatively low (usually <100 pg /mL [<100 ng/L]) compared with levels in patients undergoing dialysis who have other forms of CKD-MBD. Increasing the calcitriol dosage (Answer A) will further suppress PTH, which is not a desired outcome. Bisphosphonates such as zoledronic acid (Answer E) or alendronate (Answer C) are generally not recommended for use in patients with an estimated glomerular filtration rate less than 30 to 35 mL/min per 1.73 m2. Although denosumab (Answer D) can be used in patients receiving dialysis, it would be inappropriate to administer it now in the face of hypocalcemia and vitamin D deficiency and before excluding the presence of adynamic bone disease. Impaired mineralization, osteitis fibrosa , and mixed renal osteodystrophy are other forms of CKD-MBD, but these diagnoses are unlikely given the laboratory findings (high bone turnover would be associated with a higher PTH level; osteomalacia would be associated with a very low 25-hydroxyvitamin D level; and mixed renal osteodystrophy would have both).

Reference(s): Coen G. Adynamic bone disease: an update and overview. J Nephrol . 2005;18(2):117-122. PMID: 15931639Hruska KA, Seifert M. Pathophysiology of chronic kidney disease mineral bone disorder. In: Rosen CJ, Bouillon R, Compston JE, Rosen V, eds. Primer on the Metabolic Bone Diseases and Disorders of Mineral Metabolism. 8th ed. Washington, DC: American Society for Bone and Mineral Research; 2013: 632-639.Alshayeb HM, Quarles LD. Treatment of chronic kidney disease mineral bone disorder. In: Rosen CJ, Bouillon R, Compston JE, Rosen V, eds. Primer on the Metabolic Bone Diseases and Disorders of Mineral Metabolism. 8th ed. Washington, DC: American Society for Bone and Mineral Research; 2013: 640-650.Brandenburg VM, Floege J. Adynamic bone disease: bone and beyond. NDT Plus. 2008;1(3):135-147. PMID: 25983860 Kidney Disease: Improving Global Outcomes (KDIGO) CKD-MBD Work Group. KDIGO clinical practice guideline for the diagnosis, evaluation, prevention, and treatment of chronic kidney disease-mineral and bone disorder (CKD-MBD). Kidney Int . 2009;( Suppl 113):S1-S130. PMID: 19644521

ITE 2017 Question 80 An oncologist refers to you a 55-year-old man for evaluation of new-onset hypercalcemia. The patient has asymptomatic, diffuse bone lymphoma that has not required treatment. He describes experiencing malaise and fatigue. Laboratory test results: Serum calcium = 12.5 mg/ dL (8.2-10.2 mg/dL) (SI: 3.1 mmol /L [2.1-2.6 mmol /L]) Albumin = 4.1 mg/ dL (3.5-5.0 g/ dL ) (SI: 41 g/L [35-50 g/L]) Creatinine = 0.9 mg/ dL (0.7-1.3 mg/ dL ) (SI: 79.6 µmol/L [61.9-114.9 µmol /L]) 25-Hydroxyvitamin D = 25 ng/mL (25-80 ng/mL [optimal]) (SI: 62.4 nmol /L [62.4-199.7 nmol /L])1,25-Dihydroxyvitamin D = 52 pg/mL (16-65 pg/mL) (SI: 135.2 pmol/L [41.6-169.0 pmol/L])PTH = 5 pg/mL (10-65 pg/mL) (SI: 5 ng/L [10-65 ng/L])PTHrP, undetectable

Which of the following is the best treatment for his hypercalcemia? Cinacalcet Treatment of the lymphoma Recombinant PTH Oral furosemide Nasal calcitonin Correct Answer: B Learning objective : Recognize inappropriately high 1,25-dihydroxyvitamin D due to lymphoma.

Rationale: The mediator of this patient’s hypercalcemia is 1,25-dihydroxyvitamin D being produced by the lymphoma (while not overtly elevated, this value is higher than would be expected for his hypercalcemia and low PTH, both of which should decrease the activity of 1α-hydroxylase). Thus, treatment of the lymphoma (Answer B) will normalize his serum calcium. Recombinant PTH (Answer C) is used to treat hypocalcemia in hypoparathyroidism, but this patient’s low PTH level is appropriate. Oral furosemide (Answer D) will not improve his hypercalcemia, and cinacalcet (Answer A) is appropriate only for treating PTH-mediated hypercalcemia. Calcitonin by subcutaneous injection is used in the treatment severe symptomatic hypercalcemia, but nasal calcitonin (Answer E) is not effective for this indication.

Reference(s): Kallas M, Green F, Hewison M, White C, Kline G. Rare causes of calcitriol-mediated hypercalcemia: a case report and literature review. J Clin Endocrinol Metab. 2010;95(7):3111-3117. PMID: 20427501Horwitz MJ, Hodak SP, Stewart AF. Non-parathyroid hypercalcemia. In: Rosen CJ, Bouillon R, Compston JE, Rosen V, eds. Primer on the Metabolic Bone Diseases and Disorders of Mineral Metabolism. 8th ed. Washington, DC: American Society for Bone and Mineral Research; 2013: 562-571.

ITE 2017 Question 89 A 39-year-old man is referred to you because of low bone mass identified at a health fair (bone density of the heel assessed by ultrasonography was low). He has no notable medical history, but he does have a family history of osteoporosis. He does not smoke cigarettes and does not consume much alcohol. On physical examination, his blood pressure is 120/74 mm Hg and heart rate is 68 beats/min. His height is 69 in (175.3 cm), and weight is 180 lb (81.8 kg) (BMI = 26.6 kg/m2). Sclerae are white. He has no signs of Cushing syndrome, thyroid dysfunction, or ecchymoses , but he has a diffuse macular rash, especially on his trunk, with a positive Darier sign and associated pruritus. A photograph is shown.

DXA bone mineral density measurements: After reviewing normal values from a complete blood cell count and measurement of electrolytes, serum urea nitrogen, creatinine, calcium, and albumin, you order additional laboratory tests. Region BMD, g/cm 2 T score Z score L1-L4 0.746 –3.1 –3.0 Hip 0.847 –1.2 –0.8 Femoral neck 0.663 –2.0 –1.4

Which one of the following laboratory test results is most consistent with this clinical scenario? Total testosterone = 98 ng/ dL (3.4 nmol /L)25-Hydroxyvitamin D = 22 ng/mL (54.9 nmol/L)PTH = 65 pg /mL (65 ng/L) Tryptase = 78 ng/mL (78 µg/L) Calcitonin = 50 pg/mL (14.6 pmol/L) Correct Answer: D Learning objective : Diagnose mastocytosis as a secondary cause of osteoporosis.

Rationale: In young patients, particularly premenopausal women and men younger than 50 years, secondary causes of osteoporosis must be investigated. In men, testosterone deficiency, glucocorticoid use, and alcohol and tobacco use are the most common causes of low bone density. Mastocytosis is an important occult cause. In this case, mastocytosis is recognized as a rash known as urticaria pigmentosa . Urticaria pigmentosa has been reported in a substantial number of patients with systemic mastocytosis . When a rash is present, the diagnosis can be confirmed by skin biopsy, but in the absence of skin findings, a bone biopsy is confirmatory. Elevated serum tryptase (Answer D) supports the diagnosis, but it may be normal in some cases. Tryptase is a protease that is produced predominantly in mast cells, although a small amount is made by basophils as well. The presence of elevated serum concentrations of tryptase is one of the minor criteria for the diagnosis of mastocytosis . Measurement of N-methylhistamine in a 24-hour urine collection has been used in the diagnosis of systemic mastocytosis, particularly in the past before tryptase could be measured. Tryptase and N-methylhistamine measurements are noninvasive surrogates for bone marrow biopsy in establishing the diagnosis.

Rationale Continued: The pathognomonic feature of mastocytosis is bone marrow infiltration of morphologically abnormal mast cells. In addition to skin and bone marrow involvement, abnormal proliferation of mast cells can also be found in the liver, lymph nodes, and spleen. The clinical manifestation is due to the liberation of mast-cell products. Patients can be asymptomatic or can have fatigue, flushing, bone and muscle pain, diarrhea, nausea, vomiting, and weight loss. Osteoporosis may be associated with systemic manifestations of mast-cell activity or it may be the sole presentation of bone marrow mastocytosis . Fractures have been reported in 16% of patients. Factors released by the mast cells that affect bone turnover include heparin, histamine, and cytokines such as interleukin-6. In some trials, bisphosphonates have been reported to improve bone mineral density. Analgesics are given for bone pain, and antihistamines are given for other symptoms. The disease has no cure. A mutation in the gene encoding c-kit (the receptor for stem cell factor) that controls mast-cell development has been identified. Hypogonadism (Answer A), vitamin D insufficiency (Answer B), and secondary hyperparathyroidism (Answer C) are all secondary causes of osteoporosis, and additional laboratory tests are needed to confirm or exclude these conditions in any young man presenting with low bone density. Although calcitonin is a hormone that can affect calcium concentrations, conditions producing excess calcitonin, such as medullary thyroid cancer (Answer E), or conditions that remove calcitonin, such as after a total thyroidectomy, do not lead to osteoporosis. In this particular case, the rash is the hallmark of mastocytosis and is not seen in these other conditions.

Reference(s): Rossini M, Zanotti R, Bonadonna P, et al. Bone mineral density, bone turnover markers and fractures in patients with indolent systemic mastocytosis. Bone. 2011;49(4):880-885. PMID: 21782049

DIABETES

ITE 2017 Question 2 A 24-year-old woman is referred for evaluation after developing hyperglycemia during a hospitalization for pneumonia. She has cystic fibrosis and was admitted to the hospital last month for an exacerbation of chronic pulmonary disease thought to be due to bacterial infection. During the early period of her hospital stay for antibiotic treatment, several blood glucose values were documented to be greater than 200 mg/ dL (>11.1 mmol /L), but not persisting for more than 48 hours. Fasting blood glucose levels were below 100 mg/dL (<5.6 mmol /L) the 2 days before discharge. She had been generally well in the year before hospitalization, but she did note an 8.8-lb (4-kg) weight loss over that time. On physical examination, her BMI is 22 kg/m2. She has a resonant chest with decreased breath sounds and decreased muscle mass and subcutaneous fat.

Which of the following is the most appropriate diagnostic strategy now? Oral glucose tolerance test Continuous glucose monitoring Hemoglobin A1c measurement A mixed-meal tolerance test Measurement of fasting blood glucose on 2 occasions Correct Answer: A Learning objective : Recommend the oral glucose tolerance test to confirm the diagnosis of cystic fibrosis–related diabetes.

Rationale: With improved treatment, the life expectancy of patients with cystic fibrosis is increasing and nonpulmonary manifestations of the disease are becoming more prevalent. Cystic fibrosis–related diabetes affects 2% of children, 20% of adolescents, and nearly 50% of adults with cystic fibrosis. The principle etiology is islet-cell damage as a result of pancreatic fibrosis, but patients with cystic fibrosis–related diabetes also have slightly reduced insulin sensitivity compared with that of nondiabetic patients with cystic fibrosis. The hallmark of cystic fibrosis–related diabetes is postprandial hyperglycemia out of proportion with the fasting blood glucose level; that is, most patients present with results from oral glucose tolerance testing indicative of diabetes and normal fasting blood glucose levels (thus, Answer E is incorrect). The oral glucose tolerance test (Answer A) has become the criterion standard for diagnosis and is the test that has been studied most frequently in patients with cystic fibrosis. Continuous glucose monitoring (Answer B) has been evaluated recently and gives comparable diagnostic results to the oral glucose tolerance test, but it has not yet been standardized for clinical practice. Hemoglobin A1c measurement (Answer C) tends to underestimate the degree of glucose intolerance in cystic fibrosis and is not used for diagnosis. The response to mixed-meal tests (Answer D) is often abnormal in the setting of cystic fibrosis–related diabetes, and it provides a more physiologic challenge to glucose regulation. However, this test has not been developed into a diagnostic standard.

Reference(s): O’Shea D, O’Connell J. Cystic fibrosis related diabetes. Curr Diab Rep. 2014;14(8):511. PMID: 24915888Kelly A, Moran A. Update on cystic fibrosis-related diabetes [published correction appears in J Cyst Fibros. 2014;13(1):119]. J Cyst Fibros. 2013;12(4):318-331. PMID: 23562217Moran A, Brunzell C, Cohen RC, et al; CFRD Guidelines Committee. Clinical care guidelines for cystic fibrosis-related diabetes: a position statement of the American Diabetes Association and a clinical practice guideline of the Cystic Fibrosis Foundation, endorsed by the Pediatric Endocrine Society. Diabetes Care. 2010;33(12):2697-2708. PMID: 21115772

ITE 2017 Question 5 A 48-year-old woman with a 38-year history of type 1 diabetes mellitus has been on insulin pump therapy for 11 years with a rapid-acting insulin analogue. She has recently been experiencing recurrent high fasting blood glucose values and random episodes of hypoglycemia. You examine her log book, which documents self-monitored blood glucose levels 4 to 5 times daily before meals, several glucose values at random times in the 40s and 50s, and fasting levels most mornings in the high 100s to low 200s, although occasional values of 70 to 150 mg/ dL are noted. She has a bedtime snack of 15 g of carbohydrate each evening. The patient reports that she has no symptoms associated with the low blood glucose values. She is asking for advice about achieving better glycemic control.

Which of the following is the best recommendation? Begin monitoring glucose levels with a continuous glucose sensor Increase the basal insulin rate 2 hours before the time that fasting hyperglycemia is occurring Increase prandial insulin before the evening meal Give a bolus of insulin to cover her bedtime snack Eliminate the bedtime snack Correct Answer: A Learning objective : Recommend continuous glucose monitoring to detect hypoglycemia and devise strategies for its reduction in patients with longstanding type 1 diabetes mellitus.

Rationale: This patient exhibits hypoglycemia unawareness. She also has fasting hyperglycemia, which should raise suspicion that she is experiencing hypoglycemia overnight that is going undetected and is followed by rebound hyperglycemia in the morning. Therefore, increasing the basal rate 2 hours before the morning hyperglycemia is typically observed (Answer B) is not the correct recommendation, as it would most likely worsen the situation. Increasing evening prandial insulin (Answer C), covering the bedtime snack (Answer D), or eliminating the bedtime snack (Answer E) would help the morning hyperglycemia only if the patient remains hyperglycemic through the night, which is unknown. To determine whether this is the case, it would be necessary to check the overnight glucose level by fingerstick (not an offered option) or a continuous glucose sensor (Answer A), which would also aid with detection of hypoglycemia that the patient is having at other times of the day, so this is the best choice.

Reference(s): Yeh H-C, Brown TT, Maruthur N, et al. Comparative effectiveness and safety of methods of insulin delivery and glucose monitoring for diabetes mellitus: a systematic review and meta-analysis. Ann Intern Med . 2012;157(5):336-347. PMID: 22777524Wolpert HA. Use of continuous glucose monitoring in the detection and prevention of hypoglycemia. J Diabetes Sci Technol. 2007;1(1):146-150. PMID: 19888397DeWitt DE, Hirsch IB. Outpatient insulin therapy in type 1 and type 2 diabetes mellitus: scientific review. JAMA. 2003;289(17):2254-2264. PMID: 12734137Oyer DS. The science of hypoglycemia in patients with diabetes. Curr Diabetes Rev. 2013;9(3):195-208. PMID: 23506375

ITE 2017 Question 8 A 28-year-old woman with type 1 diabetes mellitus is estimated to be 6 weeks pregnant. Her diabetes is complicated by retinopathy, albuminuria, and hypertension treated with lisinopril . She is using a combination of basal insulin with fixed prandial doses of a rapid-acting insulin analogue. Her overnight (3 AM) and fasting blood glucose levels range between 110 and 122 mg/ dL (6.1 and 6.8 mmol/L), and her 1-hour postprandial glucose levels range between 112 and 129 mg/dL (6.2 and 7.2 mmol /L).

In addition to stopping lisinopril , which of the following should you recommend during this pregnancy? Increase basal insulin to achieve fasting glucose levels <90 mg/ dL (<5.0 mmol/L) Start prandial supplemental insulin to achieve 1-hour postprandial glucose levels <120 mg/ dL (<6.7 mmol /L) Begin insulin administration via a continuous subcutaneous insulin infusion pump Continue to monitor hemoglobin A1c every 3 months Recommend no therapy changes; her current level of glycemic control is adequate Correct Answer: A Learning objective : Make basic recommendations for management of glycemia during pregnancy.

Rationale: During pregnancy, normalization of blood glucose levels reduces the risk of congenital malformations during the first 8 to 10 weeks and the risk of macrosomia and related comorbidities over the course of the pregnancy. The American Diabetes Association Standards of Medical Care in Diabetes 2016 endorses the American College of Obstetricians and Gynecologists recommendation of the following glucose targets for pregnant women with pregestational type 1 or type 2 diabetes: Fasting: <90 mg/ dL (5.0 mmol /L) 1-hour postprandial: <130-140 mg/ dL (7.2-7.8 mmol /L) 2-hour postprandial: <120 mg/ dL (<6.7 mmol /L) “These values represent optimal control if they can be achieved safely. In practice, it may be challenging for women with type 1 diabetes to achieve these targets without hypoglycemia, particularly women with a history of severe hypoglycemia or hypoglycemia unawareness. If women cannot achieve these targets without significant hypoglycemia, the American Diabetes Association (ADA) suggests less stringent targets based on clinical experience and individualization of care.”

Rationale Continued: This patient is not meeting the recommended fasting glycemic target, so an adjustment in therapy is needed (thus, Answer E is incorrect). Increasing her basal insulin to achieve fasting glucose levels less than 90 mg/ dL (<5.0 mmol /L) (Answer A) is the best recommendation.Although switching a patient’s regimen to insulin pump therapy before pregnancy to help optimize glycemic control is appropriate, changing to pump management during pregnancy (Answer C) is risky, given that there is a learning curve for pump use and glycemic control might temporarily deteriorate. Additionally, studies have not shown a clear benefit of continuous subcutaneous insulin infusion over multiple daily injections during pregnancy. Because her peak postprandial glucose measurements are within the target range on a fixed dose of prandial insulin, supplemental prandial insulin (Answer B) is not indicated. In pregnancy, there is increased red blood cell production and the red blood cell lifespan is shortened; hemoglobin A1c can therefore be checked monthly to monitor glycemia during pregnancy. Monitoring every 3 months (Answer D) is inadequate.

Reference(s): American Diabetes Association. 12. Management of diabetes in pregnancy. Diabetes Care . 2016;39( Suppl 1):S94-S98. PMID: 26696688Kitzmiller JL, Block JM, Brown FM, et al. Managing preexisting diabetes for pregnancy: summary of evidence and consensus recommendations for care. Diabetes Care. 2008;31(5):1060- 1079. PMID: 18445730Jovanovic-Peterson L, Peterson CM, Reed GF, et al. Maternal postprandial glucose levels and infant birth weight: the Diabetes in Early Pregnancy Study. The National Institute of Child Health and Human Development--Diabetes in Early Pregnancy Study. Am J Obstet Gynecol. 1991;164(1 Pt 1):103-111. PMID: 1986596McCance DR. Pregnancy and diabetes. Best Pract Res Clin Endocrinol Metab . 2011;25(6):945-958. PMID: 22115168 Page RC, Kirk BA, Fay T, Wilcox M, Hosking DJ, Jeffcoate WJ. Is macrosomia associated with poor glycaemic control in diabetic pregnancy? Diabetic Med . 1996;13(2):170-174. PMID: 8641124Mukhopadhyay A, Farrell T, Fraser RB, Ola B. Continuous subcutaneous insulin infusion vs intensive conventional insulin therapy in pregnant diabetic women: a systematic review and metaanalysis of randomized, controlled trials. Am J Obstet Gynecol. 2007;197(5):447-456. PMID: 17678864

ITE 2017Question 10 A 27-year-old woman has an 18-year history of type 1 diabetes mellitus. Nonproliferative diabetic retinopathy has been diagnosed, but she has no other complications of diabetes. She has been on insulin pump therapy for 3 years in an effort to improve glucose control after the recognition of the early retinopathy. This change substantially improved her hemoglobin A1c level, which is currently 6.4% (46 mmol /mol). She also reports frequent hypoglycemia, particularly between meals or before sleep. There has been at least 1 episode of severe hypoglycemia requiring assistance in the last 3 months (it occurred in the mid-afternoon). She reports no recognition of nocturnal hypoglycemia on her present insulin doses. She is frustrated by weight gain over the last 2 years and has tried to reduce her carbohydrate intake, but finds this effort has resulted in more hypoglycemia. Insulin pump settings are listed (she uses the calculator function about 65% of the time). Basal rates: Midnight 0.6 units/h 3 AM 0.525 units/h 9 AM 0.55 units/h 9 PM 0.525 units/h

Insulin-to-carbohydrate ratio: 1:7 g of carbohydrate Sensitivity ratio: 1:50 Target Glucose: 130 mg/ dL (7.2 mmol/L)On physical examination, her height is 65 in (165.1 cm) and weight is 147 lb (66.8 kg) (BMI = 24.5 kg/m2). Her blood pressure is 125/65 mm Hg, and pulse rate is 71 beats/min and regular. Physical examination findings are unremarkable. Review of her individual glucose levels confirms frequent monitoring with many values less than 60 mg/ dL (<3.3 mmol /L) during the day, often about 2 to 3 hours after meals. Download of data from her pump reveals the average total daily dose of insulin to be 33.35 units and the dose of basal insulin to be 13.125 units. She averages about 138 g of carbohydrates daily. She usually changes her sets every 3 days. Measurement Prebreakfast Prelunch Pre-evening Meal Bedtime Glucose, mean (±SD) 145 ± 48 mg/ dL (8.0 ± 2.7 mmol /L) 111 ± 73 mg/ dL (6.2 ± 0.4 mmol/L)127 ± 89 mg/dL(7.0 ± 4.9 mmol/L)138 ± 81 mg/dL(7.7 ± 4.5 mmol/L)

Which of the following would be an appropriate change in her treatment regimen? Increase the carbohydrate content in her diet to achieve more consistent glucose levels Change the insulin-to-carbohydrate ratio to 1 unit per 12 g of carbohydrate Decrease the daytime basal rate by 15% Decrease basal and bolus insulin doses by 10% Decrease the sensitivity ratio to 1:75 Correct Answer: B Learning objective : Identify pitfalls in insulin pump therapy and use insulin-dosing guidelines to adjust the insulin regimen.

Rationale: A standard deviation greater than 50% of the mean glucose value is usually indicative of high glucose variability at that time point. This is the case for the patient’s glucose values measured during the day. She has good glucose control as measured by hemoglobin A1c, but undoubtedly has excess hypoglycemia as evidenced by the high glucose variability observed during the day. She has excess insulin relative to her needs during the day. Her weight has probably increased partly because of excess insulin and the related frequent requirement for unintended calories to treat hypoglycemia. The primary therapeutic goal in this case is to reduce variability and hypoglycemia. The most appropriate response to hypoglycemia after the meal is to reduce the meal-related insulin doses. Relaxing the insulin-to-carbohydrate ratio (Answer B) would help the problem. For individuals eating typical meals, the basal-to-bolus insulin ratios are generally near 50:50. Further review indicates that her bolus insulin represents more than 60% of her total daily dose of insulin, which seems quite aggressive. Increasing the carbohydrate content in her diet (Answer A) is incorrect. The number of carbohydrates she eats per day is not unreasonable, and for an individual concerned about weight gain, increased carbohydrates are not needed. Furthermore, careful evaluation of the situation identifies other issues that account for her problems.

Rationale Continued: Decreasing the daytime basal rate by 15% (Answer C) may help reduce hypoglycemia, but it will not solve the real cause of her problem. Given that her hypoglycemia usually occurs after meals and her daytime basal insulin dose is quite low compared with the bolus insulin dose, one should address the possibility of excess bolus insulin first. Decreasing basal and bolus insulin doses by 10% (Answer D) is incorrect because there is no need to decrease basal insulin. The patient does not seem to have hypoglycemia at night and has a more consistent glucose level in the morning than other times of the day, indicating that the night basal doses are appropriate. On the basis of her glucose levels, which are below or just above the target glucose, she rarely requires correctional insulin. Therefore, relaxing the insulin sensitivity ratio from 1:50 to 1:75 (Answer E) would not solve her hypoglycemia.

Reference(s): Peters AL, Ahmann AJ, Battelino T, et al; Endocrine Society. Diabetes technology – continuous subcutaneous insulin infusion therapy and continuous glucose monitoring in adults: An Endocrine Society Clinical Practice Guideline. J Clin Endocrinol Metab. 2016;101(11):3922-3937. PMID: 27588440Lenhard MJ, Reeves GD. Continuous subcutaneous insulin infusion: a comprehensive review of insulin pump therapy. Arch Intern Med. 2001;161(19):2293-2300. PMID: 11606144

ITE 2017 Question 17 A 41-year-old woman with a 12-year history of type 1 diabetes mellitus is planning to undergo an isolated pancreas transplant. She has always been highly adherent to therapy and her diabetes was most recently managed with an insulin pump. However, she had very volatile glucose readings and hypoglycemic unawareness prompting evaluation for transplant. She has early-stage chronic kidney disease, but not severe enough to indicate a renal transplant, and she has no evidence of macrovascular diabetes complications. She has proliferative retinopathy treated with laser therapy and some numbness in her feet. Her family history is notable for osteoporosis with spinal fractures in her mother and a maternal aunt. On physical examination, her blood pressure is 128/78 mm Hg. Her height is 64 in (162.6 cm), and weight is 142 lb (64.5 kg) (BMI = 24.4 kg/m2). She has diminished sensation to monofilament, but intact ankle reflexes.

Laboratory test results: Hemoglobin A1c = 7.8% (4.0%-5.6%) (SI: 62 mmol / mol [20-38 mmol/mol])Creatinine = 1.9 mg/ dL (0.6-1.1 mg/ dL ) (SI: 168.0 µ mol /L [53.0-97.2 µ mol /L]) Urinary albumin-to-creatinine ratio = 98 mg/g creat (<17 mg/g creat [male]) Total cholesterol = 142 mg/dL (<200 mg/dL [optimal]) (SI: 3.68 mmol /L [<5.18 mmol /L]) LDL cholesterol = 81 mg/dL (<100 mg/dL [optimal]) (SI: 2.10 mmol/L [<0.59 mmol/L])HDL cholesterol = 43 mg/dL (>60 mg/dL [optimal]) (SI: 1.11 mmol/L [>1.55 mmol/L])Triglycerides = 90 mg/dL (<150 mg/dL [optimal]) (SI: 1.02 mmol/L [<3.88 mmol/L])

In addition to a reduction in hypoglycemia, which of the following outcomes can this patient expect following a successful pancreas transplant? Reduced albuminuria Reduced cardiovascular risk Reduced osteoporosis risk Regression of retinopathy Recovery of peripheral sensation Correct Answer: A Learning objective : Predict outcomes of pancreas transplant in patients with diabetes mellitus.

Rationale: In the setting of successful pancreas transplant, there is restoration of euglycemia . However, the beneficial effects of euglycemia are counteracted by potential toxic effects of the immunosuppressants used. Thus, outcomes of successful pancreas transplant are balanced between these effects. The main indication for pancreas transplant is labile glucose control with hypoglycemia unawareness, as illustrated in this patient’s case. Rates of graft survival and patient survival with pancreas transplant have improved, and they are similar to those seen with simultaneous pancreas and kidney transplant. Creatinine is a better biomarker than elevated glucose for signaling early rejection, which allows for earlier recognition and intervention for rejection in pancreas and kidney transplant. As pancreas transplant alone (in the absence of renal transplant) is performed at a lower frequency than simultaneous pancreas and kidney transplant, relatively few data are available regarding long-term outcomes such as diabetes complications. In patients with end-stage renal disease, there are many benefits of receiving a renal transplant, making it difficult to determine which benefits are driven primarily from renal vs pancreas transplant. Several small studies have reported that early diabetic nephropathy can improve with pancreas transplant. In a small series of 8 patients with mild to moderate albuminuria, no benefits were seen regarding albumin excretion within the first 5 years after transplant, but normalization of urinary albumin excretion and histopathologic features of glomerular disease occurred 5 to 10 years after pancreas transplant. Furthermore, some regression of renal pathology has been seen on biopsies (thus, Answer A is correct).

Rationale Continued: The effect of pancreas transplant on neuropathy and retinopathy has not been quite as good. Several studies have reported stabilization of neuropathy and retinopathy, but improvement tends to be quite rare, especially in those who have advanced retinopathy, as is the case for this patient (thus, Answer D is incorrect). While nerve conduction studies may improve following pancreas transplant, there is little improvement in clinical symptoms or on examination (thus, Answer E is incorrect). The effect of pancreas transplant on cardiovascular mortality is confusing. In general, pancreas transplant improves lipid parameters, which would be anticipated to decrease atherosclerotic and thus cardiovascular risk. However, in the early postoperative period, there is often an increase in cardiovascular risk. Most studies demonstrating improved cardiovascular outcomes examined individuals who receive combined kidney + pancreas transplant; clear evidence shows that successful renal transplant improves cardiovascular outcomes when compared with outcomes in individuals with end-stage renal disease. However, the patient in this vignette does not have end-stage renal disease. Thus, it is unlikely that pancreas transplant will improve her risk further (thus, Answer B is incorrect).

Rationale Continued: Type 1 diabetes is associated with decreased bone mineral density, at least in part due to defects in osteoblast activity. While it is unclear what effect restoring euglycemia has on bone health, it is well established that many immunosuppressants have deleterious effects on bones and that transplant- associated osteoporosis is a common problem. Thus, although data on her current bone density are not provided in the vignette, her bone density is likely to deteriorate with immunosuppression (thus, Answer C is incorrect).

Reference(s): Fioretto P, Steffes MW, Sutherland DE, Goetz FC, Mauer M. Reversal of lesions of diabetic nephropathy after pancreas transplantation. N Engl J Med. 1998;339(2):69-75. PMID: 22241641Mittal S, Gough SC. Pancreas transplantation: a treatment option for people with diabetes. Diabet Med. 2014;31(5):512-521. PMID: 24313883

ITE 2017 Question 24 A 48-year-old man in whom type 2 diabetes mellitus was diagnosed 6 years ago presents for recommendations on how to best reduce his cardiovascular disease risk. His diabetes has been complicated by background retinopathy. He has no history of hyperlipidemia, but his internist prescribed a statin for cardiovascular protection. He follows a low-fat, carbohydrate-moderated, high-fiber diet and he walks 30 minutes 3 to 4 days per week. He does not smoke cigarettes. He does not have a personal or family history of early cardiovascular disease. He has no exertional chest pain or leg claudication. His current medications are atorvastatin, 20 mg daily; metformin, 1000 mg twice daily; and glimepiride, 4 mg daily. On physical examination, his height is 72 in (182.9 cm) and weight is 188 lb (85.5 kg) (BMI = 25.5 kg/m2). His blood pressure is 131/76 mm Hg, and pulse rate is 84 beats/min. He has no jugular venous distention, and examination findings of his heart, lungs, abdomen, extremities, and pulses are normal. There are no carotid bruits. Findings from an electrocardiogram are normal.

Laboratory test results: Hemoglobin A1c = 7.3% (4.0%-5.6%) (56 mmol / mol [20-38 mmol/mol])Total cholesterol = 143 mg/ dL (<200 mg/ dL [optimal]) (SI: 3.70 mmol /L [<5.18 mmol /L]) LDL cholesterol = 68 mg/ dL (<100 mg/ dL [optimal]) (SI: 1.76 mmol/L [<2.59 mmol/L]) HDL cholesterol = 46 mg/ dL (>60 mg/ dL [optimal]) (SI: 1.19 mmol/L [>1.55 mmol/L])Triglycerides = 139 mg/dL (<150 mg/dL [optimal]) (SI: 1.57 mmol/L [<3.88 mmol/L])Urinary albumin-to-creatinine ratio = 6 mg/g creat (<30 mg/g creat)

Which of the following do you recommend to reduce his cardiovascular risk? Initiate low-dosage aspirin Initiate clopidogrel Initiate an ACE inhibitor Initiate empagliflozin No changes to his current medications Correct Answer: E Learning objective : Discuss prevention of cardiovascular disease in patients with type 2 diabetes mellitus and low cardiovascular risk.

Rationale: Increased platelet reactivity has been demonstrated in patients with diabetes and is proposed as one of the mechanisms for increased cardiovascular risk in this population. The antiplatelet agent aspirin reduces cardiovascular events and cardiovascular mortality in patients at high risk who have had a previous myocardial infarction or stroke. Aspirin use as primary prevention of cardiovascular events is more controversial. A meta-analysis of large trials of aspirin used for primary prevention showed a 12% reduction in major cardiovascular events among patients with diabetes, a rate similar to that observed in the general population. The major observed risk of aspirin therapy is gastrointestinal bleeding, estimated to be 1 to 5 events per 1000 person-years. Patients who benefit most from aspirin therapy are those at high baseline cardiovascular risk. For this reason, low-dosage aspirin is recommended in those at increased risk, including men and women 50 years or older who have diabetes and 1 or more additional major risk factor (hypertension, dyslipidemia, smoking, albuminuria, or family history of early cardiovascular disease). This represents a population with a 10-year atherosclerotic cardiovascular disease risk greater than 10% (Framingham Heart Study risk assessment calculator). The patient in this vignette is at low cardiovascular risk and low-dosage aspirin (Answer A) is not indicated.

Rationale Continued: Clopidogrel (Answer B) has not been well studied for primary prevention of cardiovascular events. Its use is recommended in patients with atherosclerotic cardiovascular disease and aspirin allergy, as well as in combination with aspirin after an acute coronary syndrome, but not for primary prevention. The recommended blood pressure target in persons with diabetes is less than 140/90 mm Hg. More aggressive therapy is recommended for patients who are younger, those with albuminuria, and those who have additional major risk factors for cardiovascular disease. Use of ACE inhibitors (Answer C) and angiotensin receptor blockers may reduce cardiovascular disease events in normotensive patients with diabetes and known atherosclerotic vascular disease, but these agents are not recommended in normotensive patients with diabetes who have no other cardiovascular risk factors. A large study in patients with type 2 diabetes at high cardiovascular risk showed that treatment with empagliflozin (Answer D) significantly lowered risk of death from cardiovascular causes, hospitalization for heart failure, and death of any cause when compared with placebo. However, the effects of empagliflozin on cardiovascular outcomes have not been studied in patients with diabetes at low risk. In this patient with type 2 diabetes and low cardiovascular risk, no additional interventions are indicated (thus, Answer E is correct).

Reference(s): Pignone M, Alberts MJ, Colwell JA, Cushman M, Inzucchi SE, Mukherjee D, et al; American Diabetes Association; American Heart Association; American College of Cardiology Foundation. Aspirin for primary prevention of cardiovascular events in people with diabetes: a position statement of the American Diabetes Association, a scientific statement of the American Heart Association, and an expert consensus document of the American College of Cardiology Foundation [published corrections appear in Diabetes Care. 2011;34(1):247-248 and Diabetes Care . 2010;33(9):2129-2131]. Diabetes Care. 2010;33(6):1395-1402. PMID: 20508233Antithrombotic Trialists’ (ATT) Collaboration, Baigent C, Blackwell L, Collins R, Emberson J, Godwin J, et al. Antithrombotic Trialists’ (ATT) Collaboration. Aspirin in the primary and secondary prevention of vascular disease: collaborative meta-analysis of individual participant data from randomised trials. Lancet. 2009;373(9678):1849-1860. PMID: 19482214 Zinman B, Wanner C, Lachin JM, Fitchett D, Bluhmki E, Hantel S, et al; EMPA-REG OUTCOME Investigators. Empagliflozin , cardiovascular outcomes, and mortality in type 2 diabetes. N Engl J Med. 2015;373(22):2117-2128. PMID: 26378978

ITE 2017 Question 31 A 19-year-old man with type 1 diabetes mellitus reports a 6-month history of diarrhea, unintentional weight loss, poor glycemic control, and a rash. On physical examination, he has no abdominal tenderness, a normal 10-g monofilament test, and a rash that is characterized by erythematous, papular , small and large blisters that burn and itch intensely.

Which of the following is the best initial approach to evaluate his symptoms? Colonoscopy Upper gastrointestinal series with small-bowel follow-through Serum gastrin level Tissue transglutaminase and/or deaminated gliadin antibody assessment Skin biopsy Correct Answer: D Learning objective : Recognize the signs and symptoms of celiac disease and initiate appropriate evaluation.

Rationale: The incidence of celiac disease in individuals with type 1 diabetes mellitus is 5% to 7%. Affected patients may report typical symptoms of celiac sprue (bloating, diarrhea, abdominal pain, weight loss) or be relatively asymptomatic. Sometimes a chronic inflammatory rash (dermatitis herpetiformis) develops suddenly, lasts for weeks to months, and may also be associated with other gastrointestinal diseases. The best initial test is measurement of autoantibodies (Answer D). Tissue transglutaminase and/or deaminated gliadin antibodies are associated with celiac disease and are most indicative of disease when both are positive. Colonoscopy (Answer A) would not be diagnostic; an upper gastrointestinal series with small-bowel follow-through (Answer B) would probably show nonspecific small-bowel findings but would also not be diagnostic. TPO antibody measurement (Answer C) is irrelevant to this case. A skin biopsy (Answer E) should diagnose the rash as dermatitis herpetiformis, but it would not identify the specific associated gastrointestinal condition—in this case, celiac disease.

Reference(s): Barker JM. Clinical review: type 1 diabetes-associated autoimmunity: natural history, genetic associations, and screening. J Clin Endocrinol Metab. 2006;91(4):1210-1217. PMID: 16403820Vermeersch P, Geboes K, Marien G, Hoffman I, Hiele M, Bossuyt X. Serological diagnosis of celiac disease: comparative analysis of different strategies. Clin Chim Acta. 2012;413(21-22):1761-1767. PMID: 22771970Acerini CL, Ahmed ML, Ross KM, Sullivan PB, Bird G, Dunger DB. Coeliac disease in children and adolescents with IDDM: clinical characteristics and response to gluten-free diet. Diabetic Med . 1998;15(1):38-44. PMID: 9472862

ITE 2017 Question 33 A 39-year-old obese man is referred after a fingerstick blood glucose measurement at a health screening fair at work was documented to be 178 mg/ dL (9.9 mmol/L). He had recently eaten lunch. His medical history is notable for dyslipidemia that is well controlled on simvastatin, gout, and obesity. On physical examination, his blood pressure is 132/78 mm Hg and his BMI is 41.5 kg/m2. Acanthosis nigricans is present, but there are no other notable findings on physical examination. You reassess his glycemic status: Fasting plasma glucose (laboratory) = 119 mg/ dL (70-99 mg/ dL ) (SI: 6.6 mmol /L [3.9-5.5 mmol /L]) Hemoglobin A1c = 6.6% (4.0%-5.6%) (SI: 49 mmol / mol [20-38 mmol / mol])Repeated sampling 1 week later:Fasting plasma glucose = 107 mg/dL (5.9 mmol/L)Hemoglobin A1c = 6.8% (51 mmol/mol)

Which of the following diagnoses does this patient have? Normal glucose tolerance Prediabetes Impaired glucose tolerance Impaired fasting glucose Type 2 diabetes mellitus Correct Answer: E Learning objective : Define and diagnose type 2 diabetes mellitus on the basis of American Diabetes Association criteria.

Rationale: The diagnosis of type 2 diabetes has classically been made on the basis of a fasting plasma glucose value of 126 mg/ dL or greater (≥7.0 mmol /L). Mainly used in research settings, an oral glucose tolerance test with a 2-hour cutoff for plasma glucose of 200 mg/dL or greater (≥11.1 mmol /L) can also be used for diagnostic purposes. In 2010, the American Diabetes Association approved hemoglobin A1c measurement as an additional test for diabetes diagnosis, with a threshold of 6.5% or greater (≥48 mmol / mol ). According to these guidelines, the diagnosis is confirmed on the basis of 2 positive tests: either a repeat of the originally abnormal test on a separate day or 2 different tests on the same or separate day. For example, if the hemoglobin A1c level is 6.7% (50 mmol / mol ) and then 6.9% (52 mmol / mol ) on confirmatory testing, irrespective of the fasting plasma glucose, diabetes can be diagnosed. The same would apply if 2 fasting plasma glucose measurements are high ( eg, 128 and 132 mg/ dL [7.1 and 7.3 mmol /L]), regardless of the hemoglobin A1c value. Finally, a high glucose value ( eg , 129 mg/dL [7.2 mmol/L]) and high hemoglobin A1c value (eg, 6.6% [49 mmol/mol]) would also be confirmatory.

Rationale Continued: This patient has type 2 diabetes on the basis of 2 separate hemoglobin A1c measurements that are greater than 6.5% (thus, Answer E is correct). Although his impaired fasting glucose value (119 mg/ dL [6.6 mmol /L]) is somewhat discordant with his hemoglobin A1c levels, according to the American Diabetes Association guidelines, the diagnosis defaults to the confirmed tests. A normal fasting glucose level is less than 100 mg/dL (<5.6 mmol /L) (thus, Answer A is incorrect). Values between 100 and 125 mg/ dL (5.6-6.9 mmol /L) define impaired fasting glucose, the correlate of impaired glucose tolerance by oral glucose tolerance testing (2-hour plasma glucose of 140-199 mg/ dL [7.8-11.0 mmol /L]). Both impaired fasting glucose and impaired glucose tolerance are frequently referred to as prediabetes, underscoring their ability to identify a person at significantly increased risk of worsening hyperglycemia over time to the point of developing type 2 diabetes. This transition is typically characterized by progressive β-cell dysfunction. This patient has already progressed to type 2 diabetes (thus, Answers B, C, and D are incorrect).

Reference(s): International Expert Committee. International Expert Committee report on the role of the A1C assay in the diagnosis of diabetes. Diabetes Care . 2009;32(7):1327-1334. PMID: 19502545 American Diabetes Association. (2) Classification and diagnosis of diabetes. Diabetes Care. 2016;39(Suppl):S13-S22. PMID: 26696675

ITE 2017 Question 39 A 69-year-old man with interstitial pulmonary fibrosis is admitted to the hospital with respiratory failure and possible bacterial pneumonia requiring endotracheal intubation and admission to the intensive care unit. Medical history includes chronic obstructive pulmonary disease, right-sided heart failure, hypertension, and hypothyroidism. There is no history of diabetes and no record of glucose-lowering medication use in his chart. At presentation, his random blood glucose level is documented to be 183 mg/ dL (10.2 mmol /L). After treatment initiation with methylprednisolone, his blood glucose climbs to 302 mg/dL (16.8 mmol /L).

Which of the following is the best approach to manage this patient’s hyperglycemia? Human regular insulin subcutaneously every 4 hours, adjusted to maintain blood glucose greater than 180 mg/ dL (10.0 mmol /L)Intravenous human regular insulin infusion titrated to achieve blood glucose between 140 and 180 mg/dL (7.8-10.0 mmol /L) Intravenous human regular insulin infusion titrated to achieve blood glucose between 80 and 110 mg/ dL (4.4-6.1 mmol /L) Daily insulin glargine plus insulin aspart subcutaneously every 6 hours, adjusted to a target blood glucose between 140 and 180 mg/ dL (7.8-10.0 mmol/L) Subcutaneous sliding scale insulin to determine his insulin requirements Correct Answer: B Learning objective : Manage blood glucose in acutely ill hospitalized patients.

Rationale: This critically ill patient has severe hyperglycemia. The 2009 consensus statement from the American Association of Clinical Endocrinologists (AACE) and the 2016 American Diabetes Association (ADA) guidelines advise that hyperglycemic patients in the intensive care unit should receive intravenous insulin to control their glucose and that the glucose levels should be maintained between 140 and 180 mg/ dL (7.8-10.0 mmol /L) (Answer B). This was based on the NICE-SUGAR study (Normoglycemia in Intensive Care Evaluation and Surviving Using Glucose Algorithm Regulation), which demonstrated no benefit from more stringent blood glucose control to less than 110 mg/ dL (<6.1 mmol /L). Regular insulin by intermittent subcutaneous injection (Answers A and D) has little role in the intensive care unit, since blood glucose control can be achieved more quickly and more reliably with intravenous administration. Sliding scale insulin alone (Answer E) would also be inappropriate in a patient who is clearly in need of ongoing insulin for sustained glycemic control in this setting. Achieving a lower target of 80 to 110 mg/ dL (4.4-6.1 mmol /L) with intravenous insulin (Answer C) is now thought to yield no additional benefit but, instead, it markedly increases the risk of severe hypoglycemia (more than 6-fold). A basal and prandial insulin regimen, such as insulin glargine and insulin aspart , is appropriate upon transfer out of the intensive care unit, but, as with regular insulin, it is not rapid enough in its action and is not as easily adaptable as an intravenous infusion.

Reference(s): Moghissi ES, Korytkowski MT, DiNardo M, et al; American Association of Clinical Endocrinologists; American Diabetes Association. American Association of Clinical Endocrinologists and American Diabetes Association consensus statement on inpatient glycemic control. Diabetes Care. 2009;32(6):1119-1131. PMID: 19429873American Diabetes Association. 13. Diabetes care in the hospital. Diabetes Care. 2016;39(Suppl 1):S99-S104. PMID: 26696689NICE-SUGAR Study Investigators, Finfer S, Chittock DR, et al. Intensive versus conventional glucose control in critically ill patients. N Eng J Med. 2009;360(13):1283-1297. PMID: 19318384

ITE 2017 Question 42 A 38-year-old woman is referred for evaluation of hypoglycemia. The patient has had overwhelming fatigue associated with tachycardia, sweating, and tremor that occur throughout the day. These episodes have no clear association with food intake, but they seem to be precipitated by exertion. Thus, she is admitted for a 72-hour supervised fast. This proceeds uneventfully until the 60th hour when, at 1:00 AM, a reflectance meter glucose concentration obtained by fingerstick is 45 mg/ dL (2.5 mmol/L). The patient is subsequently awoken from sleep and asked about accompanying symptoms. She reports the presence of her usual symptoms. End-of-fast procedures are commenced. Laboratory values are shown: Glucose = 51 mg/ dL (70-99 mg/ dL (SI: 2.8 mmol /L [3.9-5.5 mmol /L]) Insulin = <0.1 µIU/mL (1.4-14.0 µIU/mL) (SI: <0.7 pmol /L [9.7-97.2 pmol/L]) Proinsulin = 26.5 pg /mL (26.5-176.4 pg /mL) (SI: 3.0 pmol/L [3.0-20.0 pmol/L])C-peptide = <0.1 ng/mL (0.9-4.3 ng/mL) (SI: <0.03 nmol/L [0.30-1.42 nmol/L])β-Hydroxybutyrate = 77.0 mg/dL (<3.0 mg/dL) (SI: 7400 µmol/L [<300 µmol/L])

Administration of glucagon results in the following glycemic response: Measurement Time From End of Fast, min 0 10 20 30 Glucose 52 mg/dL (2.9 mmol/L) 57 mg/ dL (3.2 mmol /L) 55 mg/ dL (3.1 mmol /L) 54 mg/ dL (3.0 mmol /L)

You conclude that the data: Cannot be interpreted because the glucose level at the time the fast was ended was not low enough; the fast must be repeated Are compatible with non–insulin-mediated hypoglycemia; an ACTH stimulation test is indicated Are compatible with a normal fast; the Whipple triad has not been fulfilled Are compatible with an insulinlike factor mediating hypoglycemia; IGF-2 measurement is indicated Are compatible with insulin-mediated hypoglycemia; the sample tubes should be assessed for hemolysis, which would explain the low concentrations of β-cell polypeptides Correct Answer: C Learning objective : Rely on fulfilling the Whipple triad to anchor a hypoglycemia diagnosis.

Rationale: The diagnosis of a hypoglycemic disorder rests on the demonstration of the Whipple triad: symptoms that are compatible with neuroglycopenia that occur at the time of a low venous glucose measurement and are corrected when the hypoglycemia is ameliorated. These criteria are not fulfilled in this instance because the fast was stopped not by the development of neuroglycopenia , but by a glucose value below 55 mg/dL (<3.1 mmol /L). The patient’s symptoms developed when she was awoken; her symptoms were reported to be typical of those she experiences frequently, but they are not typical for neuroglycopenia (fatigue is not the main symptom). The laboratory data are compatible with normal prolonged fasting where insulin secretion is suppressed and ketones serve as the primary substrate for metabolism (therefore, Answer C is correct). Subsequently, the patient had her blood drawn for a glucose measurement at the time of onset of her typical symptoms. The glucose concentration was 75 mg/ dL , a value that does not explain her symptomatology and confirms that she does not have hypoglycemia or a hypoglycemic disorder. β-Cell function is suppressed when glucose values fall below the range of 60 to 65 mg/ dL (3.3-3.6 mmol /L). It is usually best to interpret detectable concentrations of β-cell polypeptides when the glucose concentration is below the range of 55 to 60 mg/ dL (3.1-3.3 mmol /L), and it is important to remember that some of these polypeptides have prolonged half-lives in the circulation (proinsulin, C-peptide). The data from this patient’s fast are interpretable given the glucose value at the end of the fast (therefore, Answer A is incorrect).

Rationale Continued: The data would be compatible with non–insulin-mediated hypoglycemia if the patient had clear evidence of neuroglycopenia . In such circumstances, easily waking up a patient who was sleeping is unlikely—the patient in this vignette did not have symptoms of hypoglycemia that awoke her from sleep and she was responsive when awake. It is important to continually document mental status during the fast using a simple method adapted to the patient’s baseline; for example, subtraction of serial 7s if the patient is able to do this without error at baseline. In this vignette, the patient’s symptoms were not compatible with neuroglycopenia (therefore, Answer B is incorrect).If an insulinlike factor were causing hypoglycemia, one would expect β- hydroxybutyrate to be suppressed, and enough hepatic glycogen reserves, which, when mobilized in response to glucagon, generate a greater than 25 mg/ dL (1.4 mmol /L) rise in glucose concentrations. This would strongly suggest the presence of a circulating insulinlike hormone such as IGF-2, which is often a mediator of tumor-associated hypoglycemia. However, these criteria are not fulfilled in this vignette (therefore, Answer D is incorrect). Hemolysis might explain low insulin concentrations, but it would not explain the absence of other characteristics compatible with an insulin-mediated process (therefore, Answer E is incorrect).

Reference(s): Cryer PE, Axelrod L, Grossman AB, et al; Endocrine Society. Evaluation and management of adult hypoglycemic disorders: an Endocrine Society Clinical Practice Guideline. J Clin Endocrinol Metab. 2009;94(3):709-728. PMID: 19088155Service FJ. Diagnostic approach to adults with hypoglycemic disorders. Endocrinol Metab Clin North Am. 1999;28(3):519-532. PMID: 10500929Service FJ, Natt N, Thompson GB, et al. Noninsulinoma pancreatogenous hypoglycemia: a novel syndrome of hyperinsulinemic hypoglycemia in adults independent of mutations in Kir6.2 and SUR1 genes. J Clin Endocrinol Metab . 1999;84(5):1582-1589. PMID: 10323384 Service GJ, Thompson GB, Service FJ, Andrews JC, Collazo-Clavell ML, Lloyd R. Hyperinsulinemic hypoglycemia with nesidioblastosis after gastric-bypass surgery. N Engl J Med . 2005;353(3):249-254. PMID: 16034010

ITE 2017 Question 47 A 66-year-old woman presents as a new patient to your practice because of her concerns after seeing an optometrist. She has a 14-year history of type 2 diabetes mellitus. She has had poor follow-up with providers in recent years, but has managed to continue her medications (metformin, glyburide, and lovastatin) with variable adherence to her treatment regimen. She has not experienced any notable vision-related symptoms. She shows you the retinal photograph from her recent optometry examination. Laboratory test results: Hemoglobin A1c = 8.0% (4.0%-5.6%) (64 mmol / mol [20-38 mmol / mol ])

Given the photograph in the context of her presentation, which of the following should you recommend? Intensification of therapy to reduce her hemoglobin A1c level to less than 7.0% (<53 mmol / mol ) and routine referral to an ophthalmologistImmediate referral to an ophthalmologist for probable focal laser/intravitreal antivascular endothelial growth factor therapy Addition of pioglitazone and recommendation that the patient follow-up with an ophthalmologist in 6 to 12 months Expedited referral to an ophthalmologist for panretinal photocoagulation Addition of lisinopril and basal insulin and routine referral to an ophthalmologist Correct Answer: B Learning objective : Triage high-risk retinal problems that require urgent referral to an ophthalmologist in patients with diabetes mellitus.

Rationale: Diabetic retinopathy continues to be a major complication of diabetes mellitus. Although endocrinologists are not expected to definitively diagnose retinopathy, they must be aware of the types of retinopathy and recognize the need for urgent referral, the impact of medical therapies, and the prognosis to guide patients. Many patients now have retinal photographs taken with routine eye examination. This can offer pertinent information, although the quality of such photos is sometimes an issue. In this case, the patient has evidence of clinically significant macular edema that is near the fovea, and she has a high risk of vision loss. Focal argon-laser photocoagulation can reduce vision loss from macular edema by 50%. It does not usually improve the vision directly. Intravitreal antivascular endothelial growth factor is an alternative therapy. In this patient, with lesions close to the central vision, immediate evaluation by an ophthalmologist for expedited therapy (Answer B) is important. Patients should be referred quickly whenever new clinically significant macular edema is recognized.

Rationale Continued: Improving glycemic control (Answer A) in this patient would not be an adequate response without expedited referral to an ophthalmologist. There is some concern that aggressive insulinization could worsen macular edema, although this is less likely to happen with the relatively low hemoglobin A1c value.

Rationale Continued: Adding pioglitazone and recommending that the patient follow-up with an ophthalmologist in 6 to 12 months (Answer C) is also incorrect, again because of the delay in referral to an ophthalmologist. Additionally, thiazolidinediones such as pioglitazone have been associated with worsening macular edema that improves when the medication is stopped. Panretinal photocoagulation (Answer D) is intended for patients experiencing proliferative diabetic retinopathy. Thus, this answer is incorrect because macular edema requires local photocoagulation. Although some evidence exists that ACE inhibitors (such as lisinopril mentioned in Answer E) and angiotensin receptor blockers retard the progression of macular edema, this answer does not mention immediate referral to an ophthalmologist.

Reference(s): Mohamed Q, Gillies MC, Wong TY. Management of diabetic retinopathy: a systematic review. JAMA . 2007;298(8):902-916. PMID: 17712074Simó R, Hernández C. Advances in the medical treatment of diabetic retinopathy. Diabetes Care. 2009;32(8):1556-1562. PMID: 19638526

ITE 2017 Question 49 A 74-year-old man is referred for management of hypoglycemia after hyperinsulinemic hypoglycemia is documented. The following laboratory test results were obtained from samples drawn while the patient was experiencing mild cognitive dysfunction: Glucose = 39 mg/ dL (70-99 mg/dL ) (SI: 2.2 mmol /L [3.9-5.5 mmol /L]) C-peptide = 3.3 ng/mL (0.9-4.3 ng/mL) (SI: 1.09 nmol /L [0.30-1.42 nmol /L]) Insulin = 10 µIU/mL (1.4-14.0 µIU/mL) (SI: 69.5 pmol /L [9.7-97.2 pmol/L])Sulfonylurea screen (including repaglinide and glimepiride), negative He has no notable medical history and has had no abdominal operations. When you see him, he has a normal chemistry panel (including serum calcium) and a normal complete blood cell count. Findings from triphasic CT of the abdomen and transabdominal ultrasonography are normal. You then order a selective arterial calcium stimulation test to regionalize the presumed insulinoma . The following results are obtained:

  Insulin, µIU/mL Gastroduodenal Artery a Superior Mesenteric Artery a Splenic Artery a Baseline 8.1 9.8 10.1 20 s 10.5 15.7 16.5 40 s 28.5 30.7 14.3 60 s 37.0 45.415.0a Insulin values are measured in blood sampled from the hepatic vein after intra-arterial injection of calcium into the respective arteries/arterial territories supplying the pancreas.

Which of the following should be your next step? Postpone further evaluation until multiple endocrine neoplasia type 1 has been excluded Perform endoscopic ultrasonography and ask the operator to focus on the uncinate process Exclude adrenal insufficiency as the cause of hypoglycemiaAsk a surgeon to explore the pancreas with a view to extended distal pancreatectomy Confirm with the radiologist that because of aberrant pancreatic arterial anatomy, you cannot interpret the selective calcium arterial stimulation test results Correct Answer: B Learning objective : Interpret results of selective calcium arterial stimulation testing.

Rationale: Noninvasive imaging is unhelpful in approximately 25% of all insulinoma cases, and invasive tests such as endoscopic ultrasonography or selective arterial calcium stimulation testing are often required. Endoscopic ultrasonography, while less invasive than selective arterial calcium stimulation testing, has the disadvantage of often identifying incidental pancreatic abnormalities that may or may not represent an insulinoma . In practice, the 2 tests are used interchangeably or in tandem. In this particular example, selective arterial calcium stimulation testing was performed first. The criterion for a positive response in a given arterial territory is a doubling or tripling of baseline insulin in response to calcium injection. This is not observed in the splenic artery territory, which, assuming normal anatomy, would perfuse the tail. Therefore, asking the surgeon to explore the pancreas for consideration of extended distal pancreatectomy (Answer D) is incorrect given the flat response in this territory. Ensuring that the anatomy of the pancreas arterial supply is not aberrant is important, so that a positive result is correctly attributed to the right anatomic location on the basis of blood supply. However, 75% of the time, the gastroduodenal artery supplies the head of the pancreas, while the splenic artery supplies most of the tail and the superior mesenteric artery supplies the body. The uncinate process is in the watershed territory of the gastroduodenal artery and superior mesenteric artery, and an insulinoma located there easily explains the patient’s results. This was subsequently proven by endoscopic ultrasonography, which incidentally enabled a delineation of pancreatic duct anatomy in relation to the tumor (thus, Answer B is correct and Answer E is incorrect).

Rationale Continued: Although multiple endocrine neoplasia type 1 (Answer A) can present with an insulinoma or, more likely, multiple insulinomas , this disorder is unlikely in a normocalcemic 74-year-old patient. Adrenal insufficiency (Answer C) can present with hypoglycemia; however, it is usually not insulin mediated.

Reference(s): Placzkowski KA, Vella A, Thompson GB, et al. Secular trends in the presentation and management of functioning insulinoma at the Mayo Clinic, 1987-2007. J Clin Endocrinol Metab. 2009;94(4):1069-1073. PMID: 19141587

ITE 2017 Question 51 A 37-year-old woman with a 20-year history of type 1 diabetes mellitus returns to clinic for a routine appointment. Because of weight gain and modest deterioration in glycemic control, the patient has recently joined a gym and she requests guidance in safely increasing her physical activity. She has not required hospitalization subsequent to her diabetes diagnosis. She has had no problems with chest pain or dyspnea during her usual activities, which include walking around her neighborhood. Her diabetes is managed with an insulin pump, and although she has several episodes of hypoglycemia per week, her ability to sense hypoglycemia is preserved and no hypoglycemic episodes have been severe. Her diabetes is complicated by microalbuminuria, peripheral neuropathy without history of ulceration, and closely monitored nonproliferative retinopathy that has not required ophthalmologic intervention. She has no history of orthostatic hypotension or gastroparesis. She has no history of hypertension, dyslipidemia, or cigarette smoking and no family history of premature coronary disease. On physical examination, she is normotensive and her heart rate is regular. Findings on funduscopic examination are unchanged from those documented at her last visit to your clinic. She has impaired ability to sense a monofilament in a few locations on both feet, but has no foot lesions, foot deformity, or evidence of impaired peripheral circulation. Her examination findings are otherwise unremarkable.

Laboratory test results: Hemoglobin A1c = 8.2% (4.0%-5.6%) (66 mmol / mol [20-38 mmol/mol])Creatinine = 0.7 mg/ dL (0.6-1.1 mg/ dL ) (61.9 μmol /L [53.0-97.2 μmol /L]) Glomerular filtration rate (estimated) = 94 mL/min per 1.73 m2 (>60 mL/min per 1.73 m2) Urine albumin-to-creatinine ratio = 52 mg/g creat (<30 mg/g)

As the best management plan regarding physical activity, you recommend that the patient do which of the following next? Begin a regimen of resistance exercises and aerobic activity Perform only non–weight-bearing exercises Avoid resistance exercises Undergo a graded exercise stress test Begin use of a continuous glucose-monitoring device Correct Answer: A Learning objective : Plan for safely increasing physical activity in patients with complicated diabetes mellitus.

Rationale: This patient with type 1 diabetes mellitus is interested in safely increasing her physical activity. Unless clearly contraindicated, patients with type 1 diabetes should be encouraged to be consistently physically active. Regular exercise increases cardiovascular fitness and muscle strength and is useful in both glycemic and weight management. Before initiation of an exercise program, a review of both glycemic control and stability, as well as known diabetes-related complications, should be performed. Conditions that may result in a contraindication to particular types of exercise include uncontrolled hypertension, severe autonomic neuropathy, unstable proliferative retinopathy, severe peripheral neuropathy, or a history of foot lesions. Although she does have intermittent hypoglycemia, it has not been severe, and her ability to detect hypoglycemia is not compromised. As she does not have any contraindications to exercise, she may begin an exercise regimen that includes resistance exercises and aerobic activity now (Answer A). Although past recommendations have suggested that patients with peripheral neuropathy should avoid weight-bearing exercise, recent evidence and guidelines indicate that exercise such as moderate- intensity walking does not increase the risk of foot ulceration in such patients (thus, Answer B is incorrect). This patient may increase her weight-bearing activity, but she should use proper footwear and inspect her feet daily, and the activity should be suspended if injury or ulceration occurs. Her closely monitored nonproliferative retinopathy is not a contraindication to resistance exercise (thus, Answer C is incorrect). However, this type of activity should be avoided in patients with proliferative or severe nonproliferative retinopathy, as it may precipitate vitreous hemorrhage or retinal detachment.

Rationale Continued: Screening for cardiovascular disease in asymptomatic patients with diabetes who are planning exercise is not generally indicated, thus performance of a graded exercise stress test before increasing physical activity (Answer D) is incorrect. This testing may be reasonable in patients with established, advanced complications such as autonomic neuropathy, end-stage renal disease, or severe retinopathy because such patients may be at considerably higher risk for cardiovascular complications. This patient will most likely need to modify insulin delivery via her insulin pump during and after more intensive physical activity. She should increase her frequency of blood glucose measurement and consider reducing delivery of basal and perhaps bolus insulin to correspond with her increased physical activity to minimize her risk of hypoglycemia. However, as this patient has preserved ability to sense hypoglycemia and has had no severe hypoglycemic episodes, use of a glucose sensor before initiating an exercise regimen (Answer E) is not required.

Reference(s): American Diabetes Association. Standards of medical care in diabetes--2016. Diabetes Care . 2016;39( Suppl 1):S23-S35. PMID: 26696676de Ferranti SD, de Boer IH, Fonseca V, Fox CS, Golden SH, Lavie CJ, et al. Type 1 diabetes mellitus and cardiovascular disease: a scientific statement from the American Heart Association and American Diabetes Association. Diabetes Care. 2014;37(10):2843-2863. PMID: 25114297

ITE 2017 Question 57 A 30-year-old woman presents for preconception planning. She has had type 1 diabetes mellitus since age 13 years and she also has hypertension. Her recent hemoglobin A1c values have ranged from 6.7% to 7.8% (50-62 mmol / mol ) (reference range, 4.0%-5.6% [20-38 mmol/mol]). She currently has 2 to 3 episodes of hypoglycemia per week. She had an uncomplicated pregnancy at age 25 years, and the baby was born full term and healthy. Mild background retinopathy was documented on a dilated eye examination 8 months ago. The patient administers 17 units of insulin glargine at bedtime. She uses carbohydrate counting to estimate the premeal dose of insulin lispro . She takes lisinopril , 10 mg daily, and labetalol, 100 mg twice daily. She does not smoke cigarettes, and she has 1 to 2 alcoholic beverages per week. She exercises regularly. On physical examination, her height is 68 in (172.7 cm) and weight is 138 lb (62.7 kg) (BMI = 21 kg/m2). Her blood pressure is 138/85 mm Hg, and pulse rate is 70 beats/min. Funduscopic examination reveals no microaneurysms on nondilated eye exam. The rest of the examination findings are normal. Laboratory test results: Hemoglobin A1c = 6.6% (4.0%-5.6%) (49 mmol / mol [20-38 mmol/mol])Urine albumin-to-creatinine ratio = 42 mg/g creat (<30 mg/g creat)

Which of the following do you recommend before the patient becomes pregnant? Intensify insulin therapy Initiate insulin pump therapy Perform a dilated eye examination Substitute nifedipine for Lisinopril Substitute nifedipine for labetalol Correct Answer: D Learning objective : Counsel women with diabetes mellitus regarding medications that are contraindicated during pregnancy.

Rationale: This patient has hypertension and would like to become pregnant. Hypertension in pregnancy is a major cause of morbidity and mortality in the mother, fetus, and neonate. Treatment of severe hypertension (systolic blood pressure ≥160 mm Hg and diastolic blood pressure ≥110 mm Hg) during pregnancy has maternal and neonatal benefits. However, the benefits of treating mild hypertension (systolic blood pressure 140-150 mm Hg and diastolic blood pressure 90-100 mm Hg) and moderate hypertension (systolic blood pressure 150-159 mm Hg and diastolic blood pressure 100-109 mm Hg) during pregnancy are not nearly as clear-cut. Most antihypertensive agents used in pregnancy are category C; that is, human studies are limited and animal studies have demonstrated fetal risk or are lacking definitive data. Category C drugs can be used if the potential benefits outweigh the risks to the fetus. Methyldopa is a first-line agent in the treatment of hypertension in pregnancy, as it has no long-term adverse effects following in utero exposure. Nonselective β-adrenergic blockers have been reported to lead to intrauterine growth retardation in some studies, but they are generally thought to be safe for use in pregnancy. Labetalol is a nonselective β- and α-blocker, and it is often preferred over methyldopa because of fewer adverse effects. The American College of Obstetricians and Gynecologists Task Force on Hypertension in Pregnancy recommends use of labetalol, nifedipine , or methyldopa as first-line agents to treat pregnant women with hypertension. Labetalol does not need to be stopped (thus, Answer E is incorrect).

Rationale Continued: ACE inhibitors and angiotensin receptor blockers are absolutely contraindicated for treatment of hypertension during the second and third trimesters of pregnancy. These drugs increase the risk of renal dysgenesis, oligohydramnios, and anuric renal failure, which can lead to fetal demise. Other complications include calvarial and pulmonary hypoplasia and intrauterine growth retardation. Use of ACE inhibitors in the first trimester leads to a greater incidence of CNS and cardiovascular anomalies. In one study, congenital malformations in women who became pregnant while taking an ACE inhibitor increased by a relative risk of 2.71 (95% confidence interval: 1.72 to 4.27) compared with the risk of women who had no exposure to antihypertensive agents. Direct renin inhibitors such as aliskiren are also contraindicated during pregnancy. In women who are planning pregnancy, ACE inhibitors, angiotensin receptor blockers, and direct renin inhibitors should be stopped and alternative antihypertensive medications should be substituted (thus, Answer D is correct). In many women with mild or moderate hypertension, a period of observation before conception while off all antihypertensive agents is a reasonable strategy. This would not be advisable in this patient because she has borderline elevated blood pressure while taking 2 antihypertensive medications. Statins are pregnancy category X, and these drugs should also be stopped before conception.

Rationale Continued: To minimize the risk for major congenital malformations in the fetus, clinical practice guidelines from the American Diabetes Association recommend that women with preexisting diabetes have a hemoglobin A1c level as close to normal as possible (<6.5% [<48 mmol / mol ]) before conception if this can be safely achieved without frequent or severe hypoglycemia. The Endocrine Society Clinical Practice Guidelines recommend a goal hemoglobin A1c level less than 7.0% (<53 mmol/mol ) and ideally less than 6.5% (<48 mmol / mol ). This patient already has excellent glycemic control. In addition, she has modest hypoglycemia on her current insulin regimen, and tightened glucose control (Answer A) could increase her risk for hypoglycemia. Insulin pump therapy (Answer B) is certainly an option for a woman with diabetes who is contemplating pregnancy, but it is not necessary. Women with diabetes who are planning pregnancy should have a comprehensive dilated eye examination within a year leading up to the pregnancy and during the first trimester of pregnancy. This patient had a dilated eye examination 8 months ago (thus, Answer C is incorrect).

Reference(s): James PA, Oparil S, Carter BL, et al. 2014 evidence-based guideline for the management of high blood pressure in adults: report from the panel members appointed to the Eighth Joint National Committee (JNC 8) [published correction appears in JAMA. 2014;311(17):1809]. JAMA. 2014;311(5):507-520. PMID 24352797American College of Obstetricians and Gynecologists; Task Force on Hypertension in Pregnancy. Hypertension in pregnancy. Report of the American College of Obstetricians and Gynecologists’ Task Force on Hypertension in Pregnancy. Obstet Gynecol. 2013;122(5):1122-1131. PMID: 24150027Podymow T, August P. Update on the use of antihypertensive drugs in pregnancy. Hypertension. 2008;51(4):960-969. PMID: 18259046Cooper WO, Hernandez-Diaz S, Arbogast PG, et al. Major congenital malformations after first-trimester exposure to ACE inhibitors. N Engl J Med . 2006;354(23):2443-2450. PMID: 16760444 Edison RJ, Muenke M. Central nervous system and limb anomalies in case reports of first-trimester statin exposure. N Engl J Med . 2004;350(15):1579-1582. PMID: 15071140 American Diabetes Association. 12. Management of diabetes in pregnancy. Diabetes Care. 2016;39(Suppl 1):S94-S98). PMID: 26696688Blumer I, Hadar E, Hadden DR, et al. Diabetes and pregnancy: an Endocrine Society Clinical Practice Guideline. J Clin Endocrinol Metab. 2013;98(11):4227-4249. PMID: 24194617

ITE 2017 Question 62 A 42-year-old Hispanic woman has questions regarding her risk of developing diabetes. She is overweight (BMI = 28 kg/m2), and her weight has been stable for 10 years since the delivery of her third child. She had a history of gestational diabetes treated with diet and exercise during her last pregnancy. There is a family history of diabetes in approximately one-half of her family members, and none of them were diagnosed in childhood. She has mild hypertension treated with a calcium channel blocker. Laboratory test results: Fasting glucose = 112 mg/ dL (70-99 mg/dL ) (SI: 6.2 mmol /L [3.9-5.5 mmol /L]) Glucose 2 hours after 75-g oral glucose load = 174 mg/ dL (<140 mg/ dL ) (SI: 9.7 mmol /L [<7.8 mmol /L])LDL cholesterol = 153 mg/dL (<100 mg/ dL [optimal] (SI: 3.96 mmol /L [<2.59 mmol /L])HDL cholesterol = 38 mg/dL (>60 mg/dL [optimal]) (SI: 0.98 mmol/L [>1.55 mmol/L])Triglycerides = 153 mg/dL (<150 mg/dL [optimal]) (SI: 1.73 mmol/L [<3.88 mmol/L])

Which of the following components of her presentation contributes most strongly to her personal risk of diabetes? Family history of type 2 diabetes Presence of metabolic syndrome Personal history of gestational diabetes Current glucose values Hispanic ethnicity Correct Answer: D Learning objective : Identify and compare factors that confer risk for type 2 diabetes mellitus.

Rationale: Each of the answer options individually confers risk for diabetes. Therefore, this question requires the ranking of risk associated with each of these factors. The factor that lends the greatest diabetes risk is the patient’s currently elevated fasting glucose (Answer D). Evaluations of diabetes risk in a prediabetic population have consistently identified elevated fasting glucose as the dominant factor associated with progressive worsening of glycemia , even when mutually adjusted for other important risk factors such as gestational diabetes. Her history of gestational diabetes (Answer C) indeed confers risk as well, but even among women with gestational diabetes there is a further gradation of diabetes risk on the basis of current glucose levels. For example, in a quantitative analysis of the PIPOD study (Pioglitazone in Prevention of Diabetes), glucose exposure under fasting and postchallenge conditions together with the measurement of β-cell function were the only significant determinants of progression to diabetes. Family history of type 2 diabetes (Answer A), presence of metabolic syndrome (Answer B), and Hispanic ethnicity (Answer E) confer smaller increases in diabetes risk. History of type 2 diabetes in any family member confers a 2- to 3-fold increased risk over the population average; both parents having type 2 diabetes is associated with a 5- to 6-fold increased risk. Obesity-associated effects on risk independent of glycemia are encompassed in the metabolic syndrome, with a net effect to magnify risk by 3- to 5- fold. Ethnicity importantly modifies diabetes risk, owing to cultural, behavioral, and genetic phenomena, with 1.5- to 3-fold increases in risk across the major ethnicity groups in North America.

Rationale Continued: Notably, in the Diabetes Prevention Program, which enrolled participants on the basis of obesity and glycemic factors, there was no difference in rates of progression by race or ethnicity of the participants, clarifying that metabolic factors dominate the risk. Genetic determinants of risk, either as single-gene variants or aggregated into genetic risk scores, have been identified using a number of at-risk populations worldwide. However, these features do not materially improve the risk prediction that is possible using the above clinical and epidemiologic factors. Aggregates of these and other epidemiologic risk factors are arguably the best tools for diabetes risk assessment, but even in the setting of combined risk scores, current glucose values dominate risk determination.

Reference(s): Edelstein SL, Knowler WC, Bain RP, et al. Predictors of progression from impaired glucose tolerance to NIDDM: an analysis of six prospective studies. Diabetes . 1997;46(4):701-710. PMID: 9075814Aroda VR, Christophi CA, Edelstein SL, et al; Diabetes Prevention Program Research Group. The effect of lifestyle intervention and metformin on preventing or delaying diabetes among women with and without gestational diabetes: the Diabetes Prevention Program outcomes study 10-year follow-up. J Clin Endocrinol Metab. 2015;100(4):1646-1653. PMID: 25706240Cosson E, Chiheb S, Hamo-Tchatchouang E, et al. Use of clinical scores to detect dysglycaemia in overweight or obese women. Diabetes Metab . 2012;38(3):217-224. PMID: 22300975 Martin E, Ruf E, Landgraf R, Hauner H, Weinauer F, Martin S. FINDRISK questionnaire combined with HbA1c testing as a potential screening strategy for undiagnosed diabetes in a healthy population. Horm Metab Res. 2011;43(11):782-787. PMID: 22009373Tuomilehto J, Lindström J, Hellmich M, et al. Development and validation of a risk-score model for subjects with impaired glucose tolerance for the assessment of the risk of type 2 diabetes mellitus-The STOP-NIDDM risk-score. Diabetes Res Clin Pract. 2010;87(2):267-274. PMID: 20022651

Reference(s): Buijsse B, Simmons RK, Griffin SJ, Schulze MB. Risk assessment tools for identifying individuals at risk of developing type 2 diabetes. Epidemiol Rev . 2011;33:46-62. PMID: 21622851Xiang AH, Peters RK, Kjos SL, et al. Effect of pioglitazone on pancreatic beta-cell function and diabetes risk in Hispanic women with prior gestational diabetes. Diabetes. 2006;55(2):517-522. PMID: 16443789

ITE 2017 Question 71 A 34-year-old woman with a 15-year history of type 1 diabetes mellitus remains hospitalized after a cholecystectomy and intravenous insulin infusion is being used for glycemic control. She has had continued need for narcotics to address postoperative shoulder pain. Yesterday she tolerated a clear-liquid diet, and she ate less than 50% of the solid food on the meal tray that she received this morning. She is expected to be discharged from the hospital within the next 1 to 2 days. At home, she manages her diabetes with an insulin pump. Her admission hemoglobin A1c value is 6.9% (52 mmol / mol ). Her total insulin requirements when using the pump are approximately 23 units per day, with her basal insulin comprising 10.7 units per day. Her bolus insulin calculator settings include: Insulin-to-carbohydrate ratio: 1:14 g of carbohydrate Sensitivity ratio: 1:86 Target glucose: 110 mg/ dL (6.1 mmol /L) Her intravenous insulin infusion at 0.5 to 0.6 units/h has maintained her overnight blood glucose between 110 and 160 mg/ dL (6.1-8.9 mmol /L).

On physical examination, the patient is somnolent but arousable , and she is briefly disoriented after awakening. She has only mild tenderness on palpation of the abdomen, but the rest of her physical examination findings are normal.

Which of the following is the best next step in this patient’s care? Continue intravenous insulin infusion until the patient is consistently eating more than 50% of each meal Transition to 10 units basal insulin analogue once daily and a supplemental scale of rapid-acting analogue insulin given at mealtime if premeal blood glucose is greater than 110 mg/ dL (>6.1 mmol/L)Transition to 10 units basal insulin analogue once daily and rapid-acting analogue insulin given immediately after meals (2 units if <50% of meal eaten; 4 units if >50% of meal eaten) Transition to insulin pump therapy at her usual basal rate and bolus calculator settings Transition to insulin pump therapy, but decrease all basal rates by 20% and reduce the carbohydrate factor to 1 unit per 16 g of carbohydrates eaten at meals

Correct Answer: C Learning objective : Identify acceptable conditions for insulin pump use in the hospital.

Rationale: This hospitalized patient successfully manages her type 1 diabetes with an insulin pump in the outpatient setting. She is recovering from her surgery, has begun to eat solid food, and will most likely be discharged from the hospital in the near future. Thus, it is quite reasonable to plan a corresponding change in her insulin regimen. However, the choice of transition from intravenous insulin infusion to a subcutaneous insulin regimen consisting of scheduled basal and rapid-acting analogue insulins (Answer C) is the only appropriate listed option. The suggested dose of basal insulin listed is close to the amount of basal insulin used in the outpatient setting, and it is approximately 80% of her recent basal insulin requirements in the hospital. Patients who have begun eating should receive prandial insulin coverage, thus continuation of management via the intravenous insulin infusion alone until eating more at meals (Answer A) is not correct and might unnecessarily prolong her hospital stay. Use of a subcutaneous regimen that provides supplemental rather than scheduled insulin with meals (Answer B) is also incorrect as it will not adequately address the patient’s prandial coverage requirements. As outlined in Answer C, patients who are eating somewhat inconsistently may have their prandial insulin given immediately after meals at doses appropriate for their food intake and proportional to their basal insulin requirements.

Rationale Continued: Although this patient’s home insulin use via the pump appears to correspond to her current insulin requirements, she continues to receive narcotics for pain management and is somnolent and disoriented on physical examination. Patients with an altered state of consciousness are not candidates for diabetes self-management, thus neither resumption of insulin pump therapy at her usual settings (Answer D) nor pump use at reduced basal and bolus delivery settings (Answer E) would be appropriate now. If a patient is not able to independently manage an insulin pump, some institutions will permit a family member or significant other to assist with pump management. However, this individual must be trained in pump use and must stay with the patient at all times during the hospitalization to operate the device. Patients on established insulin pump therapy may be appropriate candidates for pump use in the hospital provided certain conditions are met. Clear policies and procedures should be established at institutions that permit use of this technology in the inpatient setting. Candidates for inpatient self-management via insulin pump must be physically and mentally able to monitor their blood glucose levels and administer insulin safely via the device, and they should also be proficient in carbohydrate counting if this method is used in the calculation of mealtime insulin doses. The presence of the insulin pump and orders corresponding to blood glucose testing, pump usage, and insulin delivery should be clearly outlined in the medical record. Nursing personnel should document blood glucose results, basal rates, and bolus insulin amounts administered at least daily, and this information should be recorded in a format that is accessible to all caregivers. In addition, hospital personnel with expertise in the management of insulin pump therapy must be available at all times.

Reference(s): American Diabetes Association. Standards of medical care in diabetes--2013. Diabetes Care. 2013;36( Suppl 1):S11-S66. PMID: 23264422 Umpierrez GE, Hellman R, Korytkowski MT, et al; Endocrine Society. Management of hyperglycemia in hospitalized patients in non-critical care setting: an Endocrine Society clinical practice guideline. J Clin Endocrinol Metab. 2012;97(1):16-38. PMID: 22223765Cook CB, Boyle ME, Cisar NS, et al. Use of continuous subcutaneous insulin infusion (insulin pump) therapy in the hospital setting: proposed guidelines and outcome measures [published correction appears in Diabetes Educ. 2006;32:130]. Diabetes Educ. 2005;31(6):849-857. PMID: 16288092

ITE 2017 Question 75 A 52-year-old man is admitted to the hospital for treatment of urosepsis . Antibiotics and intravenous fluids are started. On admission, his laboratory evaluation was normal except for positive urine cultures. Blood cultures are negative. He has an 8-year history of type 2 diabetes mellitus. He has been taking maximum dosages of metformin and glipizide. Canagliflozin was started 3 months ago when his hemoglobin A1c level was 8.7% (77 mmol/mol ) (reference range, 4.0%-5.6% [20-38 mmol / mol ]). Additional medications include lisinopril , metoprolol, furosemide, and simvastatin. He takes controlled-release oxycodone for chronic back pain and lorazepam for anxiety. On physical examination, the patient is somnolent but arousable . He can answer questions after prompting and is oriented. His height is 70 in (177.8 cm), and weight is 224 lb (101.8 kg) (BMI = 32.1 kg/m2). His blood pressure is 142/92 mm Hg, pulse rate is 94 beats/min.

During the first 3 days of the hospitalization, metformin, glipizide, and canagliflozin are continued at his home dosages. A carbohydrate-controlled diet is initiated, but he has been eating only about half of the food provided. Since admission, his glucose values have ranged from 98 to 201 mg/ dL (5.4-11.2 mmol/L). He has not required any supplemental insulin. On the third hospital day, the patient’s condition deteriorates and he is transferred to a large teaching hospital and additional laboratory tests are ordered.Arterial blood gases: pH = 7.13 (7.35-7.45) Po2 = 107 mm Hg (80-100 mm Hg) (SI: 14.2 kPa [10.6-13.3 kPa ]) Pco2 = 17 mm Hg (35-45 mm Hg) (SI: 2.3 kPa [4.7-6.0 kPa ]) Bicarbonate = 6 mEq /L (21-28 mEq/L) (SI: 6 mmol /L [21-28 mmol /L]) Lactic acid = 21.0 mg/ dL (5.4-20.7 mg/dL) (SI: 2.3 mmol/L [0.6 to 2.3 mmol/L])Hemoglobin A1c = 7.3% (4.0%-5.6%) (56 mmol/mol [20-38 mmol/mol])Random glucose = 152 mg/dL (8.4 mmol/L)

You discontinue his oral antihyperglycemic agents and start intravenous fluids. Which of the following should be started as the best next step? Intravenous insulin at 0.1 units/kg per h with intravenous glucose Intravenous insulin at 0.1 units/kg per h with sodium bicarbonate infusion Basal insulin, 20 units once dailyBasal insulin, 20 units once daily, plus insulin lispro , 5 units with each meal Insulin lispro , 1 unit per 50 mg/ dL for glucose values above 150 mg/ dL (>8.3 mmol /L) Correct Answer: A Learning objective : Diagnose and treat euglycemic diabetic ketoacidosis due to sodium-glucose cotransporter 2 inhibitors.

Rationale: The patient in this vignette was treated with oral antihyperglycemic medications during his initial hospitalization. Many hospitals restrict or prohibit use of oral antihyperglycemic agents because of the risk of acute renal insufficiency and other conditions that can lead to metabolic derangements, including metabolic acidosis. There is a higher risk of hypoglycemia with use of oral agents in the hospital because of skipped or missed meals and worsening renal function. Use of oral hypoglycemic agents should therefore be avoided during hospitalization, and many institutions have policies that dictate that all hospitalized patients with hyperglycemia be treated with insulin.“ Euglycemic ketoacidosis” is a rare subtype of ketoacidosis in which metabolic acidosis and elevated ketones are present but the glucose levels are not markedly elevated. Typically, the glucose levels are in the range of 100 to 250 mg/ dL (5.6-13.9 mmol /L). Euglycemic ketoacidosis can occur in the setting of decreased caloric intake, alcohol use, and infections. Sodium-glucose transporter 2 (SGLT-2) inhibitors are a new class of antihyperglycemic medications available to treat diabetes. SGLT-2 inhibitors inhibit glucose reabsorption in the proximal convoluted tubule of the kidney leading to glucosuria and secondary improvement in insulin sensitivity and insulin release and are effective antidiabetes agents. Recently, several case reports of euglycemic ketoacidosis associated with use of SGLT-2 inhibitors were published.

Rationale Continued: The largest case series includes 13 episodes of euglycemic ketoacidosis that occurred in 9 patients. Seven of these patients had type 1 diabetes and 2 had type 2 diabetes. Each patient had evidence of high anion gap metabolic acidosis, elevated ketones, and glucose readings in the range of 96 to 233 mg/ dL (5.3-12.9 mmol/L). Ketoacidosis was not recognized by the patients or the providers because of the lack of markedly elevated glucose levels. Precipitating factors in this series included reduced or omitted insulin doses, caloric restriction, surgery, and alcohol use. In this case series, all patients were treated with canagliflozin , but this is most likely a class effect. Even though this patient’s lactic acid level was mildly elevated, it does not explain the low pH in this case. Lactic acidosis can occur in rare circumstances in hospitalized patients treated with metformin. The minimal elevation in the lactic acid level is most likely related to the urosepsis . Euglycemic ketoacidosis should be treated as a typical case of diabetic ketoacidosis with intravenous fluids and continuous intravenous insulin infusion. Most diabetic ketoacidosis protocols involve the use of insulin infusions in the range of 0.1 to 0.14 units/kg per h. However, some protocols advocate an initial bolus of intravenous insulin before starting infusion. Low-dose insulin infusion (0.1 unit/kg per h) has been used to treat diabetic ketoacidosis for several decades and has been shown to effectively lower glucose levels and to correct the metabolic acidosis in a safe, gradual manner (thus, Answer A is correct).

Rationale Continued: Use of a bolus infusion of insulin is not required in this case due to the mildly elevated glucose levels and it could cause hypoglycemia. Due to the relatively low glucose levels, dextrose will need to be immediately added to the intravenous fluids once the insulin infusion is started to avoid hypoglycemia. Some clinicians use a sodium bicarbonate infusion in the treatment of diabetic ketoacidosis when the pH is less than 6.9 or 7.0. However, the use of bicarbonate in this setting is controversial. A randomized trial of 21 adult patients admitted for treatment of severe diabetic ketoacidosis found that the use of bicarbonate infusion did not affect the time to recovery for either hyperglycemia or metabolic acidosis. Use of an insulin infusion with sodium bicarbonate (Answer B) is therefore not indicated in this case. Use of subcutaneous basal insulin alone (Answer C) or combined with a fast-acting insulin analogue (Answer D) is not an appropriate method to treat metabolic acidosis. There may be inadequate tissue perfusion with use of subcutaneous insulin, which can delay correction of the metabolic acidosis. Similarly, use of insulin lispro on an as-needed basis (Answer E) will not correct the metabolic acidosis and could potentially worsen it.

Reference(s): Kitabchi AE, Umpierrez GE, Miles JM, Fisher JN. Hyperglycemic crises in adult patients with diabetes. Diabetes Care . 2009;32(7):1335-1343. PMID: 19564476Salpeter S, Greyber E, Pasternak G, Salpeter E. Risk of fatal and nonfatal lactic acidosis with metformin use in type 2 diabetes mellitus. Cochrane Database Syst Rev. 2006;(1):CD002967. PMID: 16437448Munro JF, Campbell IW, McCuish AC, Duncan LJ. Euglycemic diabetic ketoacidosis. Br Med J. 1973;2(5866):578-580. PMID: 4197963Peters AL, Buschur EO, Buse JB, Cohan P, Diner JC, Hirsch IB. Euglycemic diabetic ketoacidosis: a potential complication of treatment with sodium-glucose cotransporter 2 inhibition. Diabetes Care . 2015;38(9):1687-1693. PMID: 26078479 Roach P, Skierczynski P. Euglycemic diabetic ketoacidosis in a patient with type 2 diabetes after treatment with empagliflozin. Diabetes Care. 2016;39(1):e3. PMID: 26519331Morris LR, Murphy MB, Kitabchi AE. Bicarbonate therapy in severe diabetic ketoacidosis. Ann Intern Med. 1986;105(6):836-840. PMID: 3096181

ITE 2017 Question 79 A 62-year-old man with a 10-year history of type 2 diabetes mellitus presents for cardiovascular evaluation. He has a history of cardiovascular disease, with a myocardial infarction that occurred at age 58 years. He also has a family history of heart disease. His current medications are lisinopril , 20 mg daily; metformin, 1000 mg daily; insulin lispro , 4 units before each meal; and insulin glargine, 20 units in the morning. He quit smoking 5 years ago after a 20 pack-year history. On physical examination, his seated blood pressure is 140/90 mm Hg and BMI is 30 kg/m2.Recent laboratory test results: Hemoglobin A1c = 6.8% (4.0%-5.6%) (SI: 51 mmol / mol [20-38 mmol / mol ]) Fasting plasma glucose = 94 mg/ dL (70-99 mg/ dL ) (SI: 5.2 mmol/L [3.9-5.5 mmol /L]) Total cholesterol = 189 mg/ dL (<200 mg/ dL [optimal]) (SI: 4.90 mmol/L [<5.18 mmol/L])Triglycerides = 120 mg/dL (<150 mg/dL [optimal]) (SI: 1.36 mmol/L [<3.88 mmol/L])LDL cholesterol = 135 mg/dL (<100 mg/dL [optimal]) (SI: 3.50 mmol/L [<0.59 mmol/L])HDL cholesterol = 40 mg/dL (>60 mg/dL [optimal]) (SI: 1.04 mmol/L [>1.55 mmol/L])

Which of the following is the best treatment to address his lipid profile? Pravastatin, 40 mg daily Rosuvastatin, 20 mg daily Lovastatin, 40 mg daily Simvastatin, 20 mg daily Atorvastatin, 10 mg daily Correct Answer: B Learning objective : Recommend appropriate statin intensity dosing in patients with type 2 diabetes mellitus.

Rationale: Current recommendations for statin treatment have been revised such that treatment initiation and the initial statin dosage are personalized on the basis of risk profile, rather than LDL-cholesterol levels. In patients with type 2 diabetes who are 40 years or older, moderate-intensity statin treatment, if clinically indicated, is recommended in addition to lifestyle counseling and behavioral modification. However, for patients with a high risk for cardiovascular disease (defined as an LDL-cholesterol level of 100 mg/ dL or greater [≥2.6 mmol /L], high blood pressure, history of cigarette smoking, overweight/obesity, or a history of cardiovascular disease), high-intensity statin therapy is advised. Clinical trials have shown that individuals at high risk for cardiovascular disease have a significant reduction in further cardiovascular events with an aggressive regimen of high-intensity statin therapy. Currently, limited clinical trial evidence is available for statin therapy for persons older than 75 years or younger than 40 years. The only high-intensity statin therapy listed in the answer options is rosuvastatin , 20 mg daily (Answer B). Answers A, C, D, and E are all moderate-intensity statin therapy options

Reference(s): Stone NJ, Robinson JG, Lichtenstein AH, et al. 2013 ACC/AHA guideline on the treatment of blood cholesterol to reduce atherosclerotic cardiovascular risk in adults: a report of the American College of Cardiology/American Heart Association Task Force on Practice Guidelines [published corrections appear in J Am Coll Cardiol. 2014;63(25 Pt B):3024-3025 and J Am Coll Cardiol. 2015;66(24):2812]. J Am Coll Cardiol. 2014;63(25 Pt B):2889-2934. PMID: 24239923American Diabetes Association. Standards of medical care in diabetes--2016. Diabetes Care. 2016;39(Suppl 1):S60-S71.

ITE 2017 Question 84 A 65-year-old man is being followed up for type 2 diabetes mellitus of several years’ duration. He is treated with a combination of metformin, 1000 mg twice daily, and glimepiride, 4 mg daily. He has background retinopathy and microalbuminuria. Other than a BMI of 32.5 kg/m2, the rest of his examination findings are normal. Laboratory test results: Creatinine = 1.3 mg/ dL (0.7-1.3 mg/dL) (SI: 114.9 µ mol /L [61.9-114.9 µ mol /L]) Fasting blood glucose = 175 mg/ dL (70-99 mg/ dL ) (SI: 9.7 [3.9-5.5 mmol /L]) Hemoglobin A1c = 8.0% (4.0%-5.6%) (64 mmol /mol [20-38 mmol / mol ]) Your discussion about alternative therapies turns to the use of exenatide in combination with his existing medications.

In this patient, exenatide therapy will most likely: Unmask existing gastroparesis, resulting in nausea and vomiting Not cause hypoglycemia in combination with the patient’s current therapy Produce clinically significant and sustained weight loss by increasing energy expenditureLower hemoglobin A1c by a combination of effects on insulin and glucagon secretion, as well as caloric intake Carry a higher risk of pancreatitis than other therapies Correct Answer: D Learning objective : Assess the potential effects of adding exenatide to the treatment regimen of patients with diabetes mellitus.

Rationale: Exenatide is a glucagonlike peptide 1 receptor agonist used to treat diabetes mellitus. When added to metformin and/or a sulfonylurea, it lowers hemoglobin A1c by an average of 0.7% to 0.9%. This is achieved, in part, by glucose-dependent stimulation of insulin secretion and suppression of glucagon secretion (Answer D). In addition, probably through a direct central mechanism, it decreases caloric intake, which, together with a vagally -mediated delay in gastric emptying, leads to some weight loss, but not by increasing energy expenditure (thus, Answer C is incorrect). Clinical use of exenatide has not been associated with unmasking of previously asymptomatic gastroparesis (thus, Answer A is incorrect).Although the metformin- exenatide combination does not cause hypoglycemic symptoms, there is an excess of hypoglycemic symptomatology when exenatide is combined with a sulfonylurea (thus, Answer B is incorrect). Glucagonlike peptide 1 decreases exocrine pancreatic secretion. However, initial reports of pancreatitis in patients treated with exenatide suggested that the drug was causative. Data from subsequent examination of large prescriber databases (Kaiser, United Health) suggest that the incidence of pancreatitis in patients with diabetes (which is higher than in nondiabetic persons) is unchanged and may even be decreased by exenatide use (thus, Answer E is incorrect).

Reference(s): Delgado- Aros S, Vella A, Camilleri M, et al. Effects of glucagon-like peptide-1 and feeding on gastric volumes in diabetes mellitus with cardio-vagal dysfunction. Neurogastroenterol Motil. 2003;15(4):435-443. PMID: 12846732DeFronzo RA, Ratner RE, Han J, Kim DD, Fineman MS, Baron AD. Effects of exenatide (exendin-4) on glycemic control and weight over 30 weeks in metformin-treated patients with type 2 diabetes. Diabetes Care. 2005;28(5):1092-1100. PMID: 15855572Kendall DM, Riddle MC, Rosenstock J, et al. Effects of exenatide (exendin-4) on glycemic control over 30 weeks in patients with type 2 diabetes treated with metformin and a sulfonylurea. Diabetes Care. 2005;28(5):1083-1091. PMID: 15855571 Dore DD, Bloomgren DL, Wenten M, et al. A cohort study of acute pancreatitis in relation to exenatide use. Diabetes Obes Metab . 2011;13(6):559-566. PMID: 21320263 Li L, Shen J, Bala MM, et al. Incretin treatment and risk of pancreatitis in patients with type 2 diabetes mellitus: systematic review and meta-analysis of randomised and non-randomised studies. BMJ. 2014;348:g2366. PMID: 24736555

ITE 2017 Question 86 A 72-year-old man undergoes abdominal ultrasonography during the evaluation of newly discovered liver enzyme elevations. The study was normal except for the finding of an echogenic liver consistent with hepatic steatosis. He has a history of type 2 diabetes mellitus, obstructive sleep apnea, and hypertension and hyperlipidemia for which pravastatin was recently started. His review of systems is otherwise normal. There is no history of intravenous drug use, body piercing, or tattoos. He drinks 2 to 3 glasses of alcohol per week. Medications include glipizide, insulin glargine, pravastatin, lisinopril , and hydrochlorothiazide. On physical examination, he is alert and fully oriented. His height is 70 in (177.8 cm) and weight is 243 lb (110.5 kg) (BMI = 34.9 kg/m2). There is no jaundice. His abdomen is soft, nondistended , and nontender. His liver and spleen are nonpalpable. There is no fluid wave. He has no clubbing or edema in the extremities. There is no palmar erythema.

Laboratory test results: Hemoglobin A1c = 8.8% (4.0%-5.6%) (65 mmol / mol [20-38 mmol/mol])LDL cholesterol = 68 mg/ dL (<100 mg/ dL [optimal]) (SI: 1.76 mmol /L [<0.59 mmol /L]) HDL cholesterol = 33 mg/ dL (>60 mg/ dL [optimal]) (SI: 0.85 mmol/L [>1.55 mmol/L]) Triglycerides = 257 mg/ dL (<150 mg/ dL [optimal]) (SI: 2.90 mmol/L [<3.88 mmol/L])AST = 104 U/L (20-48 U/L) (SI: 1.7 µkat/L [0.33-0.80 µkat/L])ALT = 127 U/L (10-40 U/L) (SI: 2.1 µkat/L [0.17-0.67 µkat/L])Alkaline phosphatase = 182 U/L (50-120 U/L) (SI: 3.0 µkat/L [0.84-2.00 µkat/L])Total bilirubin = 0.6 mg/dL (0.3-1.2 mg/dL) (SI: 10.3 µmol/L [5.1-20.5 µmol/L])Albumin = 3.9 g/dL (3.5-5.0 g/dL) (SI: 39 g/L [35-50 g/L])Creatinine = 1.3 mg/dL (0.7-1.3 mg/dL) (SI: 114.9 µmol/L [61.9-114.9 µmol/L])INR = 0.92 (0.8-1.2)Hepatitis B and C serologies, negativeLiver biopsy shows changes of hepatitis with steatosis, hepatocyte swelling, and mild acute lobular and chronic portal inflammation.

Which of the following interventions would you next advise in the management of this patient’s liver abnormalities? Weight loss Intensification of insulin therapy to improve hemoglobin A1c Discontinuation of pravastatin Initiation of fenofibrate Elimination of alcohol intake Correct Answer: A Learning objective : Manage nonalcoholic fatty liver disease in patients with type 2 diabetes mellitus.

Rationale: Liver function abnormalities are common in patients with type 2 diabetes mellitus, found in 2% to 24% of patients screened, and they are most commonly due to fatty liver disease and chronic hepatitis. Patients with hepatic steatosis are characterized as having nonalcoholic fatty liver disease (NAFLD) when there is an absence of heavy alcohol consumption, defined as more than 210 g of alcohol (about 15 drinks) per week in men and more than 140 g (about 10 drinks) per week in women. Fatty infiltration of the liver can lead to inflammation, hepatic cell death, and cirrhosis, and it increases the risk of hepatocellular carcinoma. The prevalence of NAFLD is nearly 100% in obese patients with diabetes. Patients with diabetes who are older than 45 years who have NAFLD are at increased risk of hepatic fibrosis, and referral to gastroenterology should be considered. Weight loss (Answer A) of 7% or more has been shown to reduce hepatic steatosis, and it is the intervention of choice in patients with diabetes and NAFLD. Among patients with NAFLD in 1 study, bariatric surgery leading to a mean excess weight loss of 60% resulted in resolution of steatohepatitis in 84% of patients and resolution of fibrosis in 75% of patients after 2 years. Small trials have demonstrated benefits in patients with NAFLD who are treated with metformin (improvements in alanine aminotransferase but not histology) and thiazolidinediones (improvements in alanine aminotransferase and histology), but their effects on the progression to more advanced liver disease are unknown. Improvement in glycemic control with other agents including insulin (Answer B) has not been shown to benefit outcomes in patients with diabetes and NAFLD.

Rationale Continued: Patients with NAFLD with and without diabetes have an increased risk of cardiovascular disease, which is the most common cause of death in this population. Management of traditional cardiovascular disease risk factors should therefore be aggressively pursued. Statins have not been shown to cause hepatic steatosis and are safe in patients with NAFLD. Thus, discontinuing pravastatin (Answer C) is incorrect. While fibrates may confer cardiovascular protection in patients with diabetes who have high triglycerides and low HDL cholesterol, they have not been shown to improve liver enzyme abnormalities or steatosis in NAFLD. Thus, initiating fenofibrate (Answer D) is incorrect. Discontinuing the intake of moderate amounts of alcohol (Answer E) is not recommended for this patient, as its consumption has not been shown to be harmful in patients with NAFLD. Several cross-sectional studies suggest a beneficial effect of light alcohol consumption (on average less than 1 drink per day) on the presence (defined either biochemically or by imaging) and severity of NAFLD. There are no studies reporting the effect of ongoing alcohol consumption on disease severity or natural history of NAFLD or nonalcoholic steatohepatitis.

Reference(s): Chalasani N, Younossi Z, Lavine JE, Diehl AM, Brunt EM, Cusi K, et al; American Gastroenterological Association; American Association for the Study of Liver Diseases; American College of Gastroenterology. The diagnosis and management of non-alcoholic fatty liver disease: practice guideline by the American Gastroenterological Association, American Association for the Study of Liver Diseases, and American College of Gastroenterology [published correction appears in Gastroenterology. 2012;143(2):503]. Gastroenterology. 2012;142(7):1592-1609. PMID: 22656328Furuya CK Jr, de Oliveira CP, de Mello ES, et al. Effects of bariatric surgery on nonalcoholic fatty liver disease: preliminary findings after 2 years. J Gastroenterol Hepatol. 2007;22(4):510-514. PMID: 17376042Promrat K, Kleiner DE, Niemeier HM, et al. Randomized controlled trial testing the effects of weight loss on nonalcoholic steatohepatitis. Hepatology . 2010;51(1):121-129. PMID: 19827166 Tolman KG, Fonseca V, Dalpiaz A, Tan MH. Spectrum of liver disease in type 2 diabetes and management of patients with diabetes and liver disease. Diabetes Care . 2007;30(3):734-743. PMID: 17327353 Dunn W, Xu R, Schwimmer JB. Modest wine drinking and decreased prevalence of suspected nonalcoholic fatty liver disease. Hepatology. 2008;47(6):1947-1954. PMID: 18454505

ITE 2017 Question 88 A 23-year-old woman with a 15-year history of type 1 diabetes mellitus presents with a new skin lesion. She reports a nonpainful sore on her anterior left lower extremity that has enlarged over the past 3 months. On physical examination, you observe the lesion (see image).

Which of the following is the most likely diagnosis? Erythema nodosum Scleredema Necrobiosis lipoidica Necrolytic migratory erythema Granuloma annulare Correct Answer: C Learning objective : Identify dermopathies common to diabetes mellitus.

Rationale: Necrobiosis lipoidica (Answer C) is a dermatologic lesion classically seen overlying the anterior shin, characterized by a shallow depression into the dermis that is erythematous, often with a slightly yellow hue, and with telangiectasias . Lesion size varies considerably, ranging between one and several centimeters in diameter, and lesions can grow over time and are frequently bilateral. Pathologically, it is characterized by abnormal collagen degeneration with a granulomatous response and abnormal blood vessel formation. The cause is unknown and evidence for any course of treatment has been limited, but attempted treatments have included nonsteroidal inflammatory agents, cryotherapy, and topical glucocorticoid agents for early lesions and intralesional corticosteroids injected into the borders of established lesions. Systemic glucocorticoid therapy can also be effective but may prove problematic for patients with diabetes given associated hyperglycemia.

Rationale Continued: Erythema nodosum (Answer A) has painful erythematous nodules or plaquelike lesions in a similar distribution, but it does not occur with increased frequency in patients with diabetes. It commonly accompanies systemic infections or sarcoidosis. Scleredema (Answer B) is a rare diffuse thickening of the back, which is sometimes confused with scleroderma (a condition that commonly affects the face and extremities). Scleredema occurs with increased frequency in diabetes. Necrolytic migratory erythema (Answer D) is a rash on the abdomen, pelvic region, buttocks, or upper legs that is nontender and has irregular borders, sometimes associated with scaling or crusting. It is encountered in patients with glucagonoma syndrome, which often leads to hyperglycemia. Finally, granuloma annulare (Answer E), another skin condition associated with diabetes, consists of raised, nontender and nonerythematous papules, sometimes forming rings, that are usually distributed over hands, arms, feet, and legs.

Reference(s): Thiboutot DM. Clinical review 74: Dermatological manifestations of endocrine disorders. J Clin Endocrinol Metab. 1995;80(10):3082-3087. PMID: 7559901Heidenheim M, Jemec GE. Successful treatment of necrobiosis lipoidica diabeticorum with photodynamic therapy. Arch Dermatol. 2006;142(12):1548-1550. PMID: 17178979

FEMALE REPRODUCTION

ITE 2017 Question 7 A 25-year-old woman with polycystic ovary syndrome presents to discuss contraceptive options. Her menarche was at age 11 years, and her menses have always been irregular. She had onset of hirsutism and acne at age 12 years and both have progressed since adolescence. She is currently using barrier contraceptives. Her weight increased during college, and her BMI is now 29 kg/m2. Her blood pressure is 110/70 mm Hg.

Which of the following contraceptive methods would be best for this patient? Levonorgestrel -releasing intrauterine device Oral contraceptive containing drospirenone SpironolactoneMedroxyprogesterone Long-acting GnRH analogue Correct Answer: B Learning objective : Differentiate among contraceptive options for women with polycystic ovary syndrome.

Rationale: Figure. Classification of commonly used progestins . Adapted from Buzney E, Sheu J, Buzney C, Reynolds RV. Polycystic ovary syndrome: a review for dermatologists: Part II. Treatment. J Am Acad Dermatol. 2014;71(5):859.e1-859.e15. Reproduced with permission of Mosby, Inc. in the format reuse in CME materials via Copyright Clearance Center.

Rationale Continued: Contraception in women with polycystic ovary syndrome should be targeted not only to protect against unplanned pregnancy, but also to control the metabolic and hyperandrogenic phenotype. Hormonal contraceptives ( ie , oral contraceptives, patch, or vaginal ring) are recommended as first-line management for menstrual abnormalities and hirsutism and acne of polycystic ovary syndrome, as well as for contraception. Oral contraceptive agents contain a potent synthetic estrogen, ethinyl estradiol, in combination with a progestin. Most of these progestins are derived from testosterone and exhibit mild degrees of androgenicity on laboratory markers. Other progestins , including cyproterone acetate and drospirenone , are structurally unrelated to testosterone and function as androgen receptor antagonists. These agents are preferred to suppress ovarian androgens and to ensure shedding of the endometrial lining (thus, Answer B is correct).

Rationale Continued: An intrauterine device is a consideration, but the levonorgestrel-releasing intrauterine device (Answer A) has an androgenic progestin and may worsen the hyperandrogenism . Due to the teratogenic potential of spironolactone (Answer C), this agent should not be used as monotherapy. Patients taking antiandrogen therapy for treatment of hirsutism associated with polycystic ovary syndrome should be using an adequate method of contraception. Barrier contraception is not considered to be an adequate method due to higher failure rates than that of hormonal contraceptives. Medroxyprogesterone (Answer D) at high dosages is a contraceptive, but it often causes weight gain and would not be indicated in a patient with polycystic ovary syndrome. Due to the high cost and unfavorable adverse effect profile of long-acting GnRH analogues (Answer E), they would not be the preferable method of contraception for this patient.

Reference(s): Buzney E, Sheu J, Buzney C, Reynolds RV. Polycystic ovary syndrome: a review for dermatologists: Part II. Treatment. J Am Acad Dermatol. 2014;71(5):859.e1-859.e15. PMID: 25437978Raymond EG, Cleland K. Clinical practice. Emergency contraception. N Engl J Med. 2015;372(14):1342-1348. PMID: 25830424Brynhildsen J. Combined hormonal contraceptives: prescribing patterns, compliance, and benefits versus risks. Ther Adv Drug Saf. 2014;5(5):201-213. PMID: 25360241Yildiz BO. Oral contraceptives in polycystic ovary syndrome: risk-benefit assessment. Semin Reprod Med . 2008;26(1):111-120. PMID: 18181089 Gomes MP, Deitcher SR. Risk of venous thromboembolic disease associated with hormonal contraceptives and hormone replacement therapy: a clinical review. Arch Intern Med . 2004;164(18):1965-1976. PMID: 15477430 Legro RS, Pauli JG, Kunselman AR, et al. Effects of continuous versus cyclical oral contraception: a randomized controlled trial. J Clin Endocrinol Metab. 2008;93(2):420-429. PMID: 18056769Legro RS, Arslanian SA, Ehrmann DA, et al; Endocrine Society. Diagnosis and treatment of polycystic ovary syndrome: an Endocrine Society Clinical Practice Guideline. J Clin Endocriol Metab. 2013;98(12):4565-4592. PMID: 24151290

Reference(s): Martin KA, Chang RJ, Ehrmann DA, et al. Evaluation and treatment of hirsutism in premenopausal women: an Endocrine Society clinical practice guideline. J Clin Endocrinol Metab. 2008;93(4):1105-1120. PMID: 18252793

ITE 2017 Question 15 A 27-year-old woman with a 7-year history of infertility presents for evaluation. The patient has polycystic ovary syndrome with symptoms of irregular menses and hyperandrogenism . She describes irregular menses every 6 to 8 weeks since menarche at age 12 years. She shaves her face twice weekly. She married 7 years ago. She and her husband have never used contraception. Starting 1 year ago, she was treated with 6 cycles of clomiphene citrate, which included ultrasounds documenting follicle growth and β-hCG injections to ensure ovulation. She did not become pregnant. At the time of her marriage 7 years ago, she weighed 120 lb (54.5 kg), but she has gained more than 70 lb (31.8 kg) in the last 4 years. Her husband, who works out regularly as a body builder, has taken her to the gym and helped her lose 20 lb (9.1 kg) through aerobic conditioning. Since the weight loss, her menstrual cycles occur every 30 days. Her last menstrual period began 20 days prior to her visit with you. On physical examination, her blood pressure is 115/72 mm Hg. Her height is 72 in (182.9 cm), and weight is 173 lb (78.6 kg) (BMI = 23.5 kg/m2). Her skin examination is notable for dark, coarse hair over the chin, maxilla, and lower stomach. She has no supraclavicular fat, galactorrhea, or violaceous striae . Her proximal muscle strength is normal.

Laboratory test results: TSH = 1.3 mIU /L (0.5-5.0 mIU /L)FSH = 2.4 mIU/mL (2.0-12.0 mIU/mL [follicular], 4.0-36.0 mIU /mL [ midcycle ], 1.0-9.0 mIU /mL [luteal]) (SI: 2.4 IU/L [2.0-12.0 IU/L, follicular], [4.0-36.0 IU/L, midcycle ], [1.0-9.0 IU/L, luteal]) Prolactin = 5.2 ng/mL (4-30 ng/mL) (SI: 0.2 nmol /L [0.17-1.30 nmol /L]) Testosterone = 64 ng/dL (8-60 ng/dL ) (SI: 2.2 nmol /L [0.3-2.1 nmol /L])

Which of the following diagnostic tests would be the most likely to determine the cause of infertility in this couple? Transvaginal ultrasonography Progesterone measurement Estradiol measurement Hysterosalpingogram Semen analysis for her husband Correct Answer: E Learning objective : Diagnose the cause of infertility in a couple with both female- and male-factor infertility.

Rationale: In a woman with polycystic ovary syndrome, infertility is caused by anovulation. Some women with polycystic ovary syndrome will become pregnant because they have occasional, irregular ovulatory cycles. However, timing intercourse is difficult. The patient in this vignette was treated with clomiphene citrate to induce ovulation, ovulation was documented, and intercourse was timed appropriately, but she did not become pregnant. Therefore, factors other than anovulation should be considered. In couples in which the woman has polycystic ovary syndrome, up to 10% of the male partners have concomitant male-factor oligospermia and 5% of the women also have bilateral tubal occlusion. In the couple in this vignette, the possibility of male-factor infertility has not been assessed, and her husband’s body-building history is concerning for anabolic steroid abuse. Anabolic steroids suppress FSH and LH, resulting in decreased serum and intratesticular testosterone, which is required for spermatogenesis. A semen analysis demonstrated azoospermia in the patient’s husband and confirmed an additional cause of infertility in this couple (thus, Answer E is correct). Recovery of spermatogenesis can take 6 to 24 months after discontinuation of anabolic steroids.

Rationale Continued: Assessment for tubal pathology with a hysterosalpingogram (Answer D) is indicated in an infertile couple, but it would be less likely to identify an abnormality in a monogamous couple with no history of sexually transmitted disease, as in this vignette. Although endometriosis also causes tubal defects, the patient has no symptoms of pelvic pain with menses, which would be expected with severe endometriosis causing pelvic adhesions and tubal blockade. A progesterone measurement (Answer B) of 2 to 20 ng/mL (6.4-63.6 nmol /L) (drawn 7 days before menses) indicates ovulation. However, the couple did not become pregnant during ovulation induction and cycles are now regular, so further documentation of ovulation is not warranted. Estradiol levels change across the menstrual cycle with growth of the dominant follicle and development of a corpus luteum. Therefore, an isolated measurement (Answer C) would not be informative. Transvaginal ultrasonography (Answer A) can identify ovarian pathology, but the patient had multiple ultrasounds in the course of fertility treatment 1 year ago and infertility predates those examinations. Therefore, ultrasonography is unlikely to identify a cause of infertility.

Reference(s): Nieschlag E, Vorona E. Mechanisms in endocrinology: medical consequences of doping with anabolic androgenic steroids: effects on reproductive functions. Eur J Endocrinol. 2015;173(2):R47-R58. PMID: 25805894Kamel RM. Management of the infertile couple: an evidence-based protocol. Reprod Biol Endocrinol. 2010;8:21. PMID: 20205744Legro RS, Arslanian SA, Ehrmann DA, et al; Endocrine Society. Diagnosis and treatment of polycystic ovary syndrome: an Endocrine Society clinical practice guideline. J Clin Endocrinol Metab . 2013;98(12):4565-4592. PMID: 24151290

ITE 2017 Question 28 A 27-year-old woman presents with a 6-month history of amenorrhea. She has been in excellent health except for a history of hypothyroidism due to Hashimoto thyroiditis. She has been euthyroid while taking levothyroxine, 100 mcg daily. She reports mild acne, vaginal dryness, and night sweats. Physical examination is notable for a small thyroid gland. Initial laboratory test results: TSH = 3.0 mIU /L (0.5-5.0 mIU /L) Estradiol = <20 pg /mL (SI: <73.4 pmol /L) FSH = 93.6 mIU /mL (2.0-12.0 mIU/mL [follicular], 4.0-36.0 mIU/mL [ midcycle ], 1.0-9.0 mIU /mL [luteal]) (SI: 93.6 IU/L [2.0-12.0 IU/L, follicular], [4.0-36.0 IU/L, midcycle ], [1.0-9.0 IU/L, luteal])Prolactin = 6.7 ng/mL (4-30 ng/mL) (SI: 0.29 nmol/L [0.17-1.30 nmol/L])β-hCG, undetectable

Which of the following would be the most appropriate next test? LH measurement Pituitary MRI Antinuclear antibody measurement Ovarian antibody measurement Karyotype analysis Correct Answer: E Learning objective : Diagnose common causes of secondary amenorrhea and discuss the differential diagnosis of premature ovarian insufficiency.

Rationale: This question addresses the differential diagnosis of premature ovarian insufficiency, which is defined as menopause before age 40 years. It can be associated with karyotype abnormalities ( eg , 45,XO or mosaic 45,XO/46,XX [Turner syndrome] or 46,XX/47,XXX), chemotherapy, radiotherapy, an autoimmune process, defects in the FSH receptor or the FSH molecule (presenting as primary amenorrhea), and premutations in the FMR1 gene (the gene associated with fragile X syndrome). The American College of Obstetrics and Gynecology now recommends that all women with premature ovarian insufficiency be screened for mutations in FMR1. However, in a large percentage of patients with premature ovarian insufficiency, the etiology cannot be defined. Because patients with 45,XO/46,XY mosaicism have the potential to develop malignant changes in the gonad, performing a karyotype analysis (Answer E) on all patients younger than age 30 years who present with ovarian failure is recommended. The history of autoimmune thyroid disease does not exclude the possibility of a chromosomal abnormality because hypothyroidism is a common finding in women with Turner syndrome. Diagnosis of Turner syndrome is important, because affected patients should be screened for cardiac abnormalities, hearing loss, diabetes mellitus, and liver enzyme abnormalities.

Rationale Continued: In a patient in whom a chromosomal abnormality is not found, measurement of thyroid and adrenal antibodies may be useful in predicting long-term involvement of other organ systems in the autoimmune process. To date, it has not been determined that measurement of ovarian antibodies (Answer D) is of particular use. Measuring ovarian antibodies is not a reliable or sensitive test for ovarian autoimmunity. Other autoimmune associations include diabetes, pernicious anemia, rheumatoid arthritis, systemic lupus erythematosus, myasthenia gravis, vitiligo, and premature gray hair. Pituitary MRI (Answer B) is not indicated now because none of the patient’s other clinical findings suggest a central process. Measuring serum LH (Answer A) would add little to the clinical picture because the FSH level is in the postmenopausal range. Measuring antinuclear antibodies (Answer C) is a relatively low priority in this setting.

Reference(s): Nelson LM. Clinical practice. Primary ovarian insufficiency. N Engl J Med . 2009;360(6):606-614. PMID: 19196677.Welt CK. Primary ovarian insufficiency: a more accurate term for premature ovarian failure. Clin Endocrinol (Oxf). 2008;68(4):499-509. PMID: 17970776

ITE 2017 Question 44 You are asked to see a 27-year-old, 46,XX individual who identifies as male and carries a diagnosis of gender dysphoria. He has undergone psychotherapy, has lived as a man for the last year and feels comfortable with his decision. He also has a good support network. He now seeks further treatment to assist with virilization . His medical history is unremarkable. He has no family history of cardiovascular disease, breast, or ovarian cancer. He smokes one-half pack of cigarettes per day and drinks 11 alcoholic beverages per week. On physical examination, his blood pressure is 109/72 mm Hg. His height is 66 in (167.6 cm), and weight is 146 lb (66.4 kg) (BMI = 23.6 kg/m2). He has hair on the upper lip and below the umbilicus. Breasts are without masses and findings on gynecologic examination are normal.

Laboratory test results: Liver function, normal Hematocrit = 35% (35%-45%) (SI: 0.35 [0.35-0.45]) Total cholesterol = 195 mg/ dL (<200 mg/dL [optimal]) (SI: 5.05 mmol/L [<5.18 mmol /L]) Triglycerides = 84 mg/ dL (<150 mg/ dL [optimal]) (SI: 0.95 mmol /L [<3.88 mmol /L]) HDL cholesterol = 37 mg/ dL (>60 mg/ dL [optimal]) (SI: 0.96 mmol/L [>1.55 mmol /L]) LDL cholesterol = 104 mg/ dL (<100 mg/ dL [optimal]) (SI: 2.69 mmol/L [<2.59 mmol/L])Hemoglobin A1c = 5.0% (4.0%-5.6%) (31 mmol/mol [20-38 mmol/mol])Estradiol = 75 pg/mL (17-200 pg/mL) (SI: 275.5 pmol/L [62.9-739.6 pmol/L])

Which of the following is the best next step in his care plan? Oophorectomy Testosterone therapy Aromatase inhibitor therapy Testosterone plus aromatase inhibitor therapy GnRH analogue therapy Correct Answer: B Learning objective : Recommend initial medical treatment for a transgender female to male adult who desires therapy.

Rationale: Gender dysphoria occurs when the gender with which a person identifies is different from his or her biologic gender. Gender dysphoria affects 1 in 11,900 genotypic men and 1 in 30,400 genotypic women in the Netherlands, but statistics from other countries are not available or are inaccurate. Most patients note that their gender dysphoria began in childhood. Transgender is an informal diagnosis used to describe individuals whose gender identity is different from their biologic gender. The transgender diagnosis should be made by a mental health professional using the World Professionals Association for Transgender Health standards. Psychotherapy, which is necessary to identify and treat psychosocial and psychiatric issues, should be undertaken for at least 6 months before any treatment is considered and should continue after treatment begins. In addition, the patient should live in the desired gender role for at least 6 months to 1 year before starting medical therapy. Adult transgender female-to-male patients who desire therapy should start with testosterone (Answer B) after appropriate medical evaluation. The evaluation includes a hematocrit measurement, liver function tests, and lipid panel. The testosterone can be given as an injection, gel, or patch, with titration of the dosage to achieve a testosterone level in the physiologic male range. When considering cost, intramuscular testosterone enanthate and cypionate or testosterone pellets are much less expensive ($100-$200 per month) than patches, gels, and buccal preparations ($200-$500 per month).

Rationale Continued: Transgender female-to-male patients should be followed up every 2 to 3 months for the first year of therapy and once to twice yearly thereafter. Testosterone should be maintained in the normal male range, while estradiol should be followed until menses cease or estradiol falls below 50 pg /mL (<184 pmol /L). Complete blood cell counts, liver function testing, weight, blood pressure, lipids, and fasting glucose and/or hemoglobin A1c should be monitored at each visit. Papanicolaou tests should continue if the cervix is present. Mammography should also be undertaken according to American Cancer Society standards if mastectomy has not been performed. Surgical therapy (Answer A) should not be undertaken until after testosterone starts. GnRH analogues (Answer E) can be used in adolescent patients to block breast development and menses, after which testosterone can be added, but it would not be the first choice in an adult. Aromatase inhibitors would decrease estradiol levels, but would not achieve the desired masculinizing effect (thus, Answers C and D are incorrect).

Reference(s): Hembree WC, Cohen- Kettenis P, Delemarre -van de Waal HA, et al; Endocrine Society. Endocrine treatment of transsexual persons: an Endocrine Society clinical practice guideline. J Clin Endocrinol Metab. 2009;94(9):3132-3154. PMID: 19509099Van Kesteren PJ, Gooren LJ, Megens JA. An epidemiological and demographic study of transsexuals in The Netherlands. Arch Sex Behav. 1996;25(6):589-600. PMID: 8931882Spack NP. Management of transgenderism. JAMA. 2013;309(5):478-484. PMID: 23385274

ITE 2017 Question 52 A 66-year-old woman presents with hirsutism. She had regular menses until she went through menopause at age 51 years. Four years ago, she began developing hirsutism on her back, chest, upper abdomen, and face, along with frontal, male-pattern balding. She also notes increased libido. Her medical history is unremarkable. On physical examination, her blood pressure is 130/80 mm Hg. You note frontal and occipital balding and dark hair between the breasts and on the face, upper back, upper and lower abdomen, upper arms, and thighs. She has no dorsal cervical or supraclavicular fat pad. She has clitoromegaly and no palpable ovarian masses. Her height is 64.5 in (163.8 cm), and weight is 149 lb (67.7 kg) (BMI = 25.2 kg/m2). Laboratory test results: Total testosterone = 160 ng/ dL (8-60 ng/ dL ) (SI: 5.6 nmol /L [0.3-2.1 nmol /L]) DHEA-S = 45 µg/ dL (15-157 µg/ dL) (SI: 1.2 µ mol /L [0.41-4.25 µ mol /L]) LH = 25 mIU/mL (>30 mIU/mL) (SI: 25 IU/L [>30 IU/L])FSH = 10.7 mIU/mL (>30 mIU/mL) (SI: 10.7 IU/L [>30 IU/L])Estradiol = <20 pg/mL (<20 pg/mL) (SI: 73.4 pmol/L [<73.4 pmol/L])

Which of the following tests would be most useful now? Adrenal CT Free testosterone measurement Ovarian ultrasonography Oral glucose tolerance test GnRH agonist test Correct Answer: C Learning objective : Construct a differential diagnosis for postmenopausal hyperandrogenism .

Rationale: In a postmenopausal woman with an elevated total testosterone level and normal DHEA-S level, ovarian ultrasonography (Answer C) should be the first diagnostic test. The differential diagnosis includes Leydig -cell or hilus -cell tumor, ovarian carcinoma with functioning stroma, and hyperthecosis. Ultrasonography would detect a Leydig -cell tumor or carcinoma. Hyperthecosis would present as bilateral ovarian enlargement. A hilus -cell tumor would not be identifiable on ultrasonography and might require additional testing such as ovarian venous sampling. This patient’s diagnosis was hyperthecosis . Hyperthecosis occurs when the stroma of the ovary differentiates into luteinized, steroid-producing theca cells. It is not clear what causes the differentiation. These luteinized cells can be found in nests throughout both ovaries and produce high levels of testosterone. In a postmenopausal woman, testosterone is aromatized in adipose tissue, primarily to estrone . The estradiol level was therefore low in this patient. Estrone does suppress LH and FSH levels, which are lower than expected in this postmenopausal woman. In postmenopausal women, bilateral ovariectomy is the treatment of choice for hyperthecosis . Long-term GnRH agonists can be used to suppress the androgens in a nonsurgical patient.

Rationale Continued: Adrenal CT (Answer A) or MRI would not be indicated in the presence of a normal DHEA-S concentration. A free testosterone measurement (Answer B) is unnecessary in the setting of a high total testosterone level. In addition, the free testosterone assay is insensitive. Assays measuring the low levels of total testosterone in women are already inaccurate. Free testosterone assays rely on the separation of the free testosterone fraction and measurement using a total testosterone assay, thus requiring the measurement of even lower testosterone levels. Most women with hyperthecosis have insulin resistance, impaired glucose tolerance, or diabetes mellitus. An oral glucose tolerance test (Answer D) would be worthwhile, but not as the first step in this scenario. A GnRH agonist test (Answer E) can help localize the source of androgens to the ovary and may be useful in some patients for long-term treatment. However, the test is largely confirmatory and would not be performed as the first step.

Reference(s): Alpañés M, González- Casbas JM, Sánchez J, Pián H, Escobar-Morreale HF. Management of postmenopausal virilization. J Clin Endocrinol Metab. 2012;97(8):2584-2588. PMID: 22669303Vollaard ES, van Beek AP, Verburg FA, Roos A, Land JA. Gonadotropin-releasing hormone agonist treatment in postmenopausal women with hyperandrogenism of ovarian origin. J Clin Endocrinol Metab . 2011;96(5):1197-1201. PMID: 21307133 Young RH, Scully RE. Endocrine tumors of the ovary. Curr Top Pathol . 1992;85:113-164. PMID: 1321021 Rosner W, Auchus RJ, Azziz R, Sluss PM, Raff H. Position statement: Utility, limitations, and pitfalls in measuring testosterone: an Endocrine Society position statement. J Clin Endocrinol Metab. 2007;92(2):405-413. PMID: 17090633

ITE 2017 Question 63 A 28-year-old woman presents with premenstrual mood changes that are disrupting her life. Menarche was at age 12 years, and her cycles are regular. She has always had painful periods and has tried over-the-counter medications and nonsteroidal anti-inflammatory agents with modest success. Her mood is worse in the week before her period and during her period. She has no family history of clotting disorders. She is not interested in fertility at this time. She wonders what she can do.

Which of the following is the best recommendation? Tricyclic antidepressant and daily multivitamin Serotonin reuptake inhibitor and a low-dosage oral contraceptive pill Serotonin reuptake inhibitor and vitamin B6 supplementation Serotonin reuptake inhibitor and placement of a levonorgestrel-releasing intrauterine device Vitamin B6 supplementation and a low-dosage oral contraceptive pill Correct Answer: B Learning objective : Recommend treatment options for women with premenstrual dysphoric disorder who desire contraception.

Rationale: Women with premenstrual syndrome experience a wide variety of cyclic and recurrent physical, emotional, behavioral, and cognitive symptoms that start in the luteal phase and diminish and stop after the onset of menses. Major symptoms include affective symptoms such as depression, angry outbursts, irritability, and anxiety and somatic symptoms such as breast pain, bloating and swelling, and headache. Controlled studies confirm that abrogation of ovulation with continuous oral contraceptives may help in mild cases, but addition of a serotonin reuptake inhibitor that targets mood directly is the treatment of choice. Answer B therefore describes the best recommendation for this patient. Some patients may benefit from taking the serotonin reuptake inhibitor only the week before and the week of their menses, but most need it throughout the cycle. Tricyclic antidepressants (Answer A) and bupropion are less effective than serotonin reuptake inhibitors or serotonin-norepinephrine reuptake inhibitors. An oral contraceptive and vitamin B6 supplementation (Answer E), a serotonin reuptake inhibitor and vitamin B6 supplementation (Answer C), or a serotonin reuptake inhibitor and a levonorgestrel -releasing intrauterine device (Answer D) (which has an androgenic progestin that has not been shown to help premenstrual syndrome) are not optimal choices in this patient.

Reference(s): Epperson CN, Steiner M, Hartlage SA, et al. Premenstrual dysphoric disorder: evidence for a new category for DSM-5. Am J Psychiatry . 2012;169(5):465-475. PMID: 22764360Brown J, O’Brien PM, Marjoribanks J, Wyatt K. Selective serotonin reuptake inhibitors for premenstrual syndrome. Cochrane Database Syst Rev. 2009;(2):CD001396. PMID: 19370564Jarvis CI, Lynch AM, Morin AK. Management strategies for premenstrual syndrome/premenstrual dysphoric disorder. Ann Pharmacother. 2008;42(7):967-978. PMID: 18559957Pearlstein T, Steiner M. Premenstrual dysphoric disorder: burden of illness and treatment update. J Psychiatry Neurosci. 2008;33(4):291-301. PMID: 18592027

ITE 2017 Question 87 A 51-year-old healthy woman presents with hot flashes, painful intercourse, and insomnia. She does not smoke cigarettes and has no history of venous thrombosis. There is no family history of breast cancer or colon cancer. Her physical examination findings are normal. Pelvic ultrasonography shows a normal uterus with a thin endometrial stripe and small ovaries without masses. She wishes to discuss the risks of menopausal hormone therapy.

Which of the following would be the most common increased risk for this woman if prescribed postmenopausal combined estrogen-progestin hormone therapy for 5 years? Colon Cancer Dementia Stroke Deep venous thrombosis Breast Cancer Correct Answer: D Learning objective : Identify common increased risks associated with postmenopausal hormone therapy.

Rationale: The incidence of deep venous thrombosis (Answer D) was the most frequent vascular risk in both of the estrogen + progestin and estrogen-only arms of the Women’s Health Initiative (WHI) and in many other studies. It is unclear whether there is a genetic risk profile that could be identified to predict this risk. Of interest, when investigators were told to advise patients hospitalized with immobility to discontinue hormone therapy, the risk of deep venous thrombosis and pulmonary embolism decreased. Risk for colon cancer (Answer A) was decreased, not increased, in the WHI estrogen + progestin arm and neutral in the estrogen-only arm. Substudies that have examined “dementia” (Answer B) indicate that estrogen + progestin increases the risk of cognitive impairment. Although investigators initially suggested this was Alzheimer disease, adjudication indicates that the picture was more consistent with progressive microvascular disease in older women studied 4 years after intervention. The estimated absolute increase in risk was 23 cases/10,000 women treated per year. Both estrogen only and estrogen + progestin increased the risk of stroke (Answer C) in a small subset of patients (8 cases/10,000 women per year). Whether this is related to underlying hypercoagulable risk or to another modifier is unclear. Breast cancer (Answer E) was increased in the estrogen + progestin arm, but not in the estrogen-only arm of the WHI.

Rationale Continued: The WHI study of women aged 50 to 79 years showed that estrogen + progestin hormone therapy was associated with an increased risk of deep venous thrombosis, stroke, and cardiovascular events, but had a protective effect against colon cancer and bone fractures. A reanalysis of the data suggests that in women aged 50 to 59 years, hormone therapy has a cardioprotective effect, whereas it increases adverse events in older women. These data support the role of primary prevention, but not secondary prevention, of cardiac risk with estrogen + progestin. Because of risk-benefit modeling, most authorities do not recommend hormone therapy for asymptomatic women. Symptomatic woman (who often have low endogenous estrogens) may benefit the most from short-term hormone therapy. The absolute risks in the WHI were different for estrogen + progestin continuously than for estrogen therapy alone, suggesting that daily medroxyprogesterone may have contributed to some of the increased risks of cardiac disease and breast cancer in the estrogen + progestin arm. Thus, many suggest cyclic estrogen and progestin therapy for postmenopausal women who have a uterus. The WHI used conjugated estrogen ( Premarin , 625 mcg) with daily medroxyprogesterone (Provera, 5 mg). Many practitioners now use lower dosages of estrogens, often in the form of an estradiol patch or gel to bypass the liver and avoid alterations in clotting proteins, although no controlled studies have investigated the risk vs benefit in a head-to-head trial. Bioidentical hormones have not been tested in any prospective randomized controlled studies and have not undergone testing by the US FDA for bioavailability. Thus, they cannot be recommended.

Reference(s): Rosano G, Vitale C, Spoletini I, Fini M. Cardiovascular health in the menopausal woman: impact of the timing of hormone replacement therapy. Climacteric. 2012;15(4):299-305. PMID: 22424090Henderson VW, Lobo RA. Hormone therapy and the risk of stroke: perspectives 10 years after the Women's Health Initiative trials. Climacteric. 2012;15(3):229-234. PMID: 22612608Gompel A, Santen RJ. Hormone therapy and breast cancer risk 10 years after the WHI. Climacteric. 2012;15(3):241-249. PMID: 22612610Kreatsoulas C, Anand SS. Menopausal hormone therapy for the primary prevention of chronic conditions. U.S. Preventive Services Task Force Recommendation Statement. Pol Arch Med Wewn. 2013;123(3):112-117. PMID: 23396275 Marjoribanks J, Farquhar C, Roberts H, Lethaby A. Long term hormone therapy for perimenopausal and postmenopausal women. Cochrane Database Syst Rev . 2012;7:CD004143. PMID: 22786488

MALE REPRODUCTION

ITE 2017 Question 9 A 17-year-old boy is referred for evaluation of delayed puberty. He is very embarrassed about his appearance in the locker room. He does not shave and has sparse body hair. His father recalls that he himself experienced a “late puberty.” The patient has a normal sense of smell. On physical examination, his height is 66 in (167.6 cm) and BMI is 21 kg/m2. He has no facial or pubic hair and minimal axillary hair. He does not have cleft palate. There is no gynecomastia. He has a normal phallus with no hypospadias, and his testes are small (4 mL bilaterally). Laboratory test results: Total testosterone = 20 ng/ dL (300-1000 ng/ dL ) (SI: 0.7 nmol /L [10.4-34.7 nmol /L]) LH = 0.5 mIU /mL (1.8-15 mIU /mL) (SI: 0.5 IU/L [1.8-15 IU/L]) FSH = 2.5 mIU/mL (1.8-15 mIU /mL) (SI: 2.5 IU/L [1.8-15 IU/L]) Serum IGF-1, normal Free T4, normal TSH, normal Iron studies, normal Cosyntropin stimulation test, normal Sellar imaging is normal.

Which of the following would you recommend? Observe until age 18 years, then consider testosterone therapy Clomiphene, 50 mg daily Testosterone gel, 5 g daily Testosterone gel, 10 g daily Testosterone enanthate, 50 mg monthly Correct Answer: E Learning objective : Consider treatment with testosterone to induce virilization in boys with constitutional delay of puberty.

Rationale: This young man most likely has constitutional delay of puberty (and growth), a common condition in boys. His family history, physical examination findings, and hormonal profile are consistent with this diagnosis. On the basis of his phenotype, the diagnosis of Kallmann syndrome is unlikely. Adrenarche (axillary hair, pubic hair, and oily skin) is often delayed in constitutional delay of puberty, but may be normal in hypogonadotropic hypogonadal syndromes. This patient should be treated with low-dosage testosterone therapy. The usual treatment is injectable testosterone enanthate (or another short-acting testosterone ester), 50 to 100 mg monthly (thus, Answer E is correct and Answer A is incorrect). Few data are available regarding other testosterone formulations for the treatment of constitutional delay of puberty. However, adult dosages of any testosterone formulation should be avoided because these dosages will result in closure of the bony epiphyses and decreased adult height. For boys with constitutional delay of puberty, low-dosage testosterone is initiated and continued until virilization has occurred. At age 17 or 18, a trial off testosterone is attempted to determine whether spontaneous puberty has occurred. A testosterone level greater than 20 ng/ dL (>0.7 nmol /L) suggests that spontaneous puberty will occur within 12 to 18 months. It is not appropriate (or kind) to deny low-dosage testosterone therapy to this boy. A low dosage would allow him to virilize enough to avoid embarrassment and ridicule without causing any health hazards.

Rationale Continued: Testosterone gels at adult dosages of 5 or 10 g daily (Answers C and D) would be excessive, leading to fusion of epiphyses and compromising his ultimate height. Clomiphene (Answer B) is not used for the treatment of constitutional delay of puberty. The therapeutic approach to delayed puberty in boys with hypogonadotropic hypogonadal syndromes is similar to the treatment of constitutional delay of puberty. Often the two conditions cannot be distinguished until after age 18; failure to have spontaneous puberty after age 18 confirms the diagnosis of hypogonadotropic hypogonadism.

Reference(s): Palmert MR, Dunkel L. Clinical practice. Delayed puberty. N Engl J Med. 2012;366(5):443-453. PMID: 22296078Young J. Approach to the male patient with congenital hypogonadotropic hypogonadism. J Clin Endocrinol Metab. 2012;97(3):707-718. PMID: 22392951

ITE 2017 Question 18 A 35-year-old man presents for evaluation of infertility. Secondary hypogonadism was diagnosed 6 years ago after he underwent hypophysectomy for a nonsecreting pituitary macroadenoma. Transdermal testosterone gel was initiated, and he has been doing well on this therapy. Now he desires a third child; he and his wife have been trying for a year without success. Multiple semen analyses have revealed oligospermia . He does not have any other relevant medical history. On physical examination, he is normally virilized . Testicular volume is 20 mL bilaterally. Laboratory test results: Testosterone = 612 ng/ dL (300-900 ng/ dL ) (SI: 21.2 nmol /L [10.4-31.2 nmol /L]) LH = 0.8 mIU /mL (1.0-9.0 mIU /mL) (SI: 0.8 IU/L [1.0-9.0 IU/L]) FSH = 1.9 mIU /mL (1.0-13.0 mIU/mL) (SI: 1.1 IU/L [1.0-13.0 IU/L])

After stopping testosterone therapy, which of the following is the most appropriate initial treatment option for fertility induction in this patient? Aromatase inhibitor Subcutaneous GnRH pump hCG injections hCG and FSH injections Clomiphene citrate Correct Answer: C Learning objective : Recommend the most appropriate regimen for fertility induction in a postpubertal man with secondary hypogonadism.

Rationale: Unlike patients with primary hypogonadism in whom seminiferous tubules are damaged, men with secondary hypogonadism can usually undergo successful fertility induction. Treatment with gonadotropins is effective in patients with secondary hypogonadism irrespective of the site of the disease (hypothalamus or pituitary) as they act directly on the testes. HCG possesses the biologic activity of LH and stimulates the Leydig cells to produce testosterone. As a result, the concentration of intratesticular testosterone increases, which is crucial for spermatogenesis. FSH stimulates spermatogenesis by acting on the Sertoli cells and the seminiferous tubules. Studies have shown that in postpubertal men with secondary hypogonadism, hCG alone is sufficient to stimulate spermatogenesis. This patient's secondary hypogonadism is postpubertal , which suggests that monotherapy with hCG (Answer C) will be adequate. When secondary hypogonadism is caused by diseases of the hypothalamus, treatment with GnRH (Answer B) is an option. GnRH is administered in a pulsatile fashion via an infusion pump delivering a pulse every 90 to 120 minutes, stimulating the secretion of gonadotropins. However, integrity of the pituitary gland is a prerequisite for GnRH therapy, which is not the case in this patient.

Rationale Continued: FSH in combination with hCG (Answer D) is generally required in men who develop prepubertal secondary hypogonadism ( eg , Kallmann syndrome, pituitary lesions), as simultaneous stimulation of seminiferous tubules is also required in these patients. An essential component of the negative feedback loop of the gonadal axis is aromatization of testosterone to estradiol, which is a more potent inhibitor of the axis than testosterone itself. Inhibiting estradiol production by aromatase inhibitors (Answer A) stimulates endogenous production of testosterone. However, intactness of the pituitary gland is a prerequisite for it to be effective. Similarly, clomiphene citrate (Answer E) antagonizes estrogen receptors at the hypothalamus, which stimulates release of gonadotropins, which in turn, stimulates endogenous production of testosterone. However, intactness of the pituitary gland is also a prerequisite for clomiphene to be effective.

Reference(s): Finkel DM, Phillips JL, Snyder PJ. Stimulation of spermatogenesis by gonadotropins in men with hypogonadotropic hypogonadism. N Engl J Med . 1985;313(11):651-655. PMID: 3927163Burris AS, Rodbard HW, Winters SJ, Sherins RJ. Gonadotropin therapy in men with isolated hypogonadotropic hypogonadism: the response to human chorionic gonadotropin is predicted by initial testicular size. J Clin Endocrinol Metab. 1988;66(6):1144-1151. PMID: 3372679Büchter D, Behre HM, Kliesch S, Nieschlag E. Pulsatile GnRH or human chorionic gonadotropin/human menopausal gonadotropin as effective treatment for men with hypogonadotropic hypogonadism: a review of 42 cases. Eur J Endocrinol . 1998;139(3):298-303. PMID: 9758439

ITE 2017 Question 25 A 73-year-old man on testosterone therapy returns to clinic for follow-up. He was first seen 8 months ago with concerns of reduced libido and fatigue. At that visit, his morning total testosterone level was 225 ng/ dL (7.8 nmol /L) and his prostate-specific antigen level was 1.3 ng/mL (1.3 µg/L). His prostate was symmetrically enlarged without nodules.After thorough evaluation, the testosterone patch, 4 mg daily, was prescribed, which he has been taking for 6 months. The patient feels well on testosterone therapy and reports improved sexual function and energy. He has no lower urinary tract symptoms. On physical examination at today’s appointment, his prostate examination is unchanged. Laboratory test results: Testosterone = 451 ng/ dL (300-900 ng/ dL ) (SI: 15.6 nmol /L [10.4-31.2 nmol /L]) Prostate-specific antigen = 3.1 ng/mL (<7.0 ng/mL) (SI: 3.1 µg/L [<7.0 µg/L]) Repeated prostate-specific antigen = 2.9 ng/mL (SI: 2.9 µg/L)

Which of the following is the best next step in this patient’s management? Refer him to a urologist Provide reassurance and schedule a follow-up appointment in 6 months Decrease the dosage of the testosterone patch to 2 mg daily Start a 5 α-reductase inhibitor Change to intramuscular administration with testosterone enanthate Correct Answer: A Learning objective : Monitor serum prostate-specific antigen levels during testosterone therapy.

Rationale: Although this patient's prostate-specific antigen level remains within the reference range, he should be referred for urologic evaluation (Answer A) on the basis of its magnitude of change. Prostate-specific antigen levels are known to fluctuate in an individual and also have considerable test-retest variability; however, an increase of greater than 1.4 ng/mL (1.4 µg/L) (confirmed by a repeated test) over the course of a year in a man on testosterone therapy cannot be attributed to random variation alone. A systematic review of prostate risk during testosterone therapy found that the average increase in prostate-specific antigen after initiation of testosterone therapy is 0.3 ng/mL and 0.44 ng/mL in young and old men, respectively. A cutoff of 1.4 ng/mL has been adopted on the basis of the findings of a clinical trial that evaluated the effectiveness of finasteride vs placebo on lower urinary tract symptoms and prostate volume in men with benign prostatic hyperplasia. In that study, the upper limit of the 90% confidence limit for the change in prostate-specific antigen level in the placebo arm was 1.4 ng/ mL. Hence, on the basis of the findings of the finasteride study and the fact that the average increase in prostate-specific antigen levels on testosterone therapy is less than 1.5 ng/mL (regardless of patient age), the Endocrine Society's clinical practice guidelines recommend that patients with a prostate-specific antigen increase greater than 1.4 ng/mL during testosterone therapy should be referred for urologic consultation. It is important to understand that this increase of 1.4 ng/mL in prostate-specific antigen concentration does not indicate prostate cancer, but only serves as a trigger for further evaluation. Simply providing reassurance and scheduling a follow-up visit without any action (Answer B) is incorrect.

Rationale Continued: This patient has shown clinical improvement on his current testosterone dosage and his on-treatment testosterone concentration is normal. Therefore, a decrease in dosage (Answer C) or substitution with intramuscular testosterone (Answer E) is not indicated. Although he has benign prostatic hyperplasia, he does not have lower urinary tract symptoms. Therefore, treatment with a 5α-reductase inhibitor (Answer D) is not indicated.

Reference(s): Bhasin S, Cunningham GR, Hayes FJ, et al; Task Force, Endocrine Society. Testosterone therapy in men with androgen deficiency syndromes: an Endocrine Society clinical practice guideline. J Clin Endocrinol Metab. 2010;95(6):2536-2559. PMID: 20525905Bhasin S, Singh AB, Mac RP, Carter B, Lee MI, Cunningham GR. Managing the risks of prostate disease during testosterone replacement therapy in older men: recommendations for a standardized monitoring plan. J Androl. 2003;24(3):299-311. PMID: 12721204Gormley GJ, Stoner E, Bruskewitz RC, et al. The effect of finasteride in men with benign prostatic hyperplasia. The Finasteride Study Group. N Engl J Med. 1992;327(17):1185-1191. PMID: 1383816

ITE 2017 Question 37 A 62-year-old man is referred by his primary care physician to discuss management of erectile dysfunction in the setting of normal libido. The patient has a medical history of longstanding hypertension that has been well controlled. In his late 50s, he had a positive nuclear stress test. A subsequent coronary angiogram showed several stenotic lesions, and 2 stents were placed. His current medications include low-dosage aspirin, atorvastatin, lisinopril , metoprolol, and a multivitamin. The patient is married and has fathered 2 children. He plays tennis 3 times weekly without any chest pain. On physical examination, the patient appears comfortable. His height is 71 in (180.3 cm), and weight is 195 lb (88.6 kg) (BMI = 27.2 kg/m2). His blood pressure is 126/82 mm Hg, and pulse rate is 71 beats/min. His physical examination findings are unremarkable, including those from cardiac and genital examinations. He has trace lower-extremity edema.

Laboratory test results (afternoon): Complete blood cell count, normal TSH = 2.6 mIU /L (0.5-5.0 mIU/L)Total testosterone = 290 ng/dL (300-900 ng/dL ) (SI: 10.1 nmol /L [10.4-31.2 nmol /L]) 25-Hydroxyvitamin D = 27 ng/mL (25-80 ng/mL) (SI: 67.4 nmol /L [62.4-199.7 nmol /L])

Which of the following therapies would be best? Testosterone enanthate intramuscular injections every 2 weeks Transdermal testosterone gel applied daily Tadalafil, 10 mg 1 hour before sexual activityAlprostadil urethral suppository as needed for sexual activity Penile implant performed by a urologist Correct Answer: C Learning objective : Counsel a man with borderline testosterone levels and coronary disease regarding safe and effective ways to treat erectile dysfunction.

Rationale: The man in this vignette has multiple risk factors for erectile dysfunction including his age (>50 years), vascular disease, hypertension, and medication adverse effects. His androgen levels are unlikely to be a major contributor to his erectile function. Most men with erectile dysfunction who desire treatment are given a trial of a phosphodiesterase 5 inhibitor (Answer C), as most men respond to this medication class. These medications are considered to be quite safe with temporary adverse effects such as headache, flushing, and backache. There are very rare reports of serious adverse effects such as nonarteritic anterior ischemic optic neuropathy, which may cause blindness due to loss of blood flow to the optic nerve. The major contraindication to phosphodiesterase 5 inhibitors is use of a nitrate because of concerns for hypotension. Compared with phosphodiesterase 5 inhibitors, testosterone supplementation (Answers A and B) often does not improve erectile function in middle-aged and older men who have chronic conditions such as vascular disease, obesity, and diabetes mellitus. Not only is testosterone supplementation less effective than phosphodiesterase 5 inhibitors, the safety profile of testosterone is less certain and somewhat controversial with inconsistent findings among studies. Two important studies found that testosterone therapy was associated with increased cardiovascular events.

Rationale Continued: The Testosterone in Older Men with Mobility Limitations Trial was a randomized controlled trial that was stopped prematurely when men in the testosterone arm showed increased rates of myocardial infarction, sudden death, angioplasty, stroke, edema, hypertension, arrhythmias, electrocardiographic changes, and syncope. A second study involved a large health care database of more than 55,000 testosterone prescriptions and found that men aged 65 years or older who received a testosterone prescription had a 2.19-fold (1.27-3.77) increased rate of myocardial infarction in the 90 days following the prescription, compared with the period before receiving the prescription. The rate was 2.90 fold higher (1.49- 5.62) in men younger than 65 years with a history of heart disease. In a randomized controlled trial of testosterone gel for men 65 years or older with total testosterone concentrations less than 275 ng/ dL (<9.5 nmol /L), cardiovascular adverse events were similar in the testosterone and placebo groups. The alprostadil urethral suppository (Answer D) is often a second-line therapy for erectile dysfunction. While this medication is quite effective, many men would prefer not to insert a pellet into the urethra, which can cause pain and bleeding. Likewise, penile implants (Answer E) are usually reserved for men for whom other less invasive treatments have failed.

Reference(s): Basaria S, Coviello AD, Travison TG, et al. Adverse events associated with testosterone administration. N Engl J Med. 2010;363(2):109-122. PMID: 20592293Finkle WD, Greenland S, Ridgeway GK, et al. Increased risk of non-fatal myocardial infarction following testosterone therapy prescription in men. PLoS One. 2014;9(1):e85805. PMID: 24489673Gianatti EJ, Dupuis P, Hoermann R, Zajac JD, Grossmann M. Effect of testosterone treatment on constitutional and sexual symptoms in men with type 2 diabetes in a randomized, placebo-controlled clinical trial. J Clin Endocrinol Metab . 2014;99(10):3821-3828. PMID: 24978674

ITE 2017 Question 58 A 42-year-old male-to-female transsexual patient is referred to you with concerns regarding her voice. Gender dysphoria was diagnosed 3 years ago after a thorough evaluation by a mental health professional. The patient was subsequently referred to an endocrinologist who started treatment with spironolactone to block androgen action and estrogen therapy to induce feminization. Within 12 to 18 months of hormone therapy, there was satisfactory breast development, softening of the skin, decrease in terminal hair growth, and redistribution of body fat. Although the patient is satisfied with her feminine appearance, she is concerned that her voice remains unchanged despite 3 years of hormone therapy. It is still low-pitched and masculine, often resulting in gender misattributions over the phone, and is an impediment to socializing and a source of embarrassment. She discussed this issue with her local endocrinologist who has referred her to you for further evaluation. She is currently on spironolactone, 200 mg daily, and a transdermal estradiol patch, 200 mcg twice weekly, and she reports no adverse effects. The patient does not have any other notable medical history.

During your intake interview, you note that she does have a masculine voice. On physical examination, her blood pressure is 132/68 mm Hg and pulse rate is 76 beats/min. Her height is 68 in (172.7 cm), and weight is 177 lb (80.5 kg) (BMI = 26.9 kg/m2). There are no terminal hairs on the face, chest, or extremities. Her breast development is normal. The rest of her examination findings are unremarkable. Laboratory test results: Testosterone = 37 ng/ dL (SI: 1.3 nmol/L) Estradiol = 172 pg /mL (SI: 631.4 pmol /L) LH = 0.3 mIU /mL (SI: 0.3 IU/L) FSH = 0.5 mIU /mL (SI: 0.5 IU/L) Potassium = 3.9 mEq /L (3.5-5.0 mEq/L) (SI: 3.9 mmol /L [3.5-5.0 mmol / mol ])

Which of the following is the best next management step? Substitute transdermal estradiol patch with oral ethinyl estradiol, 100 mcg daily Substitute spironolactone with finasteride, 5 mg daily Refer for speech therapySubstitute spironolactone with depo injections of a GnRH agonist Refer for pitch-raising surgery Correct Answer: C Learning objective : Recommend appropriate therapy to address voice adjustment in a male-to-female transsexual patient.

Rationale: Gender dysphoria is a psychiatric diagnosis that describes a strong and persistent cross-gender identification, combined with a persistent discomfort with one’s sex or a sense of inappropriateness in the gender role of that sex that causes clinically significant distress. Transsexualism is a desire to live and be accepted as a member of the gender opposite to that assigned at birth (natal sex). The male-to-female transsexual person (as in this case) refers to a biologic male who identifies as, or desires to be, female. Once the diagnosis of transsexualism is ascertained and the patient has engaged in appropriate counseling, hormone therapy is initiated in patients desiring transition to the opposite gender. The goal of hormone therapy is two-fold: (a) suppression of endogenous hormones of the patient’s birth sex, and (b) induction of secondary sexual characteristics of the newly acquired sex. In male-to-female transsexual patients, treatment is directed towards suppression of testosterone secretion or blocking its action by antagonizing the androgen receptor, which results in reduced terminal hair growth, decreased sebum production, and body composition changes. Concurrently, estrogen treatment is also initiated to induce feminizing effects such as breast development, body fat redistribution, and softening of the skin. Estrogens also decrease testosterone production by inhibiting GnRH secretion (suppressed gonadotropins and testosterone in this case is a result of estrogen therapy).

Rationale Continued: These feminizing effects are generally accomplished by 2 years of hormone therapy. One organ that remains resistant to the feminizing effect of estrogen therapy is the larynx. Hence, the vocal tract remains of male size and the voice retains its male character (known as androphonia ). This incongruence between the voice and the phenotype is a source of concern for many patients as it affects their socializing. These patients should be referred to a specialized phoniatric clinic for speech therapy (Answer C). Speech therapy is aimed at altering the pitch and resonance of the voice and has been successful in many patients. Only if speech therapy is unsuccessful should a patient be referred for pitch-raising surgery, a procedure known as cricothyroid approximation. Therefore, surgical referral at this time (Answer E) is premature. Spironolactone, a competitive inhibitor of the androgen receptor, is considered the first-line treatment in male-to-female transsexual patients with the recommended dosage between 100 and 200 mg daily. Cyproterone acetate (a progestin and an androgen receptor antagonist) is also effective; however, it is not available in the United States. This patient has had an excellent response to the antiandrogenic effects of spironolactone, and its substitution will neither have an effect on the patient’s voice nor will it further improve other antiandrogenic benefits that she has already achieved. Although GnRH agonist therapy (Answer D) effectively suppresses serum testosterone levels into castrate range, its expense prohibits long-term use. Finasteride (Answer B) is a 5α-reductase 2 inhibitor that is used in the treatment of hirsutism in women. However, no data are available regarding its use in male-to-female transsexual patients.

Rationale Continued: The transdermal estradiol patch at a dosage of 100 to 400 mcg twice weekly effectively induces feminization, as seen in this patient. The patch has an added advantage over oral 17β-estradiol preparations in that it does not cause hypertriglyceridemia. The target serum estradiol concentration should be kept under 200 pg / mL. Although ethinyl estradiol (used in contraceptive pills) at a dosage of 50 to 100 mcg daily (Answer A) is effective in the treatment of male-to-female transsexual patients, its use has been associated with an elevated risk of venous thromboembolism and, in some reports, even cardiovascular death. Resistance to activated protein C and a reduction in the serum concentration of protein S have been proposed as the possible mechanisms behind this increased risk. Hence, its use is not recommended in these patients.

Reference(s): Gooren LJ. Clinical practice. Care of transsexual persons. N Engl J Med . 2011;364(13):1251-1257. PMID: 21449788Hembree WC, Cohen-Kettenis P, Delemarre-van de Waal HA, et al; Endocrine Society. Endocrine treatment of transsexual persons: an Endocrine Society clinical practice guideline. J Clin Endocrinol Metab. 2009;94:3132-3154. PMID: 19509099Gooren LJ, Giltay EJ, Bunck MC. Long-term treatment of transsexuals with cross-sex hormones: extensive personal experience. J Clin Endocrinol Metab . 2008;93(1):19-25. PMID: 17986639 Günzburger D. Voice adaptation by transsexuals. Clin Linguist Phon . 1989;3(2):163-172. PMID: 21166618 Spencer LE. Speech characteristics of male-to-female transsexuals: a perceptual and acoustic study. Folia Phoniatr (Basel). 1988;40(1):31-42. PMID: 3384379de Bruin MD, Coerts MJ, Greven AJ. Speech therapy in the management of male-to-female transsexuals. Folia Phoniatr Logop. 2000;52(5):220-227. PMID: 10965175Gross M. Pitch-raising surgery in male-to-female transsexuals. J Voice. 1999;13(2):246-250. PMID: 10442755 

ITE 2017 Question 65 A 52-year-old man comes for evaluation to determine whether his progressive fatigue for the past 18 months relates to a low testosterone level. He also has morning headaches and daytime somnolence. He recently saw his primary care physician who documented his total testosterone level to be 282 ng/ dL (9.8 nmol /L). The patient has a diminished libido. He has hypertension that is treated with a diuretic. He is married with 2 children. On physical examination, he is obese but not cushingoid (BMI = 36.2 kg/m2). His visual fields are normal. Acanthosis nigricans is present in the axillae. His testes are 25 mL bilaterally. Muscle strength is normal. Laboratory test results: Total testosterone (8 AM) = 296 ng/ dL (300-900 ng/ dL ) (SI: 10.3 nmol /L [10.4-31.2 nmol /L]) (measured by mass spectrometry)Fasting glucose = 112 mg/ dL (70-99 mg/ dL ) (SI: 6.2 mmol /L [3.9-5.5 mmol/L])Albumin = 4.5 g/dL (3.5-5.0 g/dL) (SI: 45 g/L [35-50 g/L])Serum cortisol (8 AM) = 16.2 µg/dL (5-25 µg/dL) (SI: 446.9 nmol/L [137.9-689.7 nmol/L])Hematocrit = 41.3% (41%-50%) (SI: 0.413 [0.41-0.51])

Measuring which of the following should be the next diagnostic step in this patient’s evaluation? Iron saturation Gonadotropins Prolactin Free testosterone 24-Hour urinary free cortisol Correct Answer: D Learning objective : Order appropriate diagnostic tests to confirm the diagnosis of androgen deficiency in obese men.

Rationale: Measurement of serum total testosterone by a reliable assay is the best screening test for the diagnosis of male hypogonadism. Documentation of low serum total testosterone levels (measured on at least 2 occasions) accurately identifies androgen deficiency in most patients. However, when alterations in serum sex hormone–binding globulin concentrations are suspected, further diagnostic steps are required. Obesity is one such condition in which circulating serum concentrations of sex hormone–binding globulin are reduced. Insulin resistance is posited to be the mechanism, as insulin suppresses sex hormone–binding globulin production from the liver. Laboratory studies have confirmed that the addition of insulin in the culture medium containing human hepatoma cell line suppresses sex hormone–binding globulin production. Human studies have also confirmed an inverse association between serum insulin and sex hormone–binding globulin levels. The next step in obese men is the measurement of free testosterone. Free testosterone can either be calculated by law of mass action equation (using sex hormone–binding globulin, albumin, and total testosterone [measured by a reliable assay]) or measured directly with the equilibrium dialysis method (the latter is not widely available to clinicians).

Rationale Continued: This patient is obese with signs of insulin resistance, and his sex hormone–binding globulin levels are likely to be low. Furthermore, his libido (a specific symptom of androgen deficiency) is normal and his fatigue and daytime somnolence suggest obstructive sleep apnea. Therefore, measurement of free testosterone (Answer D) is the next step in this patient's evaluation. Indeed, this patient's sex hormone–binding globulin levels were low and his free testosterone level was normal. Polysomnography confirmed sleep apnea, and continuous positive airway pressure improved his symptoms. Measurement of gonadotropins (Answer B) is important in the evaluation of a hypogonadal patient as they indicate whether hypogonadism is primary or secondary. However, they should only be measured after the diagnosis of androgen deficiency is confirmed, which is not the case in this patient. Similarly, measurement of prolactin (Answer C) should be performed only after the diagnosis of androgen deficiency is confirmed. Iron studies (Answer A) are indicated if hemochromatosis is suspected as the cause of hypogonadism. However, the possibility of hemochromatosis should be considered only after the diagnosis of androgen deficiency is confirmed. This patient does not have specific clinical features of Cushing syndrome; however, ruling out cortisol excess (Answer E) as a secondary cause of hypogonadism should only be considered after the diagnosis of androgen deficiency is confirmed

Reference(s): Bhasin S, Cunningham GR, Hayes FJ, et al; Task Force, Endocrine Society. Testosterone therapy in men with androgen deficiency syndromes: an Endocrine Society clinical practice guideline. J Clin Endocrinol Metab. 2010;95(6):2536-2559. PMID: 20525905Basaria S. Male hypogonadism. Lancet. 2014;383(9924):1250-1263. PMID: 24119423Glass AR, Swerdloff RS, Bray GA, Dahms WT, Atkinson RL. Low serum testosterone and sex-hormone-binding-globulin in massively obese men. J Clin Endocrinol Metab. 1977;45(6):1211-1219. PMID: 338622 Plymate SR, Matej LA, Jones RE, Friedl KE. Inhibition of sex hormone-binding globulin production in the human hepatoma ( Hep G2) cell line by insulin and prolactin. J Clin Endocrinol Metab. 1988;67(3):460-464. PMID: 2842359

LIPID / OBESITY

ITE 2017 Question 4 A 49-year-old man with a history of morbid obesity, hypothyroidism, and type 2 diabetes mellitus underwent gastric bypass 14 months ago. His preoperative BMI was 42 kg/m2. After surgery, his diabetes went into remission and his insulin therapy was stopped. His weight decreased and stabilized at a BMI of 29 kg/m2. He was told to take a potent multivitamin; calcium, 1200 mg daily; and sublingual vitamin B12, 500 mcg daily. He has not had any follow-up for the last 4 months. Over the past month, he has had several episodes where he felt shaky, sweaty, and irritable. A family member brought him to the emergency department yesterday at 11:00 AM for confusion that apparently developed after he had a large breakfast at a buffet restaurant. A glucose level of 35 mg/ dL (1.9 mmol /L) was documented. Laboratory test results (fasting): Glucose = 49 mg/ dL (70-99 mg/ dL ) (SI: 2.7 mmol /L [3.9-5.5 mmol /L]) Insulin = 1.6 µIU/mL (1.4-14.0 µIU/mL) (SI: 11.1 pmol /L [9.7-97.2 pmol/L])

After treating his acute decompensation, which of the following is the most appropriate treatment? Low-carbohydrate diet Hydrocortisone Partial pancreatectomy Octreotide Diazoxide Correct Answer: A Learning objective : Recommend an approach to the management of hyperinsulinemic hypoglycemia that develops after gastric bypass surgery.

Rationale: Gastric bypass surgery has a dramatic effect on carbohydrate metabolism. Eighty percent of patients who have diabetes preoperatively do not require medication for diabetes after surgery. Diabetes resolves in many of these individuals within weeks of the operation. It appears that the exposure of distal bowel to food results in exaggerated secretion of glucagonlike peptide 1, which may facilitate the improvement in glucose control seen after surgery. Postprandial hypoglycemia is an uncommon late complication of gastric bypass surgery that is increasingly recognized. It appears that in some individuals, perhaps in response to ongoing stimulation by glucagonlike peptide 1, β-cell proliferation occurs, resulting in nesidioblastosis, which is a state of islet hyperplasia associated with excessive insulin secretion and endogenous hyperinsulinemic hypoglycemia. Some patients develop multiple small insulinomas . The management of this condition is controversial. While partial pancreatectomy was suggested in the initial series, other authors have suggested that many of these patients can be managed by reducing the intake of carbohydrates, consuming low–glycemic index carbohydrates, and always eating carbohydrates in the context of a mixed meal. While acarbose , octreotide (Answer D), calcium channel blockers, and diazoxide (Answer E) have all been used as treatments, diet alone (Answer A) alleviates symptoms in 50% to 70% of affected individuals and is thus the initial treatment of choice.

Rationale Continued: It appears that the condition recurs in many individuals who have a subtotal pancreatectomy, and those who have more aggressive pancreatic surgery can develop pancreatic diabetes. For this reason, pancreatectomy (Answer C) is currently used only if other treatments fail and the patient remains debilitated by frequent episodes of hypoglycemia that limit functional capacity. Although use of hydrocortisone (Answer B) may seem logical, there is no evidence for chronic use of glucocorticoids in the management of hypoglycemia and there is significant potential risk with long-term glucocorticoid use.

Reference(s): Service GJ, Thompson GB, Service FJ, Andrews JC, Collazo-Clavell ML, Lloyd RV. Hyperinsulinemic hypoglycemia with nesidioblastosis after gastric-bypass surgery. N Engl J Med. 2005;353(3):249-254. PMID: 16034010Kellogg TA, Bantle JP, Leslie DB, et al. Postgastric bypass hyperinsulinemic hypoglycemia syndrome: characterization and response to a modified diet. Surg Obes Relat Dis. 2008;4(4):492-499. PMID: 18656831Cui Y, Elahi D, Andersen DK. Advances in the etiology and management of hyperinsulinemic hypoglycemia after Roux- en -Y gastric bypass. J Gastrointest Surg . 2011;15(10):1879-1888. PMID: 21671112

ITE 2017 Question 12 A 45-year-old man is referred to you because of increased transaminase levels while on statin therapy. The patient’s primary care physician previously prescribed simvastatin, 20 mg daily, in an attempt to reduce his LDL-cholesterol concentration of 170 mg/ dL (4.40 mmol /L). The patient’s BMI is 31 kg/m2 and he does not exercise regularly.Laboratory test results:Hemoglobin A1c = 5.8% (4.0%-5.6%) (40 mmol / mol [20-38 mmol / mol ]) TSH, normal Total cholesterol = 170 mg/ dL (<200 mg/ dL [optimal]) (SI: 4.40 mmol/L [<5.18 mmol /L]) HDL cholesterol = 55 mg/ dL (>60 mg/ dL [optimal]) (SI: 1.42 mmol/L [>1.55 mmol/L])Triglycerides = 200 mg/dL (<150 mg/dL [optimal]) (SI: 2.26 mmol/L [<3.88 mmol/L])LDL cholesterol = 75 mg/dL (<100 mg/dL [optimal]) (SI: 1.94 mmol/L [<2.59 mmol/L])ALT = 72 U/L (10-40 U/L) (SI: 1.20 µkat/L [0.17-0.67 µkat/L])AST = 80 U/L (20-48 U/L) (SI: 1.34 µkat/L [0.33-0.80 µkat/L])

Which of the following is the best next step? Stop atorvastatin and change to pravastatin Stop atorvastatin and remeasure transaminase levels off therapy Measure creatine phosphokinaseStop atorvastatin and start a PCSK9 inhibitor Continue atorvastatin Correct Answer: E Learning objective : Develop a management approach to increased transaminase levels in statin-treated patients.

Rationale: Increased transaminase levels while on statin therapy are common and occur in 1% to 2% of patients. However, liver disease caused by statins is rare, and as long as the transaminase increases are less than 3 times the upper normal limit, it is acceptable to continue therapy (thus, Answer E is correct and Answer B is incorrect). For this reason, the US FDA has recently changed its guidelines for monitoring liver function in statin-treated patients. It now recommends checking liver enzymes “as clinically indicated.” Creatine phosphokinase measurement (Answer C) is not indicated in a patient without muscle symptoms. Changing to an alternative statin (Answer A) is not necessary, and because pravastatin is not a high-intensity statin, this change could increase his cardiovascular disease risk. Changing to a PCSK9 inhibitor (Answer D) is not necessary; he is already on a high-intensity statin with a good drop in LDL-cholesterol level.

Reference(s): Herrick C,  Bahrainy S, Gill EA. Statins and the liver. Cardiol Clin. 2015;33(2):257-65. PMID: 25939298Collings R, Reith C, Emberson J, et al. Interpretation of the evidence for the efficacy and safety of statin therapy. Lancet. 2016 [Epub ahead of print] PMID: 27616593

ITE 2017 Question 16 A 55-year-old man is referred for evaluation of his cardiovascular risk and for primary prevention. He does not smoke cigarettes, and he has hypertension controlled with lisinopril and diabetes mellitus controlled with metformin (hemoglobin A1c, 6.2% [44 mmol /mol]). His mother is alive and well in her early 80s; his father died in a motor vehicle crash at age 72 years. The patient has taken low-dosage aspirin daily since his mid-40s, as he heard it could reduce his risk of cardiovascular disease. On physical examination, he appears healthy. His height is 71 in (180.3 cm), and weight is 202 lb (91.8 kg) (BMI = 28.2 kg/m2). His blood pressure is 138/81 mm Hg, and heart rate is 72 beats/min. Findings on examination are normal. Laboratory test results (fasting): Total cholesterol = 195 mg/ dL (<200 mg/ dL [optimal]) (SI: 5.05 mmol /L [<5.18 mmol /L]) LDL cholesterol = 97 mg/ dL (<100 mg/ dL [optimal]) (SI: 2.51 mmol /L [<2.59 mmol/L])HDL cholesterol = 39 mg/dL (>60 mg/dL [optimal]) (SI: 1.01 mmol/L [>1.55 mmol/L])Triglycerides = 295 mg/dL (<150 mg/dL [optimal]) (SI: 3.33 mmol/L [<3.88 mg/dL])Non-HDL cholesterol = 156 mg/dL (<130 mg/dL [optimal]) (SI: 4.04 mmol/L [<3.37 mmol/L])Fasting glucose = 96 mg/dL (70-99 mg/dL) (SI: 5.3 mmol/L [3.9-5.5 mmol/L])

Which of the following is the most appropriate therapy to lower this patient’s cardiovascular risk? A statin A fibrate A PCSK9 inhibitor Ezetimibe No treatment indicated Correct Answer: A Learning objective : Apply current guidelines in a patient at risk for cardiovascular events.

Rationale: This patient has 3 risk factors for cardiovascular disease: hypertension, low HDL-cholesterol concentration, and age (men ≥45 years). Although not all information is provided, this patient has metabolic syndrome (he meets criteria on the basis of high blood pressure, low HDL cholesterol, and high triglycerides). Common causes of secondary dyslipidemia include diabetes mellitus, hypothyroidism, nephrotic syndrome or renal failure, and certain medications. Given the information in the stem, he has no evidence of secondary causes of dyslipidemia. Using the American Heart Association/American College of Cardiology atherosclerotic cardiovascular disease risk calculator guidelines, his 10-year risk is calculated to be 9%; thus, he meets the indication for moderate- or high-intensity statin treatment (thus, Answer A is correct). Statins, fibrates, and niacin have all been shown to reduce cardiovascular risk in monotherapy studies. However, statins have the best evidence in support of their use and should be the first option tried. Cardiovascular risk reduction with fibrates (Answer B) has been controversial, and some, but not all, studies show risk reduction using fibrates as monotherapy. In general, patients with high triglyceride levels (>200 mg/ dL [>2.26 mmol /L]) and low HDL-cholesterol levels (<40 mg/ dL [<1.04 mmol /L]) comprise the population that is most likely to benefit. Thus, fibrate therapy is a reasonable option for this individual if statins are not tolerated.

Rationale Continued: PCSK9 inhibitors (Answer C) ( proprotein convertase subtilisin /kexin type 9 inhibitors) are a newly approved class of drugs indicated for use in patients with familial hypercholesterolemia or atherosclerotic cardiovascular disease whose lipid values are not at goal; this is not the case in this vignette. Ezetimibe (Answer D) has been shown to reduce cardiovascular events only when combined with statins. Per the current guidelines, treatment is indicated for this patient (thus, Answer E is incorrect).

Reference(s): Stone NJ, Robinson JG, Lichtenstein AH, et al; American College of Cardiology/American Heart Association Task Force on Practice Guidelines [published corrections appear in J Am Coll Cardiol. 2015;66(24):2812 and J Am Coll Cardiol. 2014;63(25 Pt B):3024-3025]. J Am Coll Cardiol. 2014;63(25 Pt B):2889-2934. PMID: 24239923Ridker PM, Danielson E, Fonseca FA, et al; JUPITER Study Group. Rosuvastatin to prevent vascular events in men and women with elevated C-reactive protein. N Engl J Med. 2008;359(21):2195-2207. PMID: 18997196Lee M, Saver JL, Towfighi A, Chow J, Ovbiagele B. Efficacy of fibrates for cardiovascular risk reduction in persons with atherogenic dyslipidemia: a meta-analysis. Atherosclerosis . 2011;217(2):492-498. PMID: 21592479 Jun M, Foote C, Lv J, et al. Effects of fibrates on cardiovascular outcomes: a systematic review and meta-analysis. Lancet . 2010;375(9729):1875-1884. PMID: 20462635Taylor F, Huffman MD, Macedo AF, et al. Statins for the primary prevention of cardiovascular disease. Cochrane Database Syst Rev. 2013;(1):CD004816. PMID: 23440795

ITE 2017 Question 22 A 47-year-old woman presents for follow-up. She has a family history of type 2 diabetes mellitus. Six months ago, impaired fasting glucose was diagnosed and you referred her to a 12-week intensive lifestyle program that incorporated dietary advice, physical activity recommendations, and behavioral therapy. She completed the intensive lifestyle program and now participates in a monthly group weight management session. She tracks her food intake and is mindful of triggers that promote overeating. She does not participate in an exercise program, although she tries to walk a few times each week. She weighs herself at least once weekly. She also has a history of hypertension, which is currently treated with lisinopril , 10 mg daily. On physical examination, her blood pressure is 130/80 mm Hg and pulse rate is 78 beats/min. Her height is 64 in (162.5 cm). Six months ago, she weighed 196 lb (89.1 kg) (BMI = 33.6 kg/m2). Today, she weighs 181.5 lb (82.5 kg) (BMI = 31.2 kg/m2). Examination findings are otherwise unremarkable.

You commend her on her weight loss. However, you worry that this patient is at high risk of weight regain because: She has no regular exercise program She has experienced insufficient weight loss given her efforts Her monthly group meeting provides inadequate support Her weight monitoring is too infrequent The intensive lifestyle program was too short Correct Answer: A Learning objective : Counsel patients on weight-loss expectations with an intensive lifestyle program and identify risk factors contributing to weight regain.

Rationale: Unfortunately, weight gain is common after a weight-loss effort and is often due to many factors. Increased physical activity has been consistently shown to promote weight maintenance, although how much physical activity is needed remains debatable. In the Diabetes Prevention Program, the exercise recommendation was greater than 150 minutes of moderate-intensity exercise per week during the weight-loss effort. In the Look AHEAD trial (Action for Health in Diabetes), the exercise recommendation was 175 minutes of moderate-intensity exercise per week. However, several studies suggest that more than 200 minutes of moderate-intensity exercise per week may be needed to counteract the drop in energy expenditure that is experienced with weight loss and to protect against weight gain. This patient is at risk of weight gain if she does not engage in weekly exercise and make the necessary adjustments to achieve at least 200 minutes of moderate-intensity exercise per week (thus, Answer A is correct). The Diabetes Prevention Program studied the effect of intensive lifestyle changes compared with the effect of standard care and metformin therapy in the prevention of type 2 diabetes mellitus among patients with dysglycemia . The main lifestyle goals in the intensive group included a weight loss greater than 7%, moderate-intensity exercise for 150 minutes per week, and diet consisting of less than 30% of calories from fat.

Rationale Continued: The risk for developing type 2 diabetes decreased by 58% in the intensive lifestyle group compared with the risk in control patients despite only 47% of the patients achieving a weight loss greater than 7%. The lifestyle goal most commonly achieved was the exercise goal (150 minutes of moderate-intensity exercise per week). This patient has lost 14.5 lb (6.6 kg), representing 7.4% of her initial weight. Hence, she has achieved the amount of weight loss expected with an intensive lifestyle intervention, as well as a lowered risk for progression to type 2 diabetes mellitus, which was her goal (thus, Answer B is incorrect). An intensive lifestyle intervention program is defined as 14 or more sessions with a trained interventionist (physician, dietitian, psychologist, exercise specialist, health coach) in a 6-month period and is associated with an average weight loss of 5% to 10%. Moderate-intensity programs, 1 to 2 sessions with a trained interventionist per month over a 6-month period, are associated with an average weight loss of 3% to 5%. Many of these programs are implemented in a group setting, and this patient’s weight loss is consistent with this type of intervention (thus, Answer E is incorrect). In patients who successfully lose weight, ongoing participation with a trained interventionist is advised on at least a monthly basis for a year. The patient is doing this via her group meeting (thus, Answer C is incorrect).

Rationale Continued: Self-monitoring is a key behavior associated with success. In addition to monitoring eating and physical activity, monitoring weight is associated with more weight loss and weight maintenance. Among participants in the National Weight Control Registry, monitoring weight at least once weekly, which this patient does, was associated with greater success at weight maintenance (thus, Answer D is incorrect).

Reference(s): American College of Cardiology/American Heart Association Task Force on Practice Guidelines, Obesity Expert Panel, 2013. Executive Summary: Guidelines (2013) for the management of overweight and obesity in adults: a report of the American College of Cardiology/American Heart Association Task Force on Practice Guidelines and the Obesity Society published by the Obesity Society and American College of Cardiology/American Heart Association Task Force on Practice Guidelines. Based on a systematic review from The Obesity Expert Panel, 2013. Obesity (Silver Spring) . 2014;22( Suppl 2):S5-S39. PMID: 24961825Jakicic JM, Winters C, Lang W, Wing RR. Effects of intermittent exercise and use of home exercise equipment on adherence, weight loss, and fitness in overweight women: a randomized trial. JAMA. 1999;282(16):1554-1560. PMID: 10546695Look AHEAD Research Group, Wing RR. Long-term effects of a lifestyle intervention on weight and cardiovascular risk factors in individuals with type 2 diabetes mellitus: four-year results of the Look AHEAD trial. Arch Intern Med. 2010;170(17):1566-1575. PMID: 20876408Tuomilehto J, Lindstrom J, Ericksson JG, Valle TT, Hamalainen H, IIanne-Parikka P, Keinanen - Kiukaanniemi S, et al. Prevention of type 2 diabetes mellitus by changes in lifestyle among subjects with impaired glucose tolerance. N Engl J Med . 2001;344(18):1343-1350. PMID: 11333990

ITE 2017 Question 29 A 52-year-old woman with familial hypercholesterolemia has been treated with rosuvastatin , 40 mg daily, and ezetimibe, 10 mg daily, for the past 5 years. This combination has reduced her total cholesterol concentration from 385 to 215 mg/ dL (9.97 to 5.57 mmol/L) and her LDL-cholesterol concentration from 300 to 120 mg/dL (7.77 to 3.11 mmol /L). She now presents with a total cholesterol concentration of 260 mg/ dL (6.73 mmol /L) and reports muscle pains. Creatine phosphokinase and blood glucose levels are not elevated.

Measuring which of the following would be most useful now? AST and ALT 8-AM cortisol TSH Urinary protein Urinary myoglobin Correct Answer: C Learning objective : Evaluate the etiology of myopathy in a patient taking a statin.

Rationale: The most common cause of medication ineffectiveness is nonadherence to the treatment regimen. In the case of statins, the development of muscle symptoms is a common cause of medication nonadherence in previously well-treated patients. In clinical trials, the prevalence of myalgias is 3% to 5% and not significantly different from the prevalence observed with placebo. However, clinical practice reports document a prevalence of myalgias of up to 25%. It is important to note that overt myopathy (myalgias with creatine phosphokinase elevations) is relatively infrequent. The cause of myalgia and myopathy with statin use is not known. Careful placebo-controlled studies have demonstrated that most patients reporting myalgias with statin therapy also have myalgias on placebo or non-statin lipid-lowering medications, suggesting that the symptoms are not specific for statins. Although patients can develop muscle symptoms any time after initiation of statin therapy, the development of hypothyroidism after 5 years of therapy suggests a new acquired cause. Hypothyroidism certainly can contribute, and TSH measurement (Answer C) is therefore the best test to order for this patient. There is no association of elevated transaminases (Answer A) with myalgias , although if values are greater than 3 times the upper normal limit, then the statin may need to be held and tests investigating causes of liver injury (such as acute hepatitis) may be appropriate. Adrenal insufficiency or excess is not known to be associated with statin bioavailability or myalgias (thus, Answer B is incorrect). Urinary protein (Answer D) and myoglobin (Answer E) are elevated in rhabdomyolysis, but this patient has relatively mild muscle symptoms and her creatine phosphokinase is not elevated, so the diagnosis of rhabdomyolysis very unlikely.

Reference(s): Thompson PD, Clarkson P, Karas RH. Statin-associated myopathy. JAMA . 2003;289(13):1681-1690. PMID: 12672737Abrams JJ, Grundy SM. Cholesterol metabolism in hypothyroidism and hyperthyroidism in man. J Lipid Res. 1981;22(2):323-338. PMID: 7240961Ito M, Arishima T, Kudo T, et al. Effect of levo-thyroxine replacement on non-high-density lipoprotein cholesterol in hypothyroid patients. J Clin Endocrinol Metab. 2007;92(2):608-611. PMID: 17148561Monzani F, Caraccio N, Kozàkowà M, et al. Effect of levothyroxine replacement on lipid profile and intima-media thickness in subclinical hypothyroidism: a double-blind, placebo-controlled study. J Clin Endocrinol Metab . 89(5):2099-2106. PMID: 15126526

ITE 2017 Question 34 A 42-year-old woman presents for her annual thyroid check. She has a history of Graves disease treated with radioactive iodine, and she has been on a stable dosage of thyroid hormone supplementation for the past 6 years. In the last 12 months, hypertension and tension headaches were diagnosed. Her blood pressure is now controlled with metoprolol, and a gradual titration of amitriptyline has helped her headaches. Her main concerns relate to her weight. Over the past year, she has gained 20 lb (9.1 kg). She worries that her thyroid hormone dosage is inadequate. Her current medications are levothyroxine, 100 mcg daily; metoprolol, 100 mg daily; cetirizine, 10 mg daily (for seasonal allergies); amitriptyline, 75 mg daily; and naproxen (as needed for headaches). Her TSH concentration is 2.4 mIU /L (0.5-5.0 mIU /L). On physical examination, her height is 65 in (165 cm) and weight is 173.5 lb (78.9 kg) (BMI = 28.9 kg/m2). Her thyroid gland is not palpable. The rest of her examination findings are unremarkable.

In addition to discussing efforts at lifestyle changes to help manage her weight, which of the following medications should you consider to be the most important factor contributing to her weight gain? Amitriptyline Inadequate levothyroxine Metoprolol Cetirizine Naproxen Correct Answer: A Learning objective : Identify medications that can contribute to weight gain and consider alternatives.

Rationale: Weight gain is common during adulthood, and recognizing potential risk factors for weight gain presents a unique opportunity to help patients successfully manage weight. Several commonly used medications for the management of chronic diseases are associated with weight changes. Antipsychotic medications ( eg , olanzapine, clozapine) are associated with the most weight gain, but significant weight gain can also be observed with several glucose-lowering agents such as insulin secretagogues , insulin, and thiazolidinediones. Addressing this patient’s concern should include a review of her current medications. Weight gain observed as a result of medication tends to occur early in therapy, within 3 to 6 months of initiation. In this patient, the 2 potential culprits are her amitriptyline (Answer A) and her metoprolol (Answer C). β-Adrenergic blockers are associated with modest weight gain (<3.3 lb [<1.5 kg]). Among β-adrenergic blockers, propranolol therapy is associated with the most weight gain. The GEMINI trial compared weight changes between carvedilol and metoprolol among patients with hypertension and type 2 diabetes. Metoprolol was associated with weight gain mainly in patients with higher baseline BMI values, particularly those meeting the criteria for obesity (BMI >30 kg/m2). Among patients with BMI values in the normal or overweight category, weight gain observed was not significant. This study’s findings may not completely apply to this patient with no history of type 2 diabetes (thus, Answer C is incorrect).

Rationale Continued: Tricyclic antidepressants are well recognized to be associated with weight gain, although the amount of weight gained varies markedly. In this patient, the initiation of amitriptyline is temporally associated with the onset of her weight gain (thus, Answer A is correct). Assessing for other therapeutic options in the management of her headaches represents an important opportunity. Weight loss is observed upon initiation of thyroid hormone replacement in hypothyroid adults. Karmisholt et al reported an average weight loss of 9.5 lb (4.3 kg) after a year of levothyroxine therapy and achieving an average TSH concentration of 2.2 mIU /L (compared with a concentration of 102 mIU /L at treatment initiation). However, most of the weight loss observed was related to a decrease in lean mass compartment and not in bone and fat mass as measured by DXA. Investigators suggest most of the weight loss is related to excretion of excess body water. Although this patient is appropriately concerned about the adequacy of her thyroid hormone supplementation, her TSH level suggests that her levothyroxine dosage is appropriate and adjustments are unlikely to offer additional benefit to her weight-loss efforts (thus, Answer B is incorrect).

Rationale Continued: Nonsteroidal anti-inflammatory agents such as naproxen have not been independently recognized to promote weight gain (thus, Answer E is incorrect), although inactivity as a result of painful joint disease could contribute to weight gain. β-Adrenergic blockers are associated with modest weight gain (<3.3 lb [<1.5 kg]). Agents used to treat allergies have also been proposed to promote weight gain, although currently available studies are insufficient to confirm the association. No studies have been conducted to evaluate the effect of cetirizine on weight (thus, Answer D is incorrect).

Reference(s): Leslie WS, Hankey CR, Lean ME. Weight gain as an adverse effect of commonly prescribed drugs: a systematic review. QJM . 2007;100(7):395-404. PMID: 17566010 Messerli FH, Bell DS, Fonseca V, et al; GEMINI Investigators. Body weight changes with beta- blocker use: results from GEMINI. Am J Med. 2007;120(7):610-615. PMID: 17602935Karmisholt J, Andersen S, Laurberg P. Weight loss after therapy for hypothyroidism is mainly caused by excretion of excess body water associated with myxoedema. J Clin Endocrinol Metab. 2011;96(1):E99- E103. PMID: 20926526

ITE 2017 Question 46 A 51-year-old man comes to see you for a second opinion about his cholesterol. He had a recent hospital admission for chest pain, and a coronary stent was placed. His father died of a myocardial infarction at age 49 years (he smoked 2 packs of cigarettes per day), and his brother developed angina at age 52 years. Four years ago, his primary care physician prescribed atorvastatin, 40 mg daily, for an LDL-cholesterol level of 135 mg/ dL (3.50 mmol /L). His LDL-cholesterol level is now 90 mg/dL (2.33 mmol /L), and his triglyceride level is 210 mg/ dL (2.37 mmol /L). His primary care physician says that he does not need any more medication. However, given his recent hospitalization, the patient thinks his LDL-cholesterol level should be lower and he comes to you for a second opinion.

Which of the following should be added as the best next step? Ezetimibe Fenofibrate Niacin Cholestyramine Omega-3 fatty acids Correct Answer: A Learning objective : Summarize the available data on the utility of adding a second lipid-lowering agent to statins with the goal of reducing cardiovascular events

Rationale: The 2013 guidelines from the American Heart Association on the management of cholesterol focus almost exclusively on the use of statins to lower cardiovascular disease risk. At the time the guidelines were written, there was not yet data from a randomized controlled trial demonstrating that the addition of another agent to a statin significantly reduced risk. Fenofibrate (Answer B), niacin (Answer C), bile acid–binding resins (Answer D), and omega-3 fatty acids (Answer E) have not been shown to reduce cardiovascular event rates when added to statins. However, data from the IMPROVE-IT trial recently demonstrated that ezetimibe (Answer A) added to a regimen of 40 mg daily of simvastatin resulted in a modest but statistically significant 6.4% reduction in a composite cardiovascular end point. Evolocumab and alirocumab (not listed as answer options) are monoclonal antibodies that target and degrade proprotein convertase subtilisin / kexin type 9 (PCSK9). PCSK9 is a protein that was discovered through genetic screening of individuals with very low LDL-cholesterol levels. PCSK9 circulates in the blood and alters the liver’s handling of LDL cholesterol. Persons with genetic mutations that lower PCSK9 levels have lower LDL-cholesterol levels. These drugs lower LDL cholesterol up to 50% in addition to statin treatment, and early studies suggest a benefit of decreased cardiovascular events. However, at this time, these agents have not yet been established to decrease cardiovascular events.

Reference(s): DiNicolantonio JJ, Chatterjee S, Lavie CJ, Bangalore S, O'Keefe JH. Ezetimibe plus moderate dose simvastatin after acute coronary syndrome: what are we IMPROVEing on? Am J Med. 2015;128(8):914.e1-e4. PMID: 25731133Sabatine MS, Giugliano RP, Wiviott SD, et al; Open-Label Study of Long-Term Evaluation against LDL Cholesterol (OSLER) Investigators. Efficacy and safety of evolocumab in reducing lipids and cardiovascular events. N Engl J Med. 2015;372;(16):1500-1509. PMID: 25773607

ITE 2017 Question 55 A 52-year-old woman comes to your office concerned about osteoporosis as a consequence of bariatric surgery. Her mother, who is 76 years old, just experienced a painful vertebral compression fracture. The patient is worried about her own fracture risk since she is considering undergoing bariatric surgery to manage her obesity. She has never had a fracture. She has taken calcium and vitamin D supplements since menopause 2 years ago. She stays physically active, walking 30 to 45 minutes 3 to 4 times per week. She underwent DXA 1 month ago, which revealed T scores of –1.1 and –1.4 at the spine and femoral neck, respectively. Current medications include a multivitamin; calcium citrate, 1200 mg daily; and vitamin D, 2000 IU daily. On physical examination, her height is 66 in (167.6 cm) and weight is 248 lb (112.7 kg) (BMI = 40 kg/m2). Her blood pressure is 125/85 mm Hg, and pulse rate is 75 beats/min. The rest of her examination findings are normal. Laboratory test results: Chemistry panel (including calcium and alkaline phosphatase), normal 25-Hydroxyvitamin D = 39 ng/mL (25-80 ng/mL [optimal]) (SI: 97.3 nmol /L [62.4-199.7 nmol /L]) PTH = 50 pg /mL (10-65 pg /mL) (SI: 50 ng/L [10-65 ng/L]) Urinary calcium = 188 mg/g creatinine

Which of the following changes is most likely to be observed in this patient if she is evaluated 12 months after undergoing a Roux- en -Y gastric bypass operation? Decreased bone resorption markers Decreased femoral neck bone mineral density Increased incidence of hip fractureDecreased PTH levels Increased urinary calcium excretion Correct Answer: B Learning objective : Assess risks of bone loss and fracture associated with bariatric surgery.

Rationale: Bariatric surgery has been associated with significant improvement in metabolic complications of excess weight. Despite this, there are concerns about the effect of bariatric surgery on bone health because of several physiologic changes observed in calcium, vitamin D, and bone metabolism that predispose to bone loss after bariatric surgery. Weight loss, even when modest, is associated with increased bone turnover markers, particularly bone resorption markers (thus, Answer A is incorrect). These changes can be observed a year after bariatric surgery. Decreased skeletal loading is one proposed mechanism. The observed bone loss is greater in the hip region (trochanter, femoral neck) than in the spine (thus, Answer B is correct). Nutritional factors also have a role in bone loss, particularly when patients have preexisting deficiencies in calcium and vitamin D that might not be identified before their operation. Serum calcium levels are rarely abnormal and calcium deficiency can be identified on the basis of decreased urinary calcium excretion (thus, Answer E is incorrect). Operations that introduce malabsorption (bypassing the duodenum and proximal jejunum) as a mechanism for weight loss can further aggravate these deficiencies. Such changes in calcium and vitamin D metabolism are often associated with increased PTH levels (thus, Answer D is incorrect), and in some patients mild elevations may persist despite calcium and vitamin D supplementation.

Rationale Continued: This is a topic of active investigation and the current literature is limited by multiple factors including the population studied (mostly premenopausal women) and duration of follow-up. Despite the increase in bone turnover and the reduction in bone density at the hip, there does not appear to be an increased risk of hip fracture in patients within the first year after bariatric surgery (thus, Answer C is incorrect). In assessing risks for bone health, ensuring the patient is calcium and vitamin D replete is important before and after bariatric surgery. Guidelines for screening with DXA have not been firmly established given the lack of studies and inherent limitations of DXA in obese patients and in the setting of active weight loss accompanied by changes in fat mass. However, in patients at high risk, such as perimenopausal women, bone mineral density assessed by DXA can be followed, but should be interpreted cautiously. Technology such as high-resolution peripheral quantitative CT provides more accurate measurements, as well as information on the extent of cortical vs trabecular bone loss. However, this technology is not widely available as part of clinical practice and has a greater radiation exposure than DXA.

Reference(s): Stein EM, Carrelli A, Young P, et al. Bariatric surgery results in cortical bone loss. J Clin Endocrinol Metab. 2013;98(2):541-549. PMID: 23295461Scibora LM, Ikramuddin S, Buchwald H, Petit MA. Examining the link between bariatric surgery, bone loss, and osteoporosis: a review of bone density studies. Obes Surg. 2012;22(4):654-667. PMID: 22271358

ITE 2017 Question 69 A 36-year-old woman with a peak lifetime BMI of 46 kg/m2 had a laparoscopic gastric bypass operation in another state 8 weeks ago. She initially did well, but over the last 3 weeks she began to experience episodes of vomiting. Over the last 5 days, she has been vomiting almost everything she eats. Over the last 2 days, her husband says that she has become increasingly confused, dysarthric , and unsteady on her feet. On neurologic examination, she is clearly confused, has nystagmus, is unsteady on standing, has decreased sensation on her lower extremities, and has a right third nerve palsy.

This patient most likely has a deficiency of which of the following? Vitamin B 12 Vitamin D (severe) Thiamine FolateZinc Correct Answer: C Learning objective : Differentiate among the vitamin deficiencies that can occur after gastric bypass surgery.

Rationale: The symptoms displayed by this patient are characteristic of Wernicke encephalopathy, which is caused by a deficiency of thiamine (Answer C). Thiamine deficiency causes neuronal death due to metabolic dysfunction of astrocytes within the central nervous system. The classic triad of this condition is confusion, ataxia, and nystagmus. A wide range of other abnormalities can be seen, including cranial nerve dysfunction, peripheral neuropathies, seizures, and psychosis. Because thiamine is a water-soluble vitamin, body stores can be depleted within days to weeks of inadequate intake. The condition typically presents 4 to 12 weeks after bariatric surgery but can occur as early as 2 weeks and as late as 18 months after surgery. Although most commonly reported following gastric bypass surgery, Wernicke encephalopathy can occur after any type of bariatric surgery. The most common antecedent is persistent vomiting, which then severely limits thiamine intake. Other less common precipitating factors are intravenous glucose or parenteral nutrition administration without thiamine supplementation. The condition is important to recognize, as treatment with parenteral thiamine (100 mg daily for 7 to 14 days, or 500 mg 3 times daily for 3 days) must be administered to prevent serious morbidity.

Rationale Continued: Although vitamin B12 deficiency (Answer A) can cause neurologic symptoms and signs, body stores of B12 are sizable, so deficiency does not usually occur until 6 to 24 months after bariatric surgery. Folate deficiency (Answer D) is uncommon and typically presents as anemia. Zinc deficiency (Answer E) is rare; it is associated with skin and hair findings and is primarily seen after biliary pancreatic diversion. Vitamin D deficiency (Answer B) can cause generalized weakness, but vitamin D is fat-soluble, so deficiency typically occurs months or years after bariatric surgery. In addition, vitamin D deficiency would not be expected to cause the focal neurologic signs that this patient exhibits

Reference(s): Aasheim ET. Wernicke encephalopathy after bariatric surgery, a systematic review. Ann Surg . 2008;248(5):714-720. PMID: 18948797 Serra A, Sechi G, Singh S, Kumar A. Wernicke encephalopathy after obesity surgery: a systematic review. Neurology. 2007;69(6):615. PMID: 17679686Mechanick JI, Kushner RF, Sugerman HJ, et al; American Association of Clinical Endocrinologists; Obesity Society; American Society for Metabolic & Bariatric Surgery. American Association of Clinical Endocrinologists, The Obesity Society, and American Society for Metabolic & Bariatric Surgery medical guidelines for clinical practice for the perioperative nutritional, metabolic, and nonsurgical support of the bariatric surgery patient [published correction appears in Obesity (Silver Spring). 2010;18(3):649]. Obesity (Silver Spring). 2009;17(Suppl 1):S1-S70. PMID: 19319140

ITE 2017 Question 73 A 32-year-old woman is seen for follow-up of a weight-loss effort. Since her diagnosis of type 2 diabetes mellitus 1 year ago, she has been focusing on implementing a more healthful lifestyle. She stopped smoking, but she gained weight. She has been participating in a meal replacement program for the past 6 months, which includes twice-weekly group meetings. She uses an activity tracking device and tries to achieve 5000 to 10,000 steps daily. Although she has lost some weight, she is hoping to lose more. She asks about medications for weight loss. Her medical history is notable for type 2 diabetes mellitus controlled with metformin monotherapy, polycystic ovary syndrome, recurrent urinary tract infections, depression, and hypothyroidism. Current medications are metformin, 850 mg twice daily; sertraline, 50 mg daily; levothyroxine, 112 mcg daily; and an oral contraceptive. Findings on physical examination are normal. Her hemoglobin A1c level is 6.8% (51 mmol / mol ) (reference range, 4.0%-5.6% [20-38 mmol / mol ]). Her TSH concentration is 4.2 mIU /L (reference range, 0.5-5.0 mIU /L). One year ago (before she began lifestyle changes), her weight was 230 lb (104.5 kg) and height was 66.5 in (168.9 cm) (BMI = 36.6 kg/m2). Her current weight is 219 lb (99.5 kg) (BMI = 34.8 kg/m2).

In addition to recommending ongoing diet and exercise, which of the following medication changes would provide the most benefit in managing her type 2 diabetes mellitus and obesity? Add dapagliflozin Add liraglutide Increase the levothyroxine dosage Increase the metformin dosage Discontinue the oral contraceptive Correct Answer: B Learning objective : Choose the best therapy to promote additional weight loss beyond that expected with lifestyle changes alone in a patient with type 2 diabetes mellitus

Rationale: This patient is a good candidate for medical therapy because she has shown motivation by implementing lifestyle changes that have been associated with a weight loss of 11 lb (5 kg) (about 5% of her original body weight). Her glycemic control is adequate; however, if improved glycemic control is a therapeutic goal, the choice of glucose-lowering therapy beyond metformin is critically important because many medications are associated with weight gain ( eg , insulin secretagogues, insulin, thiazolidinediones ) and can hamper weight-loss efforts. Thus, choosing a glucose-lowering agent that can also offer benefits with respect to weight would be desirable. Dapagliflozin (Answer A), a sodium-glucose cotransporter 2 inhibitor, and liraglutide (Answer B), a glucagonlike peptide 1 receptor agonist, can improve glycemic control and have a beneficial effect on weight. Glucagonlike peptide 1 receptor agonist therapy is associated with greater weight loss than that observed with sodium-glucose cotransporter 2 inhibitors. Sodium-glucose cotransporter 2 inhibitors promote weight loss by inducing glucosuria . Glucosuria increases the risk for genitourinary infections, an established medical problem for this patient. Initiation of dapagliflozin is therefore not the best option.

Rationale Continued: Liraglutide improves glycemia by enhancing insulin secretion and inhibiting glucagon secretion. It promotes weight loss via centrally acting satiety effects, but also by delaying gastric emptying, which restricts caloric intake. This contributes to the most common adverse effects of nausea and vomiting. However, these symptoms subside after a few weeks of therapy in most patients. In fact, clinical trials investigating liraglutide therapy for patients without diabetes (dosages of 0.6-3.0 mg daily) show completion rates greater than 80%. Liraglutide dosages for the treatment of type 2 diabetes mellitus are 0.6-1.8 mg daily. When considering her medical history and current medications, liraglutide (the preparation for treatment of type 2 diabetes, 0.6-1.8 mg daily) presents the best option (thus, Answer B is correct). Metformin is not associated with weight gain. This is attributed to satiety effects leading to decreased calorie consumption. Although results vary, meta-analyses suggest that metformin therapy is at minimum weight neutral or associated with a weight loss of 3% to 5%. Satiety effects were observed with dosages greater than 1500 mg daily. This patient’s current metformin dosage is 1700 mg daily. Although increasing her metformin dosage (Answer D) would be associated with additional glucose lowering, it is unlikely to provide additional weight loss.

Rationale Continued: Her current TSH level suggests adequate levothyroxine supplementation, so dosage adjustments (Answer C) would not be advised to help with her weight management. There is little evidence to suggest that oral contraceptive use is associated with weight gain or that discontinuing therapy with oral contraceptives (Answer E) results in better weight-loss outcomes in normal-weight or overweight women.

Reference(s): Smith SR, Weissman NJ, Anderson CM, et al; Behavioral Modification and Lorcaserin for Overweight and Obesity Management (BLOOM) Study Group. Multicenter, placebo-controlled trial of lorcaserin for weight management. N Engl J Med. 2010;363(3):245-256. PMID: 20647200Johansen K. Efficacy of metformin in the treatment of NIDDM. Meta-analysis. Diabetes Care. 1999;22(1):33-37. PMID: 10333900Lee A, Morley JE. Metformin decreases food consumption and induces weight loss in subjects with obesity with type II non-insulin-dependent diabetes. Obes Res. 1998;6(1):47-53. PMID: 9526970Astrup A, Carraro R, Finer N, et al; NN8022-1807 Investigators. Safety, tolerability and sustained weight loss over 2 years with the once-daily human GLP-1 analog, liraglutide . [published corrections appear in Int J Obes ( Lond ) . 2012;36(6):890 and Int J Obes ( Lond). 2013;37(2):322]. Int J Obes (Lond). 2012;36(6):843-854. PMID: 21844879Apovian CM, Aronne L, Rubino D, et al; COR-II Study Group. A randomized, phase 3 trial of naltrexone SR/bupropion SR on weight and obesity-related risk factors (COR-II). Obesity (Silver Spring). 2013;21(5):935-943. PMID: 23408728List JF, Woo V, Morales E, Tang W, Fiedorek FT. Sodium-glucose cotransport inhibition with dapagliflozin in type 2 diabetes. Diabetes Care. 2009;32(4):650-657. PMID: 19114612

ITE 2017 Question 77 A 41-year-old man is referred for management of his lipids. Two years ago, he was found to have markedly elevated triglycerides and therapy with a medication was initiated. This medication reportedly resulted in a good response, but the patient decided to stop taking it. He is otherwise healthy and does not take any medications. He does not smoke cigarettes, but drinks 15 alcoholic beverages every week. He eats fast food on a daily basis. Both of his parents have type 2 diabetes mellitus, and many paternal family members are known to have high triglycerides, but there is no family history of premature atherosclerotic cardiovascular disease. He has a healthy 13-year-old daughter. On physical examination, his height is 72 in (182.9 cm) and weight is 170 lb (77.3 kg) (BMI = 23.1 kg/m2). His blood pressure is 112/76 mm Hg, and pulse rate is 74 beats/min. He has no rashes, his abdomen is not tender, and there is no hepatosplenomegaly. The rest of his physical examination findings are unremarkable.

Laboratory test results (sample drawn while fasting): Total cholesterol = 456 mg/ dL (<200 mg/ dL [optimal]) (SI: 11.81 mmol/L [<5.18 mmol/L])HDL cholesterol = 16 mg/ dL (>60 mg/ dL [optimal]) (SI: 0.41 mmol /L [>1.55 mmol /L]) Triglycerides = 3790 mg/ dL (<150 mg/ dL [optimal]) (SI: 42.83 mmol/L [<3.88 mmol/L]) Apolipoprotein B = 98 mg/ dL (50-110 mg/ dL ) (SI: 0.98 g/L [0.5-1.1 g/L]) Hemoglobin A1c = 5.6% (4.0%-5.6%) (38 mmol/mol [20-38 mmol/mol])TSH = 2.4 mIU/L (0.5-5.0 mIU/L)

In addition to recommending lifestyle changes, which of the following is the most important treatment to initiate for this patient’s markedly elevated triglycerides? A statin A statin + ezetimibe Niacin A fibrate Fish oil Correct Answer: D Learning objective : Describe the risks associated with very high triglyceride levels and initiate appropriate treatment.

Rationale: This patient has marked hypertriglyceridemia with a triglyceride level greater than 1000 mg/ dL (>11.30 mmol /L). The primary goal in this setting is prevention of pancreatitis by maintaining triglyceride levels below that at which pancreatitis occurs. Although not all individuals with very high triglyceride levels develop pancreatitis, from a practical standpoint, it is best to maintain levels at less than 1000 mg/dL (11.30 mmol /L)—ideally as low as possible. Marked hypertriglyceridemia occurs when patients who have an underlying familial form of dyslipidemia develop another condition that results in secondary hypertriglyceridemia. Common secondary causes include untreated or uncontrolled diabetes, hypothyroidism, excessive alcohol consumption, or use of drugs that can raise triglycerides such as estrogens, β-adrenergic blockers, and thiazides. This patient consumes fairly large amounts of alcohol on a regular basis, which is a likely contributor to his very high triglyceride levels. Since pancreatitis is a major and life-threatening complication in these individuals, there is an urgent need to decrease his triglycerides. Reversal of the secondary cause of hypertriglyceridemia is necessary, and in this patient it would involve decreasing or stopping alcohol consumption.

Rationale Continued: Modest alcohol intake (2 drinks per day for men and 1 drink per day for women) is considered acceptable. However, patients with severe hypertriglyceridemia should abstain from alcohol. A healthy lifestyle can help reduce triglycerides. Weight loss of 5% to 10% body weight reduces triglycerides by approximately 22% and aerobic exercise also produces similar effects. The mainstay of treatment, however, is triglyceride-lowering agents to maintain triglycerides at an acceptable level. There are no set targets for triglyceride levels other than to reduce the risk of pancreatitis by decreasing levels below 1000 mg/ dL . Fibrates (Answer D) lower triglycerides by activating peroxisome proliferator–activated receptor α, increasing lipoprotein lipase activity, and increasing catabolism of triglycerides. This works particularly well in individuals with marked hypertriglyceridemia (20%-50% reduction, sometimes up to 70%). Fibrates are the recommended first- line agents in patients at risk for triglyceride-induced pancreatitis, such as this patient. Available fibrates in the United States include gemfibrozil (twice-daily dosing) and fenofibrate (once-daily dosing).

Rationale Continued: Statins are primarily cholesterol-lowering agents and they modestly decrease triglyceride levels. In combination with ezetimibe, statins decrease LDL cholesterol in an additive manner, but effects on triglycerides are not profound. Thus, neither statin use alone (Answer A) nor a statin combined with ezetimibe (Answer B) would be able to lower this patient’s triglycerides to a safe level. Niacin (Answer C) decreases triglyceride levels up to 25%, but reductions are smaller than those achieved with fibrates. Its mechanism of action on triglycerides is unclear, but it is most likely related to its effect on hepatic VLDL synthesis. Niacin can be considered as add-on therapy if triglyceride reduction with fibrates is inadequate. Fish oil (Answer E) (as concentrated eicosapentaenoic acid and docosahexaenoic acid) has modest effects on triglycerides with reductions up to 50%. Fish oil reduces triglycerides by decreasing VLDL synthesis and increasing its catabolism. However, in marked hypertriglyceridemia it is not considered first-line therapy.

Reference(s): Berglund L, Brunzell JD, Goldberg AC, et al; Endocrine Society. Evaluation and treatment of hypertriglyceridemia: an Endocrine Society clinical practice guideline. J Clin Endocrinol Metab. 2011;97(9):2969-2989. PMID: 22962670

PITUITARY

ITE 2017 Question 13 A 25-year-old woman is referred to a gynecologist because of oligomenorrhea over the past several years. At first, she began to skip occasional periods, but the interval between menstrual periods has increased in length and she has had only 2 periods over the last year. Pelvic examination findings are normal, and she does not have acne or hirsutism. Her height is 66 in (167.6 cm), and weight is 171 lb (77.7 kg) (BMI = 27.6 kg/m2). Her serum prolactin concentration is 210 ng/mL (9.1 nmol /L), confirmed on serial dilution. She does not report galactorrhea, but on physical examination, her gynecologist is able to express whitish secretory product from both nipples. In taking a history, you learn that the patient had to interrupt a year in college because she developed schizophrenia. Initially, she was treated with olanzapine, but she did not want to continue it because she gained weight. After trying several other antipsychotic medications, she was prescribed risperidone, which has been helpful.

Which of the following is the best next step? Change her antipsychotic medication Test for macroprolactinemia Measure IGF-1 Add bromocriptineOrder pituitary MRI Correct Answer: E Learning objective : Evaluate and treat hyperprolactinemia caused by antipsychotic medications.

Rationale: Thousands of patients take risperidone and other typical and atypical antipsychotic medications. Many of these drugs increase the serum prolactin concentration. The increase in prolactin is highest in patients taking medications with the greatest dopamine antagonist activity. Risperidone and paliperidone are second-generation antipsychotics that increase prolactin, usually to concentrations less than 200 ng/mL (<8.7 nmol /L). First-generation antipsychotics ( eg , haloperidol, fluphenazine) are also potent dopamine antagonists. Other atypical antipsychotic medications do not cause such high elevations. Hyperprolactinemia can cause menstrual disturbances, infertility, galactorrhea, gynecomastia, and decreased bone mineral density. Although changing her antipsychotic medication (Answer A) is certainly an option, it is not the best one. Because this patient had tried other antipsychotics and the risperidone agrees with her and is effective, maintaining this regimen would be best. It should be noted that another agent called aripiprazole functions as a D2-receptor antagonist but exhibits functional agonist properties under hypodopaminergic conditions. Thus, it lowers prolactin when added to the drug regimen that includes antipsychotic agents such as risperidone or haloperidol. Aripiprazole should only be considered after excluding the possibility of a prolactin-secreting pituitary tumor.

Rationale Continued: One must ensure that hyperprolactinemia in these patients is not a manifestation of an underlying pituitary tumor. A rule of thumb is that serum prolactin concentrations greater than 200 ng/mL (>8.7 nmol /L) are usually due to prolactin-secreting pituitary tumors. Therefore, in this patient with a serum prolactin concentration greater than 200 ng/mL, pituitary MRI (Answer E) is the best next step. It would be optimal to measure prolactin before and after starting antipsychotic medications; however, this is frequently not done. If it was known that her serum prolactin concentration was normal before she started her antipsychotic medications, then a pituitary MRI would not be needed. If macroprolactinemia (Answer B) accounted for the elevated prolactin (210 ng/mL [9.1 nmol /L]), oligomenorrhea would not have been expected because “big” and “big-big” prolactin are much less bioactive than the prolactin monomer. One usually measures IGF-1 (Answer C) to rule out cosecretion of GH from a pituitary tumor. These tumors cause acromegaly and the effects associated with elevated prolactin levels. However, the best way to reassure the patient that her hyperprolactinemia is not being caused by an underlying tumor is to obtain pituitary imaging (Answer E). Adding a dopamine agonist such as bromocriptine (Answer D) to the drug regimen can worsen psychosis, and this is not recommended.

Reference(s): Shim JC, Shin JG, Kelly DL, et al. Adjunctive treatment with a dopamine partial agonist, aripiprazole, for antipsychotic-induced hyperprolactinemia: a placebo-controlled trial. Am J Psychiatry . 2007;164(9):1404-1470. PMID: 17728426 Cookson J, Hodgson R, Wildgust HJ. Prolactin, hyperprolactinaemia and antipsychotic treatment: a review and lessons for treatment of early psychosis. J Psychopharmacol. 2012;26(Suppl 5):42-51. PMID: 22472310Inder WJ, Castle D. Antipsychotic-induced hyperprolactinaemia. Aust N Z J Psychiatry. 2011;45(10):830-837. PMID: 21714721Chahal J, Schlechte J. Hyperprolactinemia. Pituitary . 2008;11(2):141-146. PMID: 18404389

ITE 2017 Question 45 A 65-year-old woman presents with a 4-month history of fatigue, weight loss (25 lb [11.4 kg]), frequent headache, and nausea. She reports no visual symptoms, acral enlargement, increased thirst, or frequent urination. Her medical history is notable for dyslipidemia and knee osteoarthritis. Medications include atorvastatin and acetaminophen.On physical examination, her blood pressure is 105/60 mm Hg and pulse rate is 78 beats/min. Her height is 67 in (170.2 cm), and weight is 156 lb (70.9 kg) (BMI = 24.4 kg/m2). Visual fields are intact on perimetry . There are no clinical features suggestive of GH or cortisol excess. Laboratory test results: Sodium = 135 mEq /L (136-142 mEq /L) (SI: 135 mmol /L [136-142 mmol /L])Cortisol (8 AM) = 2.1 µg/ dL (5-25 μ g/ dL ) (SI: 57.9 nmol/L [137.9-689.7 nmol/L])Prolactin = 29 ng/mL (4-30 ng/mL) (SI: 1.3 nmol/L [0.17-1.30 nmol/L])TSH = 1.6 mIU/L (0.5-5.0 mIU/L)Free T4 = 0.6 ng/dL (0.8-1.8 ng/dL) (SI: 7.8 pmol/L [10.30-23.17 pmol/L])FSH = 1.0 mIU/mL (>30 mIU/mL) (SI: 1.0 IU/L [>30 IU/L])LH = 2.1 mIU/mL (>30 mIU/mL) (SI: 2.1 IU/L [>30 IU/L])

Brain MRI shows a cystic sellar mass displacing the optic chiasm (see image, left panel).

She begins hydrocortisone and levothyroxine replacement and undergoes transsphenoidal drainage of the lesion. Pathologic findings are consistent with Rathke cleft cyst with squamous metaplasia. Early postoperative MRI shows decompression and collapse of the cyst. Six months later, she develops recurrent headache. Follow- up MRI shows reexpansion of the cyst. She undergoes a second transsphenoidal procedure to drain the lesion. However, she develops recurrent headache and the cyst re-expands 3 months later. She undergoes a third transsphenoidal operation to drain the lesion, following which the cyst remains unchanged (see image, right panel)

Which of the following should you recommend now? Observation Lanreotide depot therapy Radiation therapy to the sella Craniotomy and cyst fenestrationCabergoline therapy Correct Answer: C Learning objective : Guide the management of a patient with recurrence of a Rathke cleft cyst.

Rationale: Rathke cleft cysts are cystic masses that arise from remnants of the Rathke pouch. Although many, if not most, are small and asymptomatic, some of these lesions can gradually expand and compress the normal pituitary gland, the optic apparatus, and other surrounding structures, leading to hypopituitarism, visual field deficits, and headache. Patients with symptomatic Rathke cleft cysts are generally treated with transsphenoidal surgery to drain the cyst and partly excise the cyst wall. In contrast, small, stable, asymptomatic Rathke cleft cysts can be observed with serial imaging. Recurrence of Rathke cleft cyst has been reported in approximately 12% to 20% of cases after surgery, and it appears to be more common in patients with squamous metaplasia on histologic examination. Repeated surgery has been performed successfully in some patients with recurrent Rathke cleft cyst. However, the patient described in this case had multiple recurrences despite 3 surgical procedures, suggesting that additional surgery (Answer D) would be unlikely to lead to lasting remission. Neither observation (Answer A) nor cabergoline therapy (Answer E) would be appropriate in the presence of mass effect in this patient, who did not have a prolactin-secreting adenoma and therefore would not be expected to respond to dopamine agonist therapy.

Rationale Continued: There is no evidence to suggest that somatostatin analogue therapy (including lanreotide depot [Answer B]) is effective in a patient with Rathke cleft cyst. Radiation therapy (Answer C) has been administered in some patients with recurrent Rathke cleft cyst with good effect and it is a reasonable treatment option in the present case. Patients receiving radiation therapy require lifelong periodic monitoring of their pituitary function to detect the development of anterior hypopituitarism, which is a common complication of radiotherapy. However, this potential complication is less relevant in this postmenopausal woman who already has anterior hypopituitarism before receiving radiation therapy.

Reference(s): Chotai S, Liu Y, Pan J, Qi S. Characteristics of Rathke’s cleft cysts based on cyst location with a primary focus on recurrence after resection. J Neurosurg. 2015;122(6):1380-1389. PMID: 25679272Mukherjee JJ, Islam N, Kaltsas G, et al. Clinical, radiological and pathological features of patients with Rathke’s cleft cysts: tumors that may recur. J Clin Endocrinol Metab. 1997;82(7):2357- 2362. PMID: 9215319Zada G, Lin N, Ojerholm E, Ramkissoon S, Laws ER. Craniopharyngioma and other cystic epithelial lesions of the sellar region: a review of clinical, imaging, and histopathological relationships. Neurosurg Focus . 2010;28(4):E4. PMID: 20367361 Trifanescu R, Stavrinides V, Plaha P et al. Outcome in surgically treated Rathke’s cleft cysts: long-term monitoring needed. Eur J Endocrinol. 2011;165(1):33-37. PMID: 21502327

ITE 2017 Question 54 You are called to the emergency department to advise on the management of a 25-year-old man who has been admitted to the hospital after he collapsed and had a seizure while running a marathon earlier in the day. He has no notable medical history and takes no prescribed medications. His seizure terminated after administration of intravenous lorazepam. On physical examination, he is confused and disoriented. His blood pressure is 98/65 mm Hg, and resting pulse rate is 100 beats/min. He has dry mucous membranes although skin turgor is normal. No overt focal neurologic signs are elicited. Laboratory test results: Sodium = 112 mEq /L (136-142 mEq /L) (SI: 112 mmol /L [136-142 mmol /L]) Potassium = 3.9 mEq /L (3.5-5.0 mEq /L) (SI: 3.9 mmol /L [3.5-5.0 mmol/L]) Serum urea nitrogen = 14 mg/ dL (8-23 mg/ dL ) (SI: 5.0 mmol/L [2.9-8.2 mmol/L])Creatinine = 1.2 mg/dL (0.7-1.3 mg/dL) (SI: 106.1 μmol/L [61.9-114.9 μmol/L])

Which of the following is the most appropriate treatment of this patient’s hyponatremia? 1.8% saline: 500 mL over 20 minutes 3.0% saline: 100 mL over 20 minutes 0.9% saline: 500 mL stat Intravenous conivaptanOral urea treatment Correct Answer: B Learning objective : Use current guidelines to safely manage acute, severe exercise-associated hyponatremia.

Rationale: It has been recognized for several decades that severe and potentially life-threatening hyponatremia can occur during exercise, particularly during endurance events. Most athletes who develop hyponatremia are asymptomatic or manifest only mild symptoms. However, occasionally, as in this case, more severe clinical features can occur such as seizures, cerebral edema, noncardiogenic pulmonary edema, and death. The incidence of exercise-associated hyponatremia has been studied in various groups of athletes. In the largest series of 2135 athletes who had participated in endurance events, 6% had a serum sodium concentration between 129 and 135 mEq/L (129-135 mmol /L) and 1% had a serum sodium concentration less than 129 mEq /L (<129 mmol /L). In a prospective study of 488 athletes who completed the 2002 Boston Marathon, 13% had hyponatremia (serum sodium <135 mEq /L [<135 mmol /L]) and 0.6% (3 runners) had severe hyponatremia (serum sodium <120 mEq /L [<120 mmol /L]).

Rationale Continued: Increased fluid intake is the major contributor to exercise-associated hyponatremia. However, if renal function is normal, then water loading alone is not likely to lead to more than very mild hyponatremia since water loading should suppress antidiuretic hormone release leading to high output of maximally dilute urine. There is evidence that endurance athletes who develop severe hyponatremia have submaximal suppression of antidiuretic hormone leading to an inappropriately high urine osmolality; as such, they fulfill the criteria for syndrome of inappropriate antidiuretic hormone secretion. This vignette describes a case of life-threatening hyponatremia in an otherwise healthy man who has been participating in a marathon. The diagnosis of exercise-induced hyponatremia can be made when hyponatremia develops within 24 hours of an episode of intense exercise. It is reasonable to assume that this athlete had a previously normal serum sodium level and that the severe hyponatremia is acute. The major clinical concern here is cerebral edema, which can develop in acute hyponatremia due to an inability of the brain to adapt to an altered extracellular osmolality. In this situation, a small, quick increase in the serum sodium (2-4 mEq /L [2-4 mmol /L]) is effective in treating the effects of acute hyponatremia because it is estimated that a 5 mEq /L rise in serum sodium level reduces intracranial pressure by up to 50%.

Rationale Continued: There is always justifiable concern over the rate of correction of hyponatremia given potential risks of osmotic demyelination syndrome with overly rapid correction. However, this is far less of a concern in the context of exercise-associated hyponatremia, which is a very acute event not associated with cerebral adaptations that contribute to the risk of overly rapid correction of chronic hyponatremia. It is recommended that serum sodium be increased swiftly without harm if hyponatremia has developed within 24 hours. To date, there are no reported cases of osmotic demyelination in treatment of exercise-associated hyponatremia. Therefore, largely due to evidence gathered from acute, prompt treatment of severe hyponatremia either in an athletic or neurosurgical setting, both a recent US expert panel and European consensus statement recommend that the correct treatment is prompt infusion of 100 to 150 mL of hypertonic (3%) saline over 20 minutes (Answer B). This can be repeated twice if needed. This recommendation is consistent with the management recommended by the 2007 Second International Exercise-Associated Hyponatremia Consensus Development Conference. The rationale is to quickly raise serum sodium by infusing saline but not to increase total body water in an individual who is already fluid replete. Therefore, infusion of larger volumes of salt-containing fluid (Answers A and C) is incorrect. Indeed, in one small study of hyponatraemia associated with syndrome of inappropriate antidiuretic hormone secretion, infusion of 0.9% saline either made no difference in serum sodium or decreased it further.

Rationale Continued: Conivaptan (Answer D) is an intravenous vasopressin receptor antagonist that is approved by the FDA for the short-term management of euvolemic and hypervolemic hyponatremia. In practice, its use is generally limited to chronic cases of the syndrome of inappropriate antidiuretic hormone secretion (which is not seen in exercise- induced hyponatremia) and there are no data supporting its efficacy and safety in exercise-associated hyponatremia. Overall, the use of vasopressin receptor antagonists in hyponatremia management remains controversial. Oral urea treatment (Answer E) has been (controversially) recommended by the European Clinical Practice Guideline as second-line management of hyponatremia in the context of syndrome of inappropriate antidiuretic hormone secretion. This can be effective in increasing oral solute intake and thus enhancing renal free water clearance. However, its availability and tolerability make this a fairly impractical solution. Its use is not recommended in this case of exercise-induced hyponatremia where the oral route is compromised and serum sodium must rise quickly.

Reference(s): Spasovski G, Vanholder R, Allolio B, et al; Hyponatraemia Guideline Development Group. Clinical practice guideline on diagnosis and treatment of hyponatraemia [published correction appears in Nephrol Dial Transplant. 2014;40(6):924]. Neprol Dial Transplant. 2014;29(Suppl 2):i1-i39. PMID: 24569496Verbalis JG, Goldsmith SR, Greenberg A, et al. Diagnosis, evaluation, and treatment of hyponatremia: expert panel recommendations. Am J Med. 2013;126(10 Suppl 1):S1-S42. PMID: 24074529Urso C, Brucculeri S,  Caimi G. Physiopathological , epidemiological, clinical and therapeutic aspects of exercise-associated hyponatremia. J Clin Med . 2014;3(4):1258-1275. PMID: 26237602

ITE 2017 Question 60 A 67-year-old woman is found to have hyperthyroidism with a small goiter. Thyroid function results: Free T4 concentration = 2.8 ng/ dL (0.8-1.8 ng/dL) (SI: 30.0 pmol /L [10.30-23.17 pmol /L]) TSH = 5.9 mIU /L (0.5-5.0 mIU /L) A TSH-secreting pituitary adenoma is suspected, and MRI shows a 2.4-cm tumor with cavernous sinus invasion. After transsphenoidal surgery, the patient’s free T4 level remains elevated at 1.8 ng/ dL (23.2 pmol/L), and her TSH concentration is 2.7 mIU /L. MRI shows persistent tumor in the cavernous sinus.

Which of the following medical treatments would be expected to give the best overall results in this case? Cabergoline Methimazole Radioactive iodine Lanreotide depot Temozolomide Correct Answer: D Learning objective : Describe the regulation of TSH secretion and apply that physiology to the treatment of TSH-secreting pituitary tumors ( TSHomas ).

Rationale: One must remember the basic physiology and regulation of pituitary hormones. Somatostatin inhibits both GH and TSH, and somatostatin analogues can inhibit secretion from TSH-secreting pituitary tumors, as well as decrease their size. Thus, lanreotide depot (Answer D) would be expected to give the best results in this case because it is generally successful at normalizing thyroid hormone levels and can reduce the size of the tumor, which is invading the cavernous sinus in this patient. Both methimazole (Answer B) and radioactive iodine (Answer C) can decrease thyroid hormone levels and treat hyperthyroidism, but they will not decrease tumor size. Neither cabergoline (Answer A) nor temozolomide (Answer E) effectively treats TSH-secreting pituitary tumors.

Reference(s): Beck- Peccoz P, Persani L, Mannavola D, Campi I. TSH-secreting adenomas. Best Pract Res Clin Endocrinol Metab. 2009;23(5):597-606. PMID: 19945025Amlashi FG, Tritos NA. Thyrotropin-secreting pituitary adenomas: epidemiology, diagnosis, and management. Endocrine. 2016;52(3):427-440. PMID: 26792794

ITE 2017 Question 72 A 22-year-old man is brought to your office by his parents, who are convinced that he has Cushing syndrome. Over the past 2 years, they have watched him become increasingly withdrawn and have witnessed the evolution of disturbing physical changes, including 45-lb (20.5-kg) weight gain despite dieting and exercise. During the last several months, he has developed hypertension. They have taken their son to many physicians for evaluation, all of whom have said that there is no hormonal abnormality. On physical examination, his blood pressure is 140/92 mm Hg and he has a small dorsocervical fat pad, acne, and a ruddy complexion. Laboratory test results: ACTH = 40 pg /mL (10-60 pg /mL) (SI: 8.8 pmol /L [2.2-13.2 pmol /L]) Serum cortisol (8 AM) = 18 μg / dL (5-25 μg/ dL ) (SI: 496.6 nmol /L [137.9-689.7 nmol /L])Urinary cortisol = 60 μg/24 h (4-50 μg/24 h) (SI: 165.6 nmol/d [11-138 nmol/d])

Which of the following should be performed next? Bilateral adrenal vein sampling Inferior petrosal sinus sampling High-dose dexamethasone suppression test Salivary cortisol measured at midnight on 3 consecutive nights Pituitary MRI Correct Answer: D Learning objective : Select the initial diagnostic tests that should be used to establish a diagnosis of Cushing syndrome.

Rationale: The patient has a urinary cortisol level slightly above the upper limit of the reference range (50 μg /24 h). The findings of measurable ACTH and elevated cortisol suggest that the patient may have ACTH-dependent Cushing syndrome. However, the data are not adequate to confirm Cushing syndrome. A second, confirmatory test is needed to document the presence of cortisol excess, which may include salivary cortisol at midnight (Answer D). If the cortisol level from midnight sampling is high, the diagnosis of pseudo-Cushing syndrome is unlikely. The diagnosis of Cushing syndrome must be firmly established before characterizing the etiology of the disease. Thus, bilateral adrenal vein sampling (Answer A), inferior petrosal sinus sampling (Answer B), and a pituitary MRI (Answer E) should wait until a diagnosis of Cushing syndrome is established. In fact, results of petrosal sinus sampling should not be and cannot be interpreted unless documented hypercortisolism is present. Although this patient has a morning serum cortisol level within the reference range, it does not rule out a diagnosis of Cushing syndrome. Normal morning serum cortisol levels can occur in up to 50% of patients with Cushing syndrome.

Rationale Continued: Although a number of diagnostic steps might be prudent, a frankly elevated midnight cortisol—either in serum or saliva—is highly indicative of true Cushing syndrome. The normal evening nadir in serum or salivary cortisol is preserved in obese and depressed patients, but not in those with Cushing syndrome. The nighttime saliva collection approach is convenient for patients, but it is best when at least 2 consecutive samples are collected. Results from this test can also be helpful in uncovering Cushing syndrome of a cyclical nature, which is rare. Frank elevation of nighttime serum or salivary cortisol levels has a high sensitivity and specificity for diagnosing Cushing syndrome. Finally, although the high-dose dexamethasone suppression test (Answer C) may be helpful in evaluation of the underlying cause, it should not be used to establish the diagnosis of Cushing syndrome.

Reference(s): Nieman LK, Biller BM, Findling JW, et al. The diagnosis of Cushing's syndrome: an Endocrine Society clinical practice guideline. J Clin Endocrinol Metab. 2008;93(5):1526-1540. PMID: 18334580Newell-Price J. Diagnosis/differential diagnosis of Cushing's syndrome: a review of best practice. Best Pract Res Clin Endocrinol Metab. 2009;23(Suppl 1):S5-S14. PMID: 20129193Boscaro M, Arnaldi G. Approach to the patient with possible Cushing’s syndrome. J Clin Endocrinol Metab . 2009;94(9):3121-3131. PMID: 19734443 Nieman LK. Cushing’s syndrome: update on signs, symptoms and biochemical screening. Eur J Endocrinol . 2015:173(4):M33-M38. PMID: 26156970Raff H. Cushing syndrome: update on testing. Endocrinol Metab Clin North Am. 2015:44(1):43-50. PMID: 25732641

ITE 2017 Question 81 A 60-year-old man presents with a several-week history of polyuria and polydipsia. He reports no headache, but he does have double vision. He previously smoked 20 cigarettes per day for 30 years, but stopped 3 years ago. He has no notable medical history and takes no medications. On physical examination, he appears thin and pale. His skin is dry. His height is 69 in (175.3 cm), and weight is 150 lb (68.2 kg) (BMI = 22.1 kg/m2). His blood pressure is 114/68 mm Hg, and resting pulse rate is 52 beats/min. Examination of his cranial nerves reveals a right VI nerve palsy.

A water-deprivation test confirms central diabetes insipidus. Thus, further investigation of pituitary function is ordered: Cortisol (8 AM) = 24 μg / dL (5-25 μg/dL) (SI: 662.1 nmol /L [137.9-689.7 nmol /L]) TSH = 0.38 mIU /L (0.5-5.0 mIU /L) Free T4 = 0.4 ng/ dL (0.8-1.8 ng/ dL ) (SI: 5.1 pmol/L [10.30-23.17 pmol/L]) FSH = 0.2 mIU /mL (1.0-13.0 mIU /mL) (SI: 0.2 IU/L [1.0-13.0 IU/L]) LH = 0.4 mIU/mL (1.0-9.0 mIU/mL) (SI: 0.4 IU/L [1.0-9.0 IU/L])Free testosterone = 4.0 ng/dL (9.0-30.0 ng/dL) (SI: 0.1 nmol/L [0.31-1.04 nmol/L])Prolactin = 18.0 ng/mL (4.0-23.0 ng/mL) (SI: 0.8 nmol/L [0.17-1.00 nmol/L])IGF-1 = 62.0 ng/mL (78-220 ng/mL) (SI: 8.1 nmol/L [10.2-28.8 nmol/L])Pituitary MRI is performed (see image).

Sagittal view Coronal view

Which of the following is the most likely cause of pituitary pathology in this patient? Pituitary macroadenoma Craniopharyngioma Meningioma Pituitary metastasis Rathke cleft cyst Correct Answer: D Learning objective : Diagnose metastasis to the pituitary gland on the basis of MRI characteristics and clinical findings.

Rationale: The combination of a pituitary mass and a new presentation of diabetes insipidus is most suggestive of a metastatic deposit to the pituitary (Answer D). The pituitary gland is an uncommon site for metastasis, which accounts for 1% to 2% of sellar masses. Although neoplasms from almost every tissue have been reported to metastasize to the pituitary, the most common primary tumor is breast or lung cancer. Given this man’s smoking history, lung cancer would seem the most likely underlying diagnosis (and indeed this was subsequently proven). Metastatic deposits demonstrate a predilection for the posterior lobe of the pituitary. In one review of more than 200 cases, the posterior lobe was involved alone or with the anterior pituitary in 85% of cases, whereas only the anterior lobe was affected in 15%. Therefore, metastatic deposits in the anterior lobe are usually due to contiguous spread from the posterior lobe. As a result of predominant posterior lobe involvement, diabetes insipidus is the most common presenting feature of pituitary metastases. However, pituitary metastases are usually asymptomatic. In the aforementioned review, 40 of 200 patients (20%) were symptomatic, and, of these, diabetes insipidus was documented in 28 patients (70%) while only 6 patients (15%) had 1 or more anterior pituitary deficiencies.

Rationale Continued: Sagittal view Coronal view The pituitary MRI in this case demonstrates a homogenous sellar and suprasellar mass (white arrows) with thickening of the pituitary stalk (black arrow with white outline), which is consistent with a metastatic deposit.

Rationale Continued: Although a benign macroadenoma (Answer A) is more common in general, the presence of diabetes insipidus in this case makes metastasis much more likely. A craniopharyngioma (Answer B) usually exhibits cystic and solid components on T1-weighted imaging, so while the clinical picture could fit with this diagnosis, the pituitary MRI does not. A meningioma (Answer C) is a slow-growing, usually benign, tumor that can arise from the dural surface around the sella . The typical radiologic feature is the dural “tail of enhancement” that extends forward along the planum sphenoidale and is absent in this pituitary MRI. Finally, a Rathke cleft cyst (Answer E) arises close to the insertion of the pituitary stalk, looks hyperintense on T1-weighted MRI, and usually does not demonstrate any solid component.

Reference(s): Komninos J, Vlassopoulou V, Protopapa D, et al. Tumors metastatic to the pituitary gland: case report and literature review. J Clin Endocrinol Metab. 2004;89(2):574-580. PMID: 14764764McCormick PC, Post KD, Kandji AD, Hays AP. Metastatic carcinoma to the pituitary gland. Br J Neurosurg. 1989;3(1):71-79. PMID: 2789715He W, Chen F, Dalm B, Kirby PA, Greenlee JD. Metastatic involvement of the pituitary gland: a systematic review with pooled individual patient data analysis. Pituitary. 2015:18(1):1589-1568. PMID: 24445565

ITE 2017 Question 83 A 29-year-old pregnant woman with partial diabetes insipidus from childhood head trauma is brought to the emergency department for generalized weakness and lethargy. She is 4 months pregnant. Because her diabetes insipidus was mild, she has always preferred to drink extra fluids rather than take medication. She also has Hashimoto thyroiditis and has been taking levothyroxine, 112 mcg daily. Before conceiving, she always had regular menses. Aside from this recent weakness and lethargy, she has been well. However, for the past 2 months she has noticed increasing polyuria and polydipsia with some nausea. On physical examination in the emergency department, the patient is semiobtunded with a blood pressure of 80/50 mm Hg and a pulse of 130 beats/min. She has dry mucous membranes with skin tenting. Laboratory test results: Serum sodium = 162 mEq /L (136-142 mEq /L) (SI: 162 mmol /L [136-142 mmol /L]) Urine specific gravity = 1.002

Which of the following is the most likely explanation for this woman’s clinical deterioration? Sheehan syndrome Failure to increase her thyroid hormone replacement appropriately for pregnancy Pregnancy-induced exacerbation of subclinical anterior hypopituitarism Lymphocytic hypophysitis Increased metabolism of vasopressin by placental vasopressinase Correct Answer: E Learning objective : Manage diabetes insipidus during pregnancy.

Rationale: Patients with partial diabetes insipidus can be relatively asymptomatic and compensate by increasing water intake. However, during pregnancy, placental vasopressinase can cause a greatly increased metabolic degradation of vasopressin, making manifest a previously subclinical process (thus, Answer E is correct). Impaired thirst mechanism or associated illness (such as the likely viral infection in this vignette), trauma, or surgery that affect access to water intake are usual precipitating factors for hypernatremia in patients with diabetes insipidus. Sheehan syndrome (Answer A) denotes the presence of postpartum pituitary infarction as a result of excessive peripartum hemorrhage and is not applicable in this case. During pregnancy, levothyroxine requirements commonly increase (Answer B), but this usually does not cause symptoms, and, if there were symptoms, they would be those of mild hypothyroidism rather than those of profound dehydration. Furthermore, hypothyroidism is associated with hyponatremia, not hypernatremia. Lymphocytic hypophysitis of pregnancy (Answer D) is usually associated with anterior pituitary dysfunction, and the development of ACTH/cortisol deficiency would most likely ameliorate rather than exacerbate any underlying diabetes insipidus. Lymphocytic hypophysitis can develop in the third trimester or in the early postpartum period. This patient was previously well, with normal menses. There is no reason to suspect she has anterior hypopituitarism (Answer C).

Reference(s): Aleksandrov N, Audibert F, Bedard M-J, et al. Gestational diabetes insipidus: a review of an underdiagnosed condition. J Obstet Gynaecol Can. 2010;32:225-231. PMID: 20500966Wallia A, Bizhanova A, Huang W, Goldsmith SL, Gossett DR, Kopp P. Acute diabetes insipidus mediated by vasopressinase after placental abruption. J Clin Endocrinol Metab. 2013;98(3):881-886. PMID: 23393172

ITE 2017 Question 90 A 29-year-old man is referred because a head CT performed in the emergency department after an automobile crash showed an empty sella , which was confirmed on a subsequent MRI examination of the pituitary. He was generally well before the accident. He states that he received GH injections for many years as a child and stopped when he completed growth at age 18 years. He has also taken thyroid hormone since age 12 years.

Which of the following is the most likely cause of his empty sella ? Trauma-induced pituitary infarction PROP1 mutation Burnt-out hypothalamic/pituitary sarcoidosisLangerhans cell histiocytosis Hemochromatosis Correct Answer: B Learning objective : Construct a differential diagnosis for an empty sella .

Rationale: Mutations in the genes encoding a number of transcription factors, including POU1F1 (formerly PIT1) and PROP1, can cause disruption in the development of many pituitary cell types during embryogenesis, resulting in multiple pituitary hormone deficiencies. Mutations in the PROP1 gene (Answer B) result in a decrease in PROP 1, a transcription factor important for the development of the somatotroph , lactotroph, and thyrotroph lineages with deficiencies of their respective hormones. Some affected individuals also have delayed puberty. Combined pituitary hormone deficiency (GH, prolactin, TSH) has an incidence of about 1 in 8000 births, and 10% have an affected family member. Between 25% and 50% of these cases are due to POU1F1 or PROP1 mutations. In this patient, the history of previous GH and thyroid hormone treatment starting in childhood suggests a congenital combined pituitary hormone deficiency. In some children with PROP 1 deficiency, there is an early pituitary enlargement of uncertain cause, which results in sellar enlargement and subsequent loss of pituitary volume, leading to an empty sella . In most series, when patients with empty sella have evaluation of pituitary function, between one-quarter and one-third have varying degrees of hypopituitarism.

Rationale Continued: Acute trauma causing pituitary infarction (Answer A) would show blood in the pituitary with pituitary enlargement rather than a small pituitary. Sarcoidosis (Answer C) and Langerhans cell histiocytosis (Answer D) are infiltrative diseases of the hypothalamus and pituitary stalk and might present with stalk thickening rather than an empty sella . Both usually also present with diabetes insipidus. Hemochromatosis (Answer E) can cause iron deposition in the pituitary and usually affects the gonadotroph cells; it is not associated with an empty sella.

Reference(s): Prince KL, Walvoord EC, Rhodes SJ. The role of homeodomain transcription factors in heritable pituitary disease. Nat Rev Endocrinol . 2011;7(12):727-737. PMID: 21788968Mendonca BB, Osorio MG, Latronico AC, Estefan V, Lo LS, Arnhold IJ. Longitudinal hormonal and pituitary imaging changes in two females with combined pituitary hormone deficiency due to deletion of A301,G302 in the PROP1 gene. J Clin Endocrinol Metab. 1999;84(3):942-945. PMID: 10084575Guitelman M, Garcia Basavilbaso N, Vitale M, et al. Primary empty sella (PES): a review of 175 cases. Pituitary. 2013;16(2):270-274. PMID: 22875743 Castinetti F, Reynaud R, Saveanu A, Barlier A, Brue T. Genetic causes of combined pituitary hormone deficiencies in humans. Ann Endocrinol (Paris). 2012;73(2):53-55. PMID: 22521856

THYROID

ITE 2017 Question 6 A 55-year-old woman with bipolar disorder is prescribed lithium therapy. Before starting therapy, she is noted to have a small, firm goiter. Her family history is notable for a sister with Graves disease. Laboratory test results: Serum TSH = 2.9 mIU /L (0.5-5.0 mIU/L) Free T4 = 1.0 ng/ dL (0.8-1.8 ng/ dL ) (SI: 12.9 pmol /L [10.30-23.17 pmol /L]

In this patient, which of the following is the best predictor of thyroid dysfunction over the next 3 months? Thyroid nodules on ultrasonography Positive family history of thyroid disease Serum thyroglobulin level Positive TPO antibodies Elevated serum lithium levels Correct Answer: D Learning objective : List predictors of thyroid dysfunction in patients taking lithium.

Rationale: Lithium prevents the release of thyroid hormone by interfering with colloid droplet formation within thyroid follicular cells due to decreased pinocytosis of colloid from the follicular lumen. Patients with underlying compromised thyroid reserve, such as this patient with underlying Hashimoto thyroiditis, are at a higher risk of developing hypothyroidism in response to lithium therapy. Therefore, screening for TPO antibodies (Answer D) is useful before starting lithium therapy. The presence of thyroid nodules on ultrasonography (Answer A) does not increase the risk of lithium-induced thyroid dysfunction. A positive family history of thyroid disease (Answer B), especially autoimmune thyroid disease, could increase a patient’s risk of developing lithium-induced thyroid dysfunction, but this is not as useful as the demonstration of positive TPO antibodies. Serum thyroglobulin (Answer C) is not a clinically useful predictor of thyroid dysfunction in this setting. Lithium levels (Answer E) are not related to the likelihood of thyroid dysfunction.

Reference(s): Lazarus JH. Lithium and thyroid. Best Pract Clin Endocrinol Metab. 2009;23(6):723-733. PMID: 19942149

ITE 2017 Question 14 Which of the following is the most likely explanation?

Correct Answer: A Learning objective : Identify ultrasound patterns characteristic of malignant nodules.

Rationale: On the basis of sonographic patterns, thyroid nodules can be classified as high, intermediate, low, or very low suspicion or benign. Each sonographic pattern confers a risk of malignancy. The nodule in Figure B is sonographically benign. It is a simple cyst—a hypoechoic, thin-walled structure with “enhancement,” or an increased echo pattern behind it. The nodule in Figure C has a classic spongiform appearance, which is classified as a very low-suspicion sonographic pattern (very low risk for malignancy). The nodule in Figure D is hyperechoic relative to the surrounding thyroid parenchyma, and it has an intact halo, making it most likely a benign lesion. Low-suspicion nodules such as this one are estimated to have a 5% to 10% risk of malignancy. The nodule shown in Figure A demonstrates a high-suspicion sonographic pattern, which is associated with a 70% to 90% risk of malignancy (or higher). This nodule is hypoechoic, has somewhat irregular borders, and it also has the “taller than wide” feature in the transverse view shown, which is an independent predictor of malignancy.

Reference(s): Brito JP, Gionfriddo MR, Al Nofal A, et al. The accuracy of thyroid nodule ultrasound to predict thyroid cancer: systematic review and meta-analysis. J Clin Endocrinol Metab. 2014;99(4):1253-1263. PMID: 24276450Haugen BR, Alexander EK, Bible KC, et al. 2015 American Thyroid Association Management Guidelines for Adult Patients with Thyroid Nodules and Differentiated Thyroid Cancer: The American Thyroid Association Guidelines Task Force on Thyroid Nodules and Differentiated Thyroid Cancer. Thyroid. 2016;26(1):1-133. PMID: 26462967

ITE 2017 Question 19 A 68-year-old man with Graves disease is treated with radioiodine therapy. Two months later, his free T4 concentration is 0.65 ng/ dL (0.8-1.8 ng/ dL [SI: 8.4 pmol/L (10.30-23.17 pmol/L)]) and his TSH concentration is 5.2 mIU /L (0.5-5.0 mIU /L). Levothyroxine is initiated at a dosage of 50 mcg daily. The following month, the patient notes recurrent palpitations, and the following laboratory test results are documented: TSH = <0.01 mIU /L Free T4 = 3.1 ng/ dL (0.8-1.8 ng/ dL ) (SI: 39.9 pmol /L [10.30-23.17 pmol /L]) Total T3 = 350 ng/ dL (70-200 ng/ dL ) (SI: 5.4 nmol/L [1.08-3.08 nmol/L])Thyroid hormone therapy is stopped and 2 weeks later, his free T4 and total T3 levels remain elevated.

Which of the following is the best next step in this patient’s management? Continued observation Repeat treatment with radioiodine now Initiation of prednisone Reinitiation of levothyroxine at a reduced dosageInitiation of methimazole Correct Answer: E Learning objective : Recommend an approach to managing recurrent hyperthyroidism following radioactive iodine therapy.

Rationale: This patient appears to have had transient hypothyroidism shortly after receiving radioiodine therapy for Graves disease, and a low dosage of thyroid hormone was initiated. Shortly thereafter, he was noted to have recurrent thyrotoxicosis, well beyond what would be expected due to his levothyroxine dosage. After radioiodine therapy, there is a sustained increase in thyroid-stimulating immunoglobulin titers, and patients with sufficient residual thyroid function may manifest hyperthyroidism for weeks or months after administration of the radioiodine, before it has had a full effect (generally, patients are given 6 months before retreatment with radioiodine is considered [thus, Answer B is incorrect]). This is distinct from radiation thyroiditis, which generally manifests in the first 2 weeks after radioiodine and does not last for months, as in the current case. In the interim, while awaiting the full effect from the radioiodine, this patient should be treated with methimazole (Answer E). Continued observation (Answer A) is not appropriate in this 68-year-old patient. Clearly, levothyroxine therapy (Answer D) would be contraindicated and there is no role for glucocorticoid therapy (Answer C) in this situation.

Reference(s): Stensvold AD, Jorde R, Sundsfjord J. Late and transient increases in free T4 after radioiodine treatment for Graves' disease. J Endocrinol Invest. 1997;20(10):580-584. PMID: 9438914Ross DS, Burch HB, Cooper DS, et al. 2016 American Thyroid Association guidelines for diagnosis and management of hyperthyroidism and other causes of thyrotoxicosis. Thyroid. 2016 [Epub ahead of print] PMID: 27521067

ITE 2017 Question 27 A 63-year-old man is being treated for a nonthyroid cancer. Laboratory test results before treatment initiation: TSH = 2.14 mIU/L (0.5-5.0 mIU /L) Free T4 = 1.3 ng/ dL (0.8-1.8 ng/ dL ) (SI: 16.3 pmol /L [10.30-23.17 pmol /L]) Ten weeks after starting the cancer therapy, he reports fatigue, constipation, and cold intolerance. Results from repeated thyroid function tests document a TSH concentration of 42 mIU /L and a free T4 concentration of 0.5 ng/ dL (6.4 pmol /L).

Which of the following medications is the most likely cause of his thyroid dysfunction? Sunitinib Denileukin diftitox Bexarotene IpilimumabTamoxifen Correct Answer: A Learning objective : Explain the thyroidal effects of drugs used for cancer treatment.

Rationale: Sunitinib (Answer A) and other tyrosine kinase inhibitors may cause de novo primary hypothyroidism or worsen preexisting primary hypothyroidism. In patients treated with tyrosine kinase inhibitors who did not have preexisting thyroid dysfunction, overt hypothyroidism has been reported in 32% to 85%, and subclinical hypothyroidism has been reported in up to 100%. Transient hyperthyroidism has also been reported to occur in up to 24% of patients treated with tyrosine kinase inhibitors. Bexarotene (Answer C), which is rarely used for the treatment of late-stage cutaneous T-cell lymphoma, has been associated with severe central hypothyroidism in up to 70% of patients. Effects are reversible with discontinuation of treatment. Other pituitary function is not affected. Denileukin diftitox (Answer B) may be associated with thyroiditis. If this patient had recently started denileukin diftitox , thyrotoxicosis rather than hypothyroidism would be expected. Ipilimumab (Answer D), an immune-modulating monoclonal antibody directed against CTLA-4, is used in patients with advanced melanoma. It has been associated with the development of irreversible central hypothyroidism in up to 17% of patients. The selective estrogen receptor modulator tamoxifen (Answer E), which is used as an adjuvant therapy in breast cancer treatment, can cause elevation in thyroid-binding globulin, but it has not been associated with thyroid dysfunction.

Reference(s): Graeppi-Dulac J, Vlaeminck-Guillem V, Perier-Muzet M, Dalle S, Orgiazzi J. Endocrine side-effects of anti-cancer drugs: the impact of retinoids on the thyroid axis. Eur J Endocrinol. 2014;170(6):R253-R262. PMID: 24616413Torino F, Barnabei A, Paragliola R, Baldelli R, Appetecchia M, Corsello SM. Thyroid dysfunction as an unintended side effect of anticancer drugs. Thyroid. 2013;23(11):1345-1366. PMID: 23750887

ITE 2017 Question 32 A 24-year-old man is noted to have a goiter and is referred for further evaluation. The patient’s medical history is otherwise unremarkable. On physical examination, his pulse rate is 90 beats/min. He has a diffuse goiter, twice normal size, without nodule or bruit. Deep tendon reflexes are normal and he has no tremor. Laboratory test results: Free T 4 = 2.7 ng/dL (0.8-1.8 ng/ dL ) (SI: 34.8 pmol /L [10.3-23.2 pmol /L]) Total T 3 = 300 ng/ dL (70-200 ng/ dL ) (SI: 4.6 nmol/L [1.1-3.1 nmol/L]) TSH = 3.8 mIU /L (0.5-5.0 mIU /L) α-subunit = 0.3 ng/mL (<1.2 ng/mL) (SI: 0.3 µg/L [<1.2 µg/L])MRI of the pituitary is normal.

Which of the following is the most likely explanation? Thyroid hormone resistance A TSH-producing pituitary adenoma Heterophilic antibody interference with the TSH assay T 4 antibody interference with the free T4 assay Surreptitious use of thyroid hormone Correct Answer: A Learning objective : Recognize clinical features of thyroid hormone resistance.

Rationale: This patient has discordant TSH and free T 4 values, suggesting inappropriate TSH release. The α-subunit-to-TSH molar ratio less than 1.0 favors thyroid hormone resistance (Answer A) rather than a TSH-producing pituitary tumor (Answer B) (pituitary MRI is also normal and most TSH-producing tumors are macroadenomas ). The prevalence of thyroid hormone resistance is about 1 in 40,000 live births. Another possible cause is heterophilic antibodies (Answer C), but serial dilutions would not be expected to yield linear results if these were present. T 4 antibodies (Answer D) tend to increase the measured value of free T 4 . Surreptitious use of thyroid hormone (Answer E) is unlikely to give the consistent pattern described in this patient.

Reference(s): Ortiga-Carvalho TM, Sidhaye AR, Wondisford FE. Thyroid hormone receptors and resistance to thyroid hormone disorders. Nat Rev Endocrinol. 2014;10(10):582-591. PMID: 25135573Macchia E, Lombardi M, Raffaelli V, et al. Clinical and genetic characteristics of a large monocentric series of patients affected by thyroid hormone (Th) resistance and suggestions for differential diagnosis in patients without mutation of Th receptor β. Clin Endocrinol (Oxf). 2014;81(6):921-928. PMID: 25040256

ITE 2017 Question 43 A 28-year-old man undergoes thyroidectomy with central neck dissection for a 2.5-cm papillary thyroid cancer. The tumor shows microscopic local invasion but no aggressive histology and 1 of 12 central lymph nodes contains tumor. The patient undergoes radioiodine remnant ablation using 100 mCi of 131 I, and the posttreatment scan shows no uptake outside of the thyroid bed. Surveillance testing at 6 months reveals a stimulated thyroglobulin concentration of 15 ng/mL (15 µg/L) with negative thyroglobulin antibodies, no abnormal uptake on radioiodine whole-body scan, and no adenopathy on neck ultrasonography. No additional therapy is given. Six months later, a suppressed thyroglobulin level is 0.5 ng/mL (0.5 µg/L), stimulated thyroglobulin is 5 ng/mL (5 µg/L), thyroglobulin antibodies are negative, and neck ultrasonography is unchanged.

Which of the following should be the next step in this patient’s management? Thyroglobulin testing using a different assay PET-CT scan CT of the chest MRI of the neck Repeated surveillance testing in 1 year Correct Answer: E Learning objective : Recommend an appropriate surveillance strategy for differentiated thyroid cancer.

Rationale: This patient has microscopic local invasion of tumor and positive central compartment lymph node disease. According to the AJCC-7 staging system (American Joint Committee on Cancer, 7th edition), his tumor is classified as T3,N1a,M0, stage I. After surgery and radioiodine therapy, the patient has persistent elevation of serum thyroglobulin, but this is down-trending over time. According to a recently recommended restratification of risk on the basis of response to initial therapy, he has had an acceptable response to therapy, with an unstimulated thyroglobulin concentration less than 1.0 ng/mL (<1.0 µg/L) and stimulated thyroglobulin concentration less than 10 ng/mL (<10 µg/L). Numerous studies have demonstrated progressive spontaneous decreases in thyroglobulin over years after initial therapy. The best option in this patient is to continue to monitor without intervention or additional unnecessary testing (thus, Answer E is correct and Answers A, B, C, and D are incorrect).

Reference(s): Al- Saif O, Farrar WB, Bloomston M, Porter K, Ringel MD, Kloos RT. Long-term efficacy of lymph node reoperation for persistent papillary thyroid cancer. J Clin Endocrinol Metab. 2010;95(5):2187-2194. PMID: 20332244Haugen BR, Alexander EK, Bible KC, et al. 2015 American Thyroid Association Management Guidelines for Adult Patients with Thyroid Nodules and Differentiated Thyroid Cancer: The American Thyroid Association Guidelines Task Force on Thyroid Nodules and Differentiated Thyroid Cancer. Thyroid. 2016;26(1):1-133. PMID: 26462967Padovani RP, Robenshtok E, Brokhin M, Tuttle RM. Even without additional therapy, serum thyroglobulin concentrations often decline for years after total thyroidectomy and radioactive remnant ablation in patients with differentiated thyroid cancer. Thyroid . 2012:22(8):778-783. PMID: 22780333 Tuttle RM, Tala H, Shah J, et al. Estimating risk of recurrence in differentiated thyroid cancer after total thyroidectomy and radioactive iodine remnant ablation: using response to therapy variables to modify the initial risk estimates predicted by the new American Thyroid Association staging system. Thyroid . 2010;20(12):1341-1349. PMID: 21034228

ITE 2017 Question 59 A 28-year-old woman with Graves disease in the 18th week of pregnancy and on methimazole is referred for medication adjustment.

Which of the following sets of laboratory results is within recommended targets for this patient? Correct Answer: B Learning objective : To understand goals of therapy for Graves disease in pregnancy.   TSH Free T 4 Total T 3 A. 0.05 mIU /L 2.1 ng/ dL (27.0 pmol /L) 400 ng/dL (6.2 nmol/L) B. 0.1 mIU/L 1.8 ng/ dL(23.2 pmol/L)350 ng/dL(5.4 nmol/L)C.1.5 mIU/L1.5 ng/dL(19.3 pmol/L)330 ng/dL(5.1 nmol/L)D.2.5 mIU/L1.3 ng/dL(16.7 pmol/L)280 ng/dL(4.3 nmol/L)E.3.5 mIU/L1.0 ng/dL(12.9 pmol/L)210 ng/dL(3.2 nmol/L)Reference ranges: TSH, 0.5-5.0 mIU/L; free T4, 0.8-1.8 ng/dL (SI: 10.30-23.17 pmol/L); total T3, 70-200 ng/dL (SI: 1.08-3.08 nmol/L).

Rationale: The goal of antithyroid drug therapy for Graves disease during pregnancy is to keep the serum TSH slightly suppressed and the free T4 high-normal. During pregnancy, the lower limits of normal of serum TSH are 0.1 mIU /L in the first trimester, 0.2 mIU/L in the second trimester, and 0.3 mIU /L in the third trimester (note the pattern). The rationale for these targets is to minimize the amount of antithyroid drug exposure to the fetus by using the lowest effective dosage. Both methimazole and propylthiouracil freely cross the placenta and can affect the fetal thyroid. Thus, the only set of laboratory results within the recommended target range for this patient is listed in Answer B. Answer A is incorrect because these values are consistent with overt hyperthyroidism, which is associated with an increased risk of intrauterine growth retardation and fetal demise. Answers C, D, and E each would be associated with higher dosages of antithyroid drugs than the minimally effective dosage. As an aside, it is important to note that the total T3 level, like the total T4 level, is affected by the increases in thyroxine-binding globulin that occur during pregnancy.

Reference(s): Stagnaro-Green A, Abalovich M, Alexander E, et al; American Thyroid Association Taskforce on Thyroid Disease During Pregnancy and Postpartum. Guidelines of the American Thyroid Association for the diagnosis and management of thyroid disease during pregnancy and postpartum. Thyroid . 2011;21(10):1081-1125. PMID: 21787128

ITE 2017 Question 61 A 20-year-old woman recently underwent total thyroidectomy for papillary thyroid cancer. Her surgical pathology revealed multiple foci of thyroid cancer within the right lobe. The largest focus was 1.1 cm; the other 3 foci were 0.1 to 0.2 cm. Surgical resection margins were negative, no extrathyroidal extension or vascular invasion was identified, and no suspicious lymph nodes were found or removed at the time of her operation. The patient has no family history of thyroid cancer. She was prescribed levothyroxine and referred to you by her surgeon, who had explained that she would be treated with radioactive iodine ( 131I).

Which of the following summarizes what you should discuss with her regarding radioiodine remnant ablation? Correct Answer: E Learning objective : Determine whether radioiodine remnant ablation is necessary to decrease the risk of thyroid cancer recurrence and death.   Effect on Thyroid Cancer Recurrence Effect on Risk of Death From Thyroid Cancer Recommended 131 I Administered Activity A. Decreased Decreased 30 mCi B. Decreased Decreased 100 mCi C. Decreased No change 30 mCiD.DecreasedNo change100 mCiE.No changeNo changeNone

Rationale: This patient has low-risk thyroid cancer regardless of which staging system is used. She has stage I thyroid cancer using the TNM system (see the American Joint Committee on Cancer Staging Manual for differentiated thyroid cancer). Although radioiodine for remnant ablation would render her thyroglobulin undetectable and ablate remnant tissue on radioiodine scanning, there are mixed data as to whether this would reduce her recurrence risk. Her risk of thyroid cancer recurrence is about 2% to 10%. Studies do not consistently suggest that radioiodine would decrease this already low recurrence rate. Regarding the fact that this patient has multifocal disease, data from patients with microscopic multifocal disease show similar recurrence rates in those who receive radioiodine therapy and in those who do not. Although this patient’s largest focus of papillary thyroid cancer is 1.1 cm, her prognosis is likely to be similar to that observed if this focus were 0.9 cm. Radioiodine would be unlikely to reduce her risk of death from thyroid cancer, which is already close to 0%. Thus, in this case, Answer E is correct.

Rationale Continued: If this patient’s course is followed without remnant ablation, her thyroglobulin levels while taking thyroid hormone and the appearance of her postoperation neck, as assessed by ultrasonography, should be periodically monitored. If radioiodine administration were pursued on the basis of more sensitive surveillance, most data would support the use of the lowest possible administered activity of radioactive iodine. Studies suggest that 30 mCi of radioiodine would achieve this goal. Factors that might support the use of radioiodine in this young patient include an aggressive variant of papillary thyroid cancer, involvement of more than 5 cervical lymph nodes 0.2 cm or larger or clinically apparent lymph node metastases, extrathyroidal extension, and incomplete tumor resection. However, only the presence of distant metastases would advance her TNM staging beyond stage I.

Reference(s): Vaisman F, Shaha A, Fish S, Tuttle R. Initial therapy with either thyroid lobectomy or total thyroidectomy without radioactive iodine remnant ablation is associated with very low rates of structural disease recurrence in properly selected patients with differentiated thyroid cancer. Clin Endocrinol (Oxf). 2011;75(1):112-119. PMID: 21521273Haugen BR, Alexander EK, Bible KC, et al. 2015 American Thyroid Association Management Guidelines for Adult Patients with Thyroid Nodules and Differentiated Thyroid Cancer: The American Thyroid Association Guidelines Task Force on Thyroid Nodules and Differentiated Thyroid Cancer. Thyroid. 2016;26(1):1-133. PMID: 26462967Jonklaas J, Sarlis NJ, Litofsky D, et al. Outcomes of patients with differentiated thyroid carcinoma following initial therapy. Thyroid. 2006;16(12):1229-1242. PMID: 17199433 Lamartina L, Durante C, Filetti S, Cooper DS. Low-risk differentiated thyroid cancer and radioiodine remnant ablation: a systematic review of the literature. J Clin Endocrinol Metab . 2015;100(5):1748-1761. PMID: 25679996 Sawka AM, Thephamongkhol K, Brouwers M, Thabane L, Browman G, Gerstein HC. Clinical review 170: A systematic review and metaanalysis of the effectiveness of radioactive iodine remnant ablation for well- differentiated thyroid cancer. J Clin Endocrinol Metab. 2004;89(8):3668-3676. PMID: 15292285Edge SB, Byrd DR, Compton CC, Fritz AG, Greene FL, Trotti A, eds. AJCC Cancer Staging Manual. 7th ed. Springer International Publishing: 2010

ITE 2017 Question 66 A 56-year-old man is referred for evaluation of a neck mass. The patient notes dysphagia with solid foods and positional dyspnea when lying on his right side. His medical history is noncontributory. On physical examination, he has a large goiter extending below the clavicle on the left side. His serum TSH concentration is 0.2 mIU /L, and radioiodine uptake is 12% at 24 hours. CT of the neck is shown (see image).

Which of the following is the best next step in this patient’s management? Levothyroxine suppressive therapy Radioiodine therapy using recombinant human TSH Thermal ablation therapy Thyroidectomy from collar incision No intervention until symptoms progress Correct Answer: D Learning objective : Recommend the best approach to manage a symptomatic substernal goiter.

Rationale: The CT shows a substernal goiter with mass effect on the trachea. The patient is symptomatic, with positional dyspnea, most likely due to the compression of his trachea by the asymmetrically enlarged thyroid when he lies on his side. More than 90% of substernal goiters can be “delivered” through a collar incision (Answer D). The remaining therapeutic options listed are less helpful. Specifically, this euthyroid patient’s thyroid mass is unlikely to respond to levothyroxine suppressive therapy (Answer A) and his serum TSH is already suppressed. Recombinant human TSH treatment (Answer B) with radioiodine in a patient with an intact thyroid is potentially dangerous because of a release of thyroid hormone from the gland under the influence of recombinant human TSH. Thermal ablation (Answer C) would not prove useful in reducing the size of this very large substernal goiter. Proceeding with no intervention (Answer E) is inappropriate given his symptomatic disease.

Reference(s): Stang MT, Armstrong MJ, Ogilvie JB, et al. Positional dyspnea and tracheal compression as indications for goiter resection. Arch Surg . 2012;147(7):621-626. PMID: 22430090

ITE 2017 Question 70 A 27-year-old woman with a history of hypothyroidism after thyroidectomy has been trying to become pregnant and is now 1 week late for her menses. A home pregnancy kit had a positive result. Thyroid function tests 1 month ago documented a serum TSH value of 1.2 mIU /L (0.5-5.0 mIU /L). Her primary care physician requests your management advice.

Which of the following recommendations is consistent with current American Thyroid Association guidelines applicable to this circumstance? Decrease the levothyroxine dosage to achieve a target TSH concentration of 2.5 mIU /L Continue the current levothyroxine dosage Increase the levothyroxine dosage by 30%Increase the levothyroxine dosage by 50% Initiate liothyronine , 5 mcg twice daily, and continue the current levothyroxine dosage Correct Answer: C Learning objective : Recommend that hypothyroid women who become pregnant promptly increase their levothyroxine dosage by approximately 30%.

Rationale: Two recent American Thyroid Association (ATA) guidelines address this circumstance, as cited in the following text. The 2011 ATA Pregnancy and Postpartum Thyroid Management Clinical Practice Guidelines: RECOMMENDATION 13: Treated hypothyroid patients (receiving levothyroxine) who are newly pregnant should independently increase their dose of levothyroxine by 25% to 30% upon a missed menstrual cycle or positive home pregnancy test and notify their caregiver promptly. One means of accomplishing this adjustment is to increase levothyroxine from once daily dosing to a total of nine doses per week (29% increase). Level B-USPSTF The 2013 ATA/American Association of Clinical Endocrinologists Hypothyroidism Clinical Practice Guidelines: When a woman with hypothyroidism becomes pregnant, the dosage of levothyroxine should be increased as soon as possible to ensure that serum TSH is <2.5 mIU /L and that serum total T 4 is in the normal reference range for pregnancy. The remaining answer options either fail to recognize the rapid increase in thyroid hormone requirements in most women during early pregnancy (Answers A and B) or overreact by recommending an increase in the levothyroxine dosage that is potentially too much (Answer D). Initiation or continuation of liothyronine or other T 3 -containing preparations in pregnancy is not recommended due to the poor placental transfer of T 3 .

Reference(s): Garber JR, Cobin RH, Gharib H, et al; American Association of Clinical Endocrinologists and American Thyroid Association Taskforce on Hypothyroidism in Adults. Clinical practice guidelines for hypothyroidism in adults: cosponsored by the American Association of Clinical Endocrinologists and the American Thyroid Association [published corrections appear in Thyroid. 2013;23(2):251 and Thyroid. 2013;23(1):129]. Thyroid. 2012;22(12):1200-1235. PMID: 22954017Stagnaro-Green A, Abalovich M, Alexander E, et al; American Thyroid Association Taskforce on Thyroid Disease During Pregnancy and Postpartum. Guidelines of the American Thyroid Association for the diagnosis and management of thyroid disease during pregnancy and postpartum. Thyroid. 2011;21(10):1081-1125. PMID: 21787128

ITE 2017 Question 76 A 44-year-old man with a remote history of Graves disease is referred for evaluation. Three years earlier, he developed ocular pain, chemosis , and eyelid swelling after receiving radioiodine therapy. Currently he has persistent diplopia and inability to fully close his eyes while sleeping. On physical examination, he has pain with eye movement, but no conjunctival erythema or edema and no lid swelling. Proptosis measures 26 mm on the right side and 28 mm on the left side, with dysconjugate gaze (see image). He takes levothyroxine, and his current TSH concentration is 1.2 mIU/L (0.5-5.0 mIU /L).

Which of the following should be recommended for this patient? Orbital radiotherapy Rituximab therapy Pulse corticosteroid therapy Orbital decompression followed by strabismus surgery Strabismus surgery alone Correct Answer: D Learning objective : Recommend surgical treatment of inactive Graves ophthalmopathy that is unlikely to respond to immunomodulatory therapy.

Rationale: This patient has moderately severe Graves ophthalmopathy manifested by proptosis and extraocular muscle dysfunction. In terms of active inflammation as assessed using the clinical activity score, this would be considered inactive and therefore unlikely to respond to immunomodulatory therapy. The clinical activity score assigns 1 point each for spontaneous retroocular eye pain, pain with eye movement, eyelid swelling, eyelid redness, conjunctival redness, conjunctival edema ( chemosis ), and caruncle edema.Grading Ophthalmopathy With Clinical Activity Score Point Assignment (maximum of 7 total) Pain behind globe 1 pt Pain with eye movement 1 pt Conjunctival redness 1 pt Eyelid redness 1 pt Chemosis 1 pt Swelling of caruncle 1 pt Lid edema 1 pt Adapted from: Classification of eye changes of Graves’ disease. Thyroid. 1992;2(3):235-236.

Rationale Continued: Clinical activity scores greater than 3 or 4 suggest active inflammatory disease that will most likely respond to immunomodulatory therapy such as glucocorticoids. Because this patient does not have active inflammatory changes, the primary treatment options are surgical. He will require both orbital decompression surgery to reduce the degree of proptosis and extraocular muscle surgery to correct his dysconjugate gaze (Answer D). Orbital radiotherapy (Answer A), which may have modest beneficial effects on extraocular eye muscle function, will not relieve his excessive proptosis and it is therefore not the best approach. Rituximab therapy (Answer B), which leads to B-lymphocyte depletion, has recently been studied in 2 randomized prospective trials (showing net benefit in one trial but not the other), and it would not be expected to provide benefit in the absence of active inflammation. Likewise, corticosteroid therapy (Answer C) would not be expected to be beneficial in this setting. Finally, strabismus surgery alone (Answer E) would not relieve the patient’s excessive proptosis, which, in addition to a cosmetic defect, is causing exposure keratitis.

Reference(s): Bartalena L, Baldeschi L, Boboridis K, et al; European Group on Graves’ Orbitopathy (EUGOGO). The 2016 European Thyroid Association/European Group on Graves' orbitopathy guidelines for the management of Graves' orbitopathy. Eur Thyroid J. 2016:5(1):9-26. PMID: 27099835Bartalena L, Tanda ML. Clinical practice. Graves’ ophthalmopathy. N Engl J Med. 2009;360(10):994-1001. PMID: 19264688[No authors listed]. Classification of eye changes of Graves’ disease. Thyroid. 1992;2(3):235-236. PMID: 1422237

ITE 2017 Question 78 While combing her hair, a 50-year-old woman noticed a swelling in her neck and vague discomfort in the area. She has some difficulty swallowing and has also noted hoarseness in her voice. She has no family history of thyroid disease and no personal history of radiation exposure. On physical examination, vital signs are normal. A 3-cm nodule involves most of the right lobe of the thyroid gland. It is quite hard, but it moves with swallowing. The left lobe is small and soft. No lymph nodes are palpable. Laboratory test results: TSH = 2.1 mIU /L (0.5-5.0 mIU /L) Free T4 = 1.5 ng/ dL (0.8-1.8 ng/ dL ) (SI: 19.31 pmol /L [10.30-23.17 pmol /L]) Thyroid ultrasonography (see image) demonstrates a noncalcified isoechoic solid nodule in the right thyroid lobe. Fine-needle aspiration biopsy reveals a very cellular aspirate. Little colloid is present, except in a few dense clumps surrounded by follicular cells. There are multiple tiny ringlets of follicular cells forming microfollicles .

You recommend thyroid surgery.

Which of the following is the most reasonable next step to perform preoperatively? Repeated FNAB targeting the capsule of the nodule 123 I thyroid scan Neck ultrasonography to evaluate cervical lymph nodes Measurement of serum thyroglobulinNo further tests Correct Answer: C Learning objective : Determine the appropriate steps after fine-needle aspiration biopsy of a thyroid nodule yields a follicular neoplasm cytologic result.

Rationale: This discussion pertains to the follicular neoplasm cytologic result obtained by FNAB. Older studies reported that 20% to 25% of thyroid nodules with these cytologic characteristics were found to be malignant at surgery and that these were follicular thyroid cancers, differentiated from benign follicular adenomas by demonstration of capsular, local, or blood vessel invasion on surgically removed specimens. However, more recent studies have demonstrated that only half of malignant nodules with these cytologic characteristics are follicular cancer; the other half are the follicular variant of papillary thyroid cancer. Classic papillary thyroid cancer generally yields cytologic findings demonstrating its characteristic nuclear features, but the follicular variant of papillary thyroid cancer, which accounts for up to 40% of all papillary cancers, may not demonstrate these on cytologic examination. Rather, it can demonstrate microfollicular arrangement of cells that lack these nuclear features. The nuclear features are only seen histologically after surgery. In addition, as seen in this case, the sonographic appearance of both follicular cancer and the follicular variant of papillary cancer is generally isoechoic to hyperechoic compared with the appearance of classic papillary cancer, which is more likely to be hypoechoic.

Rationale Continued: Since there is a possibility that this cytologic result may indicate the presence of the follicular variant of papillary cancer, ultrasonography of the central and lateral cervical lymph nodes should be performed before surgery (Answer C). If metastatic lymph nodes are detected, this will alter the surgical procedure performed. Therefore, no further testing (Answer E) is incorrect. Repeating the FNAB to assess whether capsular invasion is present (Answer A) is not helpful. This is determined by carefully sectioning and examining the final pathologic specimen. Iodine 123I imaging (Answer B) is not indicated in euthyroid patients, and in this vignette the serum TSH concentration is 2.1 mIU/L. In addition, serum thyroglobulin levels (Answer D) in this situation do not distinguish between benign and malignant nodules. Recently, new approaches to nodules with these cytologic characteristics have become available that may allow for better classification into categories of benign vs malignant. Although this was not offered as an option in this case, use of a molecular test could be considered if available. .

Reference(s): Haugen BR, Alexander EK, Bible KC, et al. 2015 American Thyroid Association Management Guidelines for Adult Patients with Thyroid Nodules and Differentiated Thyroid Cancer: The American Thyroid Association Guidelines Task Force on Thyroid Nodules and Differentiated Thyroid Cancer. Thyroid . 2016;26(1):1-133. PMID: 26462967 Nikiforov YE, Seethala RR, Tallini G, et al. Nomenclature revision for encapsulated follicular variant of papillary thyroid carcinoma: a paradigm shift to reduce overtreatment of indolent tumors. JAMA Oncol. 2016;2(8):1023-1029. PMID: 27078145

ITE 2017 Question 82 An 80-year-old man is admitted to the intensive care unit for overwhelming sepsis and multiorgan failure. Despite aggressive therapy, the patient’s condition continues to deteriorate. One month earlier, he had a normal thyroid laboratory panel.

Which of the following patterns is expected in this patient? Answer TSH Total T 4 Total T 3 Free T 4 A. 10.0 mIU/L 7.0 µg/ dL (90.1 nmol /L) 70 ng/ dL (1.1 nmol /L) 0.7 ng/dL (9.0 pmol/L) B. 7.5 mIU/L 5.5 µg/dL(70.8 nmol/L)55 ng/dL(0.8 nmol/L)0.8 ng/dL(10.3 pmol/L)C.0.2 mIU/L2.5 µg/dL(31.2 nmol/L)25 ng/dL(0.4 nmol/L)0.5 ng/dL(6.4 pmol/L)D.5.0 mIU/L12.0 µg/dL(154.4 nmol/L)70 ng/dL(1.1 nmol/L)2.2 ng/dL(28.3 pmol/L)E.0.01 mIU/L12.0 µg/dL(154.4 nmol/L)360 ng/dL(5.5 nmol/L)2.2 ng/dL(28.3 pmol/L)Reference ranges: TSH, 0.5-5.0 mIU/L; total T4, 5.5-12.5 µg/dL (91.02-213.68 nmol/L); total T3, 70-200 ng/dL (1.08-3.08 nmol/L); free T4, 0.8-1.8 ng/dL (10.30-23.17 pmol/L).

Correct Answer: C Learning objective : Identify expected thyroid function test patterns in severe nonthyroidal illness.

Rationale: This patient has been admitted to the intensive care unit for sepsis and multiorgan failure and his condition is deteriorating. He would be expected to have the classic changes in thyroid hormone levels that occur as a result of an acute nonthyroidal illness of this severity. The most prevalent and pronounced change of thyroid function during nonthyroidal illness is a depression in the serum total and free T3 levels and a concurrent elevation in reverse T 3 . A low T 3 level, present in 70% or more of hospitalized patients, may be considered, with a few notable exceptions, the sine qua non of the euthyroid sick syndrome. Total T 4 is frequently low in patients with severe nonthyroidal illness and a very low T 4 portends a poor prognosis. Free T 4 generally remains in the normal range, but may be frankly low in gravely ill or moribund patients, such as the patient in this case. TSH is often normal early in illness, but it may also be suppressed in critical illness and then elevated during the recovery stages (generally it is <20 mIU /L). Using the process of elimination, only Answers B and C have a low T T 3 value. Answer B has a slightly elevated serum TSH, which can be seen largely in the recovery stage after a nonthyroidal illness, but this patient is unfortunately not recovering. Answer C, with more severe alterations in T3, as well as depressions in both total and free T4, is a better fit for this particular moribund patient.

Reference(s): Fliers E, Bianco AC, Langouche L, Boelen A. Thyroid function in critically ill patients. Lancet Diabetes Endocrinol. 2015;3(10):816-825. PMID: 26071885Van den Berghe G. Non-thyroidal illness in the ICU: a syndrome with different faces. Thyroid. 2014;24(10):1456- 1465. PMID: 24845024

ITE 2017 Question 85 A 70-year-old man in whom American Joint Committee on Cancer stage III (T3,N0,M0) follicular thyroid cancer was diagnosed 5 years ago has just moved to your area and presents for a first visit. His initial treatment consisted of total thyroidectomy followed by radioactive iodine therapy with a 100-mCi dose of 131 I. Postoperative histopathologic evaluation of the specimen documented a 4.1-cm widely invasive follicular thyroid carcinoma with numerous foci of vascular invasion. His posttherapy 131I whole-body scan demonstrated uptake in the thyroid bed only, and a recombinant TSH-stimulated radioiodine scan 12 months after his initial treatment was negative. Serial measurements of serum thyroglobulin and thyroglobulin antibodies are detailed (see table).   Date Serum Thyroglobulin Thyroglobulin Antibodies Serum TSH Recombinant TSH Stimulation 4 years ago <0.1 ng/mL (<0.1 μg/L) <4.0 IU/mL (<4.0 kIU/L) <0.01 mIU /L Yes 2 years ago 0.9 ng/mL (0.9 μg/L) <4.0 IU/mL (<4.0 kIU/L)<0.01 mIU/LNo9 months ago8.3 ng/mL(8.3 μg/L)<4.0 IU/mL(<4.0 kIU/L)<0.01 mIU/LYes4 months ago15.6 ng/mL(15.6 μg/L)<4.0 IU/mL(<4.0 kIU/L)<0.01 mIU/LYes

The results of a recombinant TSH-stimulated 131I whole-body scan performed 4 months ago are displayed (see image).

He has noticed some shortness of breath on exertion in recent months, but is otherwise feeling well. His only medication is levothyroxine, 150 mcg daily. On physical examination, his blood pressure is 130/80 mm Hg, and pulse rate is 76 beats/min. There is no palpable thyroid tissue in the thyroid bed and no cervical adenopathy. You perform cervical ultrasonography, which does not reveal any suspicious findings. Laboratory test results: TSH = <0.01 mIU /L (0.5-5.0 mIU /L) Free T4 = 1.8 ng/ dL (0.8-1.8 ng/ dL ) (SI: 23.2 pmol /L [10.30-23.17 pmol /L]) Serum thyroglobulin = 22 ng/mL (<0.1 ng/mL) (SI: 22 µg/L [<0.1 µg/L]) Thyroglobulin antibodies = <4.0 IU/mL (≤4.0 IU/mL) (SI: <4.0 kIU/L [≤4.0 kIU /L])

Which of the following is the best next step in this patient’s care? MRI of the neck and chest with intravenous contrast CT of the neck and chest with intravenous contrast Repeated thyroglobulin measurement and cervical ultrasonography in 3 months 18 F-fluorodeoxyglucose PET imaging (FDG-PET)Diagnostic whole-body radioiodine scan following recombinant TSH stimulation Correct Answer: D Learning objective : Direct appropriate imaging in a patient with differentiated thyroid cancer and rising serum thyroglobulin with negative radioiodine scanning.

Rationale: This patient’s initial risk stratification according to the American Joint Committee on Cancer (AJCC)/TNM staging system is stage III thyroid cancer because he was older than 45 years when he was found to have a 4.1-cm tumor. The use of the AJCC/TNM staging system is based on its utility in predicting disease mortality and its requirement for cancer registries; however, it does not accurately predict the risk of recurrence in differentiated thyroid cancer. A more useful approach is the use of an initial 3-tiered clinicopathologic risk stratification system that classifies patients as having low, intermediate, or high risk of recurrence and/or persistence following thyroidectomy as recommended in the most recent American, as well as the British, Thyroid Association Guidelines. The patient in this vignette would have been classified in the high-risk group on the basis of a 4.1-cm widely invasive tumor with extensive vascular invasion.

Rationale Continued: In accordance with American Thyroid Association and other international guidance, this patient has undergone adjuvant radioactive iodine ablative therapy. Most current American and European guidelines recommend modifying the risk stratification during follow-up because the risk of recurrence and disease-specific mortality can change over time as a function of the clinical course and the response to therapy. A 4-tiered response to therapy or dynamic risk stratification system is recommended in the revised 2015 American Thyroid Association guidelines, which categorizes patients into excellent, biochemical incomplete, structural incomplete, or indeterminate response groups. Twelve months after completion of treatment, this patient would have been categorized in the excellent response group since there was no clinical, biochemical, or structural evidence of disease. Three years after completion of treatment, however, the patient’s unstimulated serum thyroglobulin started to rise progressively and subsequent stimulated serum thyroglobulin measurements indicate that recurrence is likely. This patient can now be categorized as having a biochemically incomplete response. Diagnostic whole-body scanning, however, has not identified any structural evidence of disease. Moreover, there is no clinical or ultrasonographic evidence of disease recurrence.

Rationale Continued: In patients with moderate- to high-risk thyroid cancer, a serum thyroglobulin concentration greater than 10 ng/mL (>10 µg/L), and negative radioiodine imaging (such as the patient in this vignette), 18 F-fluorodeoxyglucose PET imaging (FDG-PET) (Answer D) should be considered to identify the presence of recurrent or persistent disease. 18 FDG-PET is more sensitive in patients with an aggressive histologic subtype, and 18FDG uptake on PET in patients with metastatic differentiated thyroid cancer is a major negative predictive factor for response to radioiodine treatment, as well as an independent prognostic factor for survival. It is complementary to 131I whole-body scanning, even in the presence of detectable 131 I uptake in metastases because 18 FDG uptake may be present in neoplastic foci with no 131 I uptake. In a meta-analysis of 25 studies that included 789 patients, the sensitivity of 18 FDG-PET/CT was 83% (ranging from 50% to 100%) and the specificity was 84% (ranging from 42% to 100%) in non– 131 I-avid differentiated thyroid cancer. Factors influencing 18 FDG-PET sensitivity include tumor dedifferentiation, larger tumor burden, and, to a lesser extent, TSH stimulation. In patients with a TSH-stimulated thyroglobulin value less than 10 ng/mL (<10 µg/L), the sensitivity of 18 FDG is low, ranging from 10% to 30%.

Rationale Continued: With modern PET/CT equipment, the CT scan of the PET/CT is as reliable as CT scans used for radiology, and many lesions can be found on 18 FDG-PET scanning even if no contrast medium has been injected. CT (Answer B) or MRI (Answer A) of the neck and chest will not identify metastatic disease in the abdomen or elsewhere and these imaging modalities are therefore not the best options in this setting. These techniques may be required preoperatively for delineation between lymph node metastases or local recurrence and vessels or the aerodigestive axis, which may not be well visualized on 18FDG-PET in the absence of contrast injection. The patient is unlikely to have 131 I-avid disease given that a recent whole-body scan was negative. Another diagnostic radioiodine scan (Answer E) is therefore unlikely to be informative. Since there has been a progressive rise in the serum thyroglobulin concentration but no structural evidence of disease on neck ultrasonography, repeating these tests after 3 months (Answer C) is unlikely to determine the full extent of the disease and this approach will not detect recurrent or metastatic disease outside the neck.

Rationale Continued: An 18 FDG-PET scan in this patient (see images) indicated the presence of an abnormal cervical node (white arrow*), as well as pulmonary metastases (white arrow**) and mediastinal metastases (white arrow***). Normal metabolic activity in the brain (red arrows) and normal 18 FDG accumulation in the bladder following glomerular filtration (green arrows) are also displayed. *** ** *

Reference(s): Haugen BR, Alexander EK, Bible KC, et al. 2015 American Thyroid Association Management Guidelines for Adult Patients with Thyroid Nodules and Differentiated Thyroid Cancer: The American Thyroid Association Guidelines Task Force on Thyroid Nodules and Differentiated Thyroid Cancer. Thyroid . 2016;26:1-133. PMID: 26462967 Perros P, Boelaert K, Colley S, et al; British Thyroid Association. Guidelines for the management of thyroid cancer. Clin Endocrinol (Oxf). 2014;81(Suppl 1):1-122. PMID: 24989897Leboulleux S, Schroeder PR, Schlumberger M, Ladenson PW. The role of PET in follow-up of patients treated for differentiated epithelial thyroid cancers. Nat Clin Pract Endocrinol Metab . 2007;3(2):112-121. PMID: 17237838